Kaplan Crim Law Flashcards

1
Q

A defendant had an argument with his neighbor. As they were quarreling, the defendant pulled out his penknife intending only to frighten the neighbor. The defendant accidentally slightly nicked the neighbor’s arm with the knife. Unknown to the defendant, his neighbor was a hemophiliac who then died from the cut.What is the most serious crime that the defendant can be convicted of?(A) Murder.(B) Voluntary manslaughter.(C) Involuntary manslaughter.(D) Battery.

A
  1. (C) Choice (C) is the correct answer. Remember the distinction between voluntary and involuntary manslaughter. Voluntary manslaughter involves the intentional killing of another in the “heat of passion” brought about by adequate provocation. Provocation is adequate if it would cause a reasonable person would to lose self-control. Except for the mitigation of this objectively reasonable emotional condition, the intentional killing would be murder. On the other hand, involuntary manslaughter is an unintended homicide where there is an unlawful killing in the commission of an unlawful act not amounting to a felony. Here we have a battery (a misdemeanor at common law) that results in an unintended death. The most serious crime that the defendant can be convicted of is involuntary manslaughter. Choice (A) is incorrect because the defendant only intended to frighten his neighbor (an assault) and did not intend to kill him or to inflict great bodily harm; because there is no malice, the defendant cannot be convicted of murder. Choice (B) is incorrect because this was not an intentional killing. Choice (D) is incorrect. While there was, in fact, a battery, the victim died, and the question prompt asks the student to identify the most serious crime for which the defendant can be convicted.
How well did you know this?
1
Not at all
2
3
4
5
Perfectly
2
Q
  1. A defendant wished to see his high school basketball team win the state championship. During an important game, the defendant pulled out a gun and shot at the leg of a key player on the opposing team. The defendant intended only to inflict a slight wound so that the opposing player would be unable to complete the game. When the defendant fired the shot, he unintentionally hit a player on his own high school team in the chest, killing him instantly.What is the most serious crime that the defendant can be convicted of?(A) Murder.(B) Voluntary manslaughter.(C) Involuntary manslaughter.(D) Battery.
A
  1. (A) At common law, murder is the unlawful killing of another human being with malice aforethought. Malice is found where there was intent to kill, intent to inflict serious bodily harm, reckless indifference to human life, or a death caused during the commission of a dangerous felony. Although the defendant may have intended only to cause serious bodily harm, his actions did, in fact, cause the death of another human being. Such a killing is murder unless justified, excused, or mitigated. The fact that the defendant shot the “wrong” basketball player would not change the outcome, because he was aware (or should have been aware) that the shooting created a serious risk of death or grave bodily harm. Therefore, the malice is present, and the defendant can be convicted of murder, since there is no mitigation, justification, or excuse. Choice (B) is incorrect because voluntary manslaughter involves the intentional killing of another in the “heat of passion” brought about by adequate provocation. Provocation is adequate if it would cause a reasonable person would to lose self-control. There is no adequate provocation here. Choice (C) is incorrect because a homicide will be classified as murder where there is intent to cause serious bodily harm, which was the case here. Choice (D) is incorrect because the victim died.
How well did you know this?
1
Not at all
2
3
4
5
Perfectly
3
Q
  1. A defendant hated his boss, who had recently demoted him to a less prestigious position. Late one afternoon, the defendant saw his boss walking down the hallway. The defendant pulled out a gun and fired four shots at his boss. Although none of the bullets directly hit his boss, one of the shots ricocheted against a wall and struck the boss in the head, killing him instantly.What is the most serious crime that the defendant can be convicted of?(a) Murder.(b) Voluntary manslaughter.(c) Involuntary manslaughter.(d) Battery.
A
  1. (A) This is an example of a “depraved-heart” killing. Be aware of the following types of conduct that have been held to involve a very high degree of unjustifiable risk to human life, upon which to ground a “depraved-heart” murder conviction: (1) firing a bullet into a room which that the defendant knows is occupied by several people; (2) shooting into a caboose car of a passing train or into a moving automobile, necessarily occupied by human beings; (3) throwing a beer glass at one who is carrying a lighted oil lamp; (4) playing a game of “Russian roulette” with an other person; (5) shooting at a point near, but not aiming directly at, another person; (6) driving a car at very high speeds along a main Street; (7) shaking an infant so long and so vigorously that it cannot breathe. Choice (B) is incorrect because voluntary manslaughter involves the intentional killing of another in the “heat of passion” brought about by adequate provocation. Provocation is adequate if it would cause a reasonabLe person would to lose self-control. The defendant’s hatred for his boss does not constitute adequate provocation on these facts. Choice (C) is incorrect because the defendant’s reckless indifference to human life constitutes malice for the purposes of common law murder. Choice (D) is incorrect. While there was, in fact, a battery, the victim died, and the question prompt asks the student to identify the most serious crime for which the defendant can be convicted.
How well did you know this?
1
Not at all
2
3
4
5
Perfectly
4
Q
  1. An owner of a 3-story building, with apartments on the top two floors and a convenience store and gas station on the ground floor, resides with his family in one of the second-floor apartments. One day, a truck driver pulled into the gas station for some gas. After filling up the gas tank, the truck driver entered the convenience store for a bite to eat. He was eating a bowl of chili when he noticed a cockroach in the food. The truck driver demanded a new serving of chili. When the owner refused, the truck driver stormed out of the store in anger.The truck driver jumped into his truck and decided to get even by causing some damage. He purposely drove into one of the gas tanks, which ruptured the gas line. The collision caused a terrible explosion and the building became engulfed in flames. The owner could not escape the blaze and burned to death.At common law, the truck driver should be found guilty of(A) arson only.(B) murder only.(C) arson and murder.(D) arson, burglary, and murder.
A
  1. (C) Extremely negligent conduct which that creates an unjustifiable but also a very high-degree risk of death or serious bodily injury to another or to others (though unaccompanied by any intent to kill or do serious bodily injury), and which that actually causes the death of another constitutes “depraved-heart” murder. By deliberately driving his truck into a gas tank, the truck driver’s conduct created an unreasonable risk of causing death or serious bodily injury. Consequently, he will be guilty of “depraved-heart” murder for the death that resulted, even though he did not intend to kill, only to cause some damage to the owner’s property. In addition, he will be guilty of arson, defined as the malicious burning of the dwelling house of another. The truck driver’s reckless conduct satisfies the malice requirement. Since the owner and his family resided in the second-floor apartment of the convenience store, it will qualify as a dwelling house. Note: on a bar exam essay question, students should also discuss the truck driver’s criminal liability under the felony-murder rule. Answer choices (A) and (B) are incorrect because, as discussed above, he will be found guilty of both arson and murder. Choice (D) is easily eliminated because the truck driver did not break and enter the convenience store with the intent to commit a felony therein.
How well did you know this?
1
Not at all
2
3
4
5
Perfectly
5
Q
  1. A city has enacted an anti-noise statute that “prohibits amplification devices operated in public places which emit loud and raucous noises.” The municipality passed the law to protect the quiet and tranquility of its residents and to avoid distractions to traffic.A high school student was driving in his car and listening to punk music on his car stereo system, which included a set of 12-inch subwoofers mounted in the rear. As the student drove past the high school, he turned up the volume to its highest level. A police car drove alongside the student and, hearing the loud and raucous music; the police officers arrested the student and charged him with violating the anti- noise statute.Before the student was brought to trial, the highest court in the state ruled the statute unconstitutional as vague and overbroad. As a result, the charges against the student were dropped, and his case was dismissed. Thereafter, the district attorney’s office decided to prosecute the student for violating a state attempt statute.The attempt statute provided that legal impossibility was no defense. Thereupon, the student was charged with attempting to violate the anti-noise statute. At trial, the student took the stand to testifi on his own behalf. He testified that when he was arrested, he had purposely turned up the stereo to its highest level because his favorite song was playing.Will the student be found guilty of violating the attempt statute?(A) Yes, because he possessed the requisite intent.(B) Yes, because dismissal of the initial charge was not a final judgment and, therefore, double jeopardy does not attach.(C) No, because double jeopardy precludes prosecution for attempt after the initial prosecution was dismissed.(D) No, because a defendant cannot be retried for attempted commission of a crime that has been ruled unconstitutional by a state’s highest court.
A
  1. (A) The crime of attempt consists of: (1) an intent to commit the “target” offense, and (2) a “substantial step” in the furtherance of the crime. At common law, legal impossibility (but not factual impossibility) was a defense to attempt. The key to this question, however, is recognizing that legal impossibility is not a valid defense under this jurisdiction’s attempt statute. Therefore, the student can be found guilty of attempt despite the fact that the “target” offense (i.e., the anti-noise statute) has been ruled unconstitutional. Choice (B) is a true statement, but that is not a sufficient explanation of why there should be guilt for this attempt. Choice (A) is a better answer than choice (B) because it addresses part of the reason why the defendant should be found guilty. All choice (B) does is explain to you that he could be found guilty without giving any explanation why. Choice (C) is wrong because double jeopardy does not attach. Double jeopardy prohibits re-prosecution for a crime after there has been a final judgment (such as a conviction or acquittal). A dismissal of the initial charge does not constitute a final judgment. Choice (D) is an attractive answer because at common law, this attempt crime would be legally impossible because the law the student attempted to violate is unconstitutional. However, it is wrong, because, as discussed above, the statute legal impossibility is no defense. The common law rule does not support this result. It only works only in the closed universe where the hypothetical statute applies.
How well did you know this?
1
Not at all
2
3
4
5
Perfectly
6
Q
  1. Two college students were members of the same fraternity. During a fraternity party, the defendant pulled out a gun, pointed it at the other student and said, “Beg for your life, sucker.” The student, who knew that the defendant had a reputation as a practical joker, said, “Get lost, you jerk.” The defendant then pulled the trigger and shot the student to death.A statute in this jurisdiction provides: “Any intentional and premeditated killing with malice aforethought is murder in the first degree. Murder in the second degree is any killing that occurs during the commission or attempted commission of a serious or inherently dangerous felony. Manslaughter includes all other types of unlawftil homicide and unjustifiable killings.”The defendant was subsequently prosecuted for killing the student. At trial, the defendant testified that a fellow fraternity member gave him the gun before the shooting and told him that it was unloaded.If the jury believes the defendant, it should find him(A) guilty of assault, but not murder or manslaughter.(B) guilty of manslaughter.(C) guilty of second-degree murder.(D) guilty of first-degree murder.
A
  1. (B) This question requires that the reader properly interpret and apply the statute provided in the fact pattern. On the MBE, the Common Law rule will generally apply, unless the fact pattern or question stem triggers application of a different rule. Here, the statute provided makes choice (B) the best answer. Choice (A) is incorrect because, although the defendant only intended an assault (putting the student in fear of being shot), the victim died. Choice (B) is a better answer than choice (A) because the defendant is guilty of manslaughter which, in this jurisdiction, includes “all other types of unlawful homicide.” This definition thus encompasses misdemeanor-manslaughter, which covers assault or battery causing death situations. At common law, assault and battery were misdemeanors, and any-time the crime of assauLt (or battery) causes an unintentional killing, the defendant is guilty of misdemeanor-manslaughter. Choices (C) and (D) are wrong because the killing was not premeditated or committed during the commission of a serious felony.
How well did you know this?
1
Not at all
2
3
4
5
Perfectly
7
Q
  1. One afternoon, a man was driving his car in the downtown section of a city. Already late for a ‘onsiness appointment, he was becoming very upset because of the heavy traffic. Suddenly, he decided to drive his car along the sidewalk to avoid the traffic jam. While he was speeding down the sidewalk, people were hastily scurrying to avoid his car. His car struck a pedestrian, who had tripped trying to get out of his way, killing her.Which of the following is the best theory regarding the man’s criminal liability?(A) Felony murder, because he was committing an assault with a deadly weapon.(B) Transferred intent, because one is presumed to intend the natural and probable consequences of his acts.(C) Intentional killing, because the man’s conduct was a substantial factor in causing the pedesthan’s death.(D) Commission of an act highly dangerous to life, without an intent to kill but with disregard for the consequences.
A
  1. (D) Extremely reckless conduct, which creates what a reasonable person should recognize as an unjustiflably high degree of risk of death or serious bodily injury to another, constitutes “depraved-heart” murder when that conduct causes the death of another, even though there was no intent to kill or do serious bodily injury. Choice (A) is incorrect. It’s attractive because assault with a deadly weapon would constitute aggravated assault; a felony at common law and potential grounds for a felony murder charge. However, no facts say that the man was committing an assault. He was not trying to put the pedestrian in imminent fear of harm; he drove on the sidewalk to avoid a traffic jam. Choice (B) is incorrect because the doctrine of transferred intent cannot apply here because the man never intended to strike anyone with his car. Choice (C) is wrong. Intent to kill cannot be inferred from the fact that the man’s conduct caused the pedestrian’s death.
How well did you know this?
1
Not at all
2
3
4
5
Perfectly
8
Q
  1. A woman was employed as a sales clerk at a department store. She worked in the women’s lingerie section. One evening around closing time, the woman took a diamond necklace from a cabinetin the jewelry section of the department store and wrapped it in tissue paper. The woman then surreptitiously stuffed the necklace in a shopping bag, intending to remove it when she left the store. Moments later, the store owner approached the woman and told her she needed to stay late in order to do inventory. While the woman was completing inventory, the owner discovered the necklace and called the police.The woman is most likely guilty ofa. attempted larceny. b. larceny.c. false pretenses. d. embezzlement.
A
  1. (B) There are many key distinctions that are repetitively tested on the MBE. Surely, in the area of Criminal Law, students can anticipate being tested on the following distinctions: depraved-heart murder / involuntary manslaughter; larceny / embezzlement; Larceny by trick I false pretenses; attempt I conspiracy; Legal impossibility / factual impossibility; robbery I extortion; mistake of law / mistake of fact. This question tests another popular distinction: larceny I embezzlement. The key to the distinction is that larceny is a crime against possession, and embezzlement is a crime against ownership. For bar exam purposes, remember that lower echelon employees (janitors, secretaries, drivers) will usuaLly have mere custody over their employer’s property, so they are usually guilty of larceny when they steal. On the other hand, higher echelon employees (office managers and corporate officials) are often deemed to be in lawful possession of their employer’s property. Thus, when they steal, their crime is usually embezzlement. Here, the woman, as a low-leveL employee, has no right to possess the necklace (not even temporarily), so she was guilty of larceny when she took it from the cabinet with the intent to keep it. Answer choice (A) is wrong because the crime was complete when she took the necklace from the jewelry cabinet with the intent to steal. Choice (C) is incorrect because there was no misrepresentation of a material fact. Choice (D) is incorrect because, as stated above, the defendant did not have lawful possession.
How well did you know this?
1
Not at all
2
3
4
5
Perfectly
9
Q
  1. A defendant met her friend at the electronics store where the friend worked. Unbeknownst to the defendant, her friend had taken a music player from the display case and stuffed it in her bag earlier in the day. Knowing that employees are subject to search when they leave the store, the friend surreptitiously put the music player into the defendant’s bag.The defendant and her friend proceeded to a local tavern and drank shots of tequila. The friend became intoxicated and forgot to retrieve the stolen music player from the defendant’s bag. After leaving the tavern, the defendant proceeded home. Upon opening her bag, she was surprised to see the music player. Believing that the music player had accidentally fallen into her bag at the bar, the defendant planned to return it the next day. The following day, however, the defendant changed her mind and decided to keep it.The defendant is guilty of(A) receiving stolen property.(B) larceny.(C) receiving stolen property and larceny.(D) neither receiving stolen property nor larceny.
A
  1. (D) One of the most important rules in Criminal Law is the requirement ofa concurrence of mens rea and actus reus. The defendant found the music player in her bag when she returned home from the bar. Believing that the music player had accidentally fallen into her bag, she initially intended to return it. Choice (A) is wrong because, in order to be guilty of receiving of stolen property, the defendant must know that the property is stolen at the time when the property comes into her possession. The defendant was unaware the property was stolen. Choice (B) is incorrect because larceny requires a “trespass in the taking.” It was the friend, not the defendant, who took the music player out of the possession of the storeowner. Choice (C) is incorrect for the reasons stated above. Note: lost or mislaid property can be the subject of larceny, if the person who finds it intends to steal it, and knows who the owner is, or has reason to believe that either the markings on the property or the circumstances of its discovery would reveal the identity of the owner.
How well did you know this?
1
Not at all
2
3
4
5
Perfectly
10
Q
  1. Howard did not know it, but his wife was having an affair with another man. One day Howard came home early from work and caught his wife and the other man in bed together. In a violent rage, Howard picked up a large metal lamp and severely beat his wife and the other man. Both victims were seriously injured and hospitalized.Howard was charged with the attempted murder of the other man. He was acquitted of the crime. After Howard’s acquittal, his wife and the other man died from injuries that were related to the beating. The state now wishes to prosecute Howard for manslaughter for the deaths of his wife and the other man.Howard should be found(A) guilty of manslaughter of his wife, but not guilty of manslaughter of the other man.(B) guilty of manslaughter of the other man, but not guilty of manslaughter of his wife.(C) guilty of manslaughter of both his wife and the other man.(D) not guilty of manslaughter of either his wife or the other man.
A
  1. (C) The prosecution may charge and prosecute Howard for manslaughter of the his wife. The facts indicate that he was responsible for her death. He would likely be guilty of voluntary manslaughter, since he attacked her in the “heat of passion” with adequate provocation after catching her in bed with her lover. Choice (A) is an attractive one because prosecuting Howard for the manslaughter of the other man is more problematic, because as it raises double jeopardy concerns. The facts indicate that Howard was acquitted of the attempted murder of the other man. Thus, there was a “final judgment” on the attempted homicide. The issue is whether this acquittal precludes the state from prosecuting Howard for the manslaughter of the other man. As a general rule, attachment of jeopardy for a lesser included offense bars retrial forthe greater offense (Brown v. Ohio, 1977). Choice (A) is wrong because an exception exists where the state is unable to proceed on the more serious charge at the outset because additional facts necessary to sustain that charge have not yet occurred. Here, for example, the victim did not die until after the prosecution for attempted murder. As a result, the state could not prosecute the defendant on the greater charge at the outset. Therefore, the prosecution is not precluded under double jeopardy from prosecuting Howard for the manslaughter of the other man. Choice (B) is a distracter here. It tempts the reader to confuse the facts. For the reasons stated above, Howard may be found guilty of manslaughter for killing the wife, so choice (D) is incorrect.
How well did you know this?
1
Not at all
2
3
4
5
Perfectly
11
Q
  1. A regular customer at a lounge is aware that the owner customarily keeps a gun hidden behind the bar. One night the customer was sitting at the bar and ordered a club soda. After serving him, the owner walked to the other end of the bar to wait on another customer. As he did so, the customer leaned over the bar and grabbed the gun. The customer pointed the gun at the owner and said, “This is a robbery.. . hand over all your money.”Knowing the gun was unloaded, the owner refused to comply. The customer dropped the gun and ran out of the bar.Besides larceny of the gun, the customer will be convicted of(A) attempted larceny.(B) attempted robbery.(C) assault.(D) assault and attempted larceny.
A
  1. (B) The crime of attempt consists of two elements: (1) a specific intent to commit the “target” offense, and (2) an act that constitutes a “substantial step” in the commission or attempted commission of the crime. Choice (A) is incorrect because the customer is guilty of attempted robbery, not attempted Larceny. Robbery differs from larceny in that the taking must be accomplished by force, violence, or intimidation. By threatening the owner at gunpoint, the customer clearly intended to commit a robbery. Some students may incorrectly think that, since the ownerwas not intimidated, the customer is not guilty of attempted robbery. However, even though thecustomer cannot be guilty of robbery, he nonetheless may be convicted of attempted robbery. Answer choices (C) and (D) are incorrect because larceny, assault, and battery are all a lesser-included offense to robbery.
How well did you know this?
1
Not at all
2
3
4
5
Perfectly
12
Q
  1. A student was a practical joker who liked to perform zany antics. Late at night after studying, he would often walk around the campus dressed only in a raincoat and boxers. As he approached young women, he would flash them by opening his raincoat and exposing himself in his shorts. The student believed that he was committing a crime by flashing himself in front of the co-eds. In this jurisdiction, it is a felony to flash or expose oneself in the nude, so, unknown to the student, what he was doing was not a crime.One night, the student broke into a sorority house intending to flash the sorority sisters. Clad only in his raincoat and shorts, he entered the bedroom of one of the sisters who was lying in her bed studying for an exam. The sister, who knew the student, said, “What are you doing here? Shouldn’t you be studying for finals?” The student then took off his raincoat and responded, “Study this, baby!” He then began to run through the sorority house in his shorts. Shortly thereafter, the police arrived and placed the student under arrest.If the student is prosecuted, he should be found guilty of which, if any, of the following crimes?(A) Burglary only.(B) Attempted violation of the statute.(C) Both burglary and attempted violation of the statute.(D) Neither burglary nor attempted violation of the statute.
A
  1. (D) In order to answer this question, it is necessary to interrelate the following four rules of law: (1) burglary, (2) attempt, (3) legal impossibility, and (4) mistake of Law. Choice (A) is wrong because burglary is defined at common law as breaking and entering another’s dwelling house at night with intent to commit a felony (a misdemeanor will not do) therein. Consequently, the student would not be guilty of burglary because he did not have the required mental state (i.e., intent to commit a felony). Therefore, his mistake of law will be a valid defense, since it negates a mental state required for the crime. Choice (B) is incorrect because legal impossibility would constitute a valid defense to the inchoate crime of attempted violation of the “flashing” statute. Legal impossibility is commonly defined as the case in which the defendant did everything he intended to do but yet had not committed the completed crime. In effect, the student is not guilty of attempt because what beset out to do was nota crime. For the foregoing reasons, choice (C) is also incorrect.
How well did you know this?
1
Not at all
2
3
4
5
Perfectly
13
Q
  1. In which one of the following situations would the defendant most likely be found guilty of felony murder?(A) A baseball fan traveled all night to the stadium, planning to purchase tickets from a scalper at the stadium. However, the fan arrived at the stadium after the game had started, and no tickets were available. He proceeded to sneak around the guard at the entrance gate and illegally entered the stadium. He watched the remainder of the game in the standing room only section. The fan became so enraged when the opposition took the lead in the ninth inning that he pulled out a gun from under his jacket and aimed it at the section of spectators in front of him. Before he could be subdued he had fired three shots, killing one person and severely injuring another.(B) A customer was in a bank filling out a deposit slip when a robber entered. The robber went to the teller’s window and quietly slipped a note to the teller, demanding everything in the cash drawer. The customer, who was unaware of the robbery taking place, went to the adjoining teller’s window and made his deposit. Upon obtaining his receipt, the customer suddenly collapsed and died of a heart attack. When the robber saw this, he panicked and left the bank before the teller could comply with his demand.(C) Two men planned to rob a luncheonette during the noontime rush. At 12:30 p.m., as planned, one of the men entered the luncheonette and pointed a gun at the cashier, who handed over the money. The man left to meet the other man outside as he was standing guard as a lookout. As soon as the man ran out of the door, a police officer tried to grab him. The man evaded the officer for a brief moment. Thinking that the other man had double-crossed him and informed the police of their plans, he fatally shot the other man.(D) A man and a woman had been feuding for years. In order to show him a thing or two, the woman decided to set fire to the man’s house. Before dawn one morning, the woman went to the man’s house and lit a match to gasoline she had poured at the man’s front door. The house caught fire and quickly spread to the man’s bedroom, where the man was able to leap out of the window without injury. A police officer who was driving by saw the woman running away one block from the man’s house. The police officer pulled his car up to the woman to determine if she was involved in setting the fire. Upon seeing the officer, the woman starts to flee the scene. As she is running away, the woman pulled out a revolver and started shooting indiscriminately behind her. The police officer was struck by one of the shots and was killed.
A
  1. (D) The woman in choice (0) would most likely be found guilty of felony murder. At common Law, one whose conduct brought about an unintentional death in the commission of a dangerous felony was guilty of felony murder. Today, the majority of jurisdictions have limited the felony murder rule in regard to the causal connection between the felony and the resulting homicide. More than a mere “but —for” causal relationship is required in most jurisdictions. The death must be the foreseeable and natural result of the felony. In short, whether there is a sufficient causal connection depends on whether the defendant’s felony dictated his conduct which that led to the homicide. Under the circumstances presented in choice (D), the woman was fleeing from the scene of the arson, (an inherently dangerous felony). Escape from the scene of the felony, and evading capture is a natural and foreseeable event, as is the use of force to effectuate the escape, and the police officer’s death was the direct result of being shot by the woman. Thus, the necessary causal connection exists placing the homicide “in the commission or’ the arson. Choice (A) is an example of “depraved-heart” type murder. Choice (B) is incorrect, since a causal connection did not exist between the bank robbery (the felony) and the customer’s fatal heart attack. The customer was not even aware of the bank robbery taking place. Choice (C) is aLso wrong, since the man intentionally shot the co-felon, and would be found guilty of murder. No causal relationship existed between the robbery and the death of the co-felon.
How well did you know this?
1
Not at all
2
3
4
5
Perfectly
14
Q
  1. Late one night, a defendant and a man broke into a jewelry store. As they were looting the store, the owner, who lived across the street, looked out his window and saw additional lights on in the store. The owner ran across the street to his store. Before he arrived, however, the defendant became scared and left the store through a back entrance. When the owner entered the store, the other man hid behind a display counter. As the owner walked toward the cash register, he discovered the man in a crouched position. Startled, the man pulled out a knife and stabbed the owner to death. In this jurisdiction, the applicable burglary statute includes all buildings.If he is subsequently arrested and charged with murder, can the defendant be found guilty?(A) No, because the killing was unintentional.(B) No, because he had renounced his participation in the burglary before the killing.(C) No, because the owner’s death was not a foreseeable consequence of the burglary.(D) Yes, provided that he is also found guilty of burglary.
A
  1. (D) The issue tested here is accomplice Liability to felony murder. There’s also a bit of testing strategy involved. Choice (D) is the best choice because of the way it’s phrased. The words “provided that…” is a contingency. It’s another way of saying “given that…” or “if we presume that” he is also found guilty of burglary…” Indeed, the defendant would have been found to have met all of the elements of burglary in that jurisdiction. He and the other man broke into the store with the intent to rob it. With regard to the defendant’s vicarious responsibility of for the owner’s death, it is often said that all parties are guilty for deviations from the common plan which that are the foreseeable consequences of carrying out the plan (e.g., an accidental shooting during an armed robbery being a typical example of a foreseeable deviation from the plan to rob). Under the established rule, accomplice liability extends to acts of the principal in the first degree which that were a “natural and probable consequence” of the criminal scheme the accomplice encouraged or aided. Therefore, choice (D) is correct, and choice (C) is incorrect because it is a foreseeabLe that one of the felons might use violence to avoid being caught. Choice (A) is easily eliminated because felony murder is one of the scenarios in which an unintended killing constitutes murder. Choice (B) is wrong because a mere change of heart or flight from the crime scene will not effectuate withdrawal from the crime. The accomplice must (1) to repudiate his prior aid, or (2) do all that is possible to countermand his prior aid or counsel, and (3) do so before the chain of events has become unstoppable.
How well did you know this?
1
Not at all
2
3
4
5
Perfectly
15
Q
  1. After waiting in line for two hours to gain entry into a popular nightclub, a man was denied admission because his attire failed to conform to the club’s dress code. When he was refused admittance, the man angrily shouted to the club’s doorman, “You’ll be sorry for this. After I’m through, this club will be reduced to rubble.” Later that same evening, the man returned to the club with two explosives in his possession. He noticed the defendant leaving the club and followed him into a nearby parking lot. As the defendant was about to enter his car, the man grabbed him, pointed a gun and said, “Follow me, you punk, or I’ll blow your brains out.” The man led the defendant to the rear of the club, handed him the explosives, and directed him to throw them through an open window of the club. In fear of his life, the defendant tossed the explosives into the club, causing an inferno that killed 25 people. In this jurisdiction, the applicable arson statute includes all buildings.If the defendant is charged with felony murder for the death of the people in the club, he will most likely be found(A) guilty, because they were killings that occurred during the commission of an inherently dangerous felony.(B) guilty, because duress is not a defense to murder.(C) not guilty, because duress is a defense to arson.(D) not guilty, because the defendant was justified under the circumstances.
A
  1. (C) The defendant is charged under the felony murder rule with arson as the underlying felony. Duress is a defense to arson. A defense to the underlying felony is a defense to a murder charge under the felony murder rule. That is why choice (C) is correct. Choice (A) is easily eliminated because it contains only a conclusory statement with no supporting reasoning. Choice (B) is an attractive choice because it is a true statement. Indeed, throwing explosives into a populated nightclub would, in real life, most likely give rise to a murder charge, for which duress is never a defense. Even though duress is not a defense to murder, it may be a defense to felony-murder if it negates the underlying felony. It is always important to pay close attention to what is being asked. Here, the question is: will the defendant be convicted under a felony murder theory? Since choice (B) does not address felony murder, it is not the best answer. Choice (D) is attractive because it addresses the key issue of criminal liability for the underlying felony. However, choice (D) is incorrect because duress is a defense based on excuse, rather than justification.
How well did you know this?
1
Not at all
2
3
4
5
Perfectly
16
Q
  1. Many years ago, a nuclear power plant had been accused of poisoning local oyster beds with waste. The plant had been shut down for 10 years. Much to Darlene’s chagrin, officials at the plant had announced plans to reopen the plant.Darlene, who was a waitress at a local restaurant, devised a scheme whereby she would contaminate oysters at her restaurant with small amounts of a poisonous substance. Although she didn’t want to seriously injure anyone, Darlene believed that if some people got sick from eating the oysters, they would attribute their illness to nuclear waste from the power plant. To facilitate her plan, she stole some poison from a local pharmacy. She sprinkled it on some oysters at the restaurant where she worked. Mistakenly, she put too much of the poison on some of the oysters. A few of the customers who ate the oysters became sick and nauseated. One person suffered convulsions and died from the poisoning.A statute in the jurisdiction provides: “Any intentional and premeditated killing, or one occurring during the perpetration of a felony, is murder in the first degree. Murder in the second degree is murder with intent to cause serious bodily harm. Manslaughter is an unlawful killing due to recklessness.”Assault with intent to kill or to cause serious physical injury is a felony in the jurisdiction. There are no separate felonies relating to adulterating food or poisoning.The most serious crime for which Darlene can properly be convicted is(A) first-degree murder, because her acts were intentional and premeditated.(B) first-degree murder, because the death occurred during the commission of a felony.(C) second-degree murder, because of the intent to cause physical harm.(D) manslaughter, because her conduct was reckless.
A
  1. (D) On the MBE, follow the statutes given in the facts. In this case, first-degree murder requires an intentional and premeditated killing or a killing caused during the commission of a felony. Second-degree murder requires an intentto cause serious bodily harm, whereas manslaughter requires an unlawful killing due to recklessness. The main issue to be determined is the defendant’s mens rea. Darlene did not intend to kill anyone with the poison, although her plan was premeditated. First- degree murder requires both premeditation and intent. Likewise, the killing did not occur during the commission of a felony, since Darlene’s action did not constitute an assault. Assault is defined in the facts as a felony when it is committed with intentto kill or with intent to cause serious physical injury. Darlene did not possess either of these mental states, since the facts say that she “didn’t want to seriously injure anyone.” Therefore, no first-degree murder conviction is proper, and choices (A) and (B) are incorrect. Answer choice (C), second-degree murder, is incorrect for the same reason, namely that Darlene did not intend to cause serious bodily harm. By process of elimination, choice CD), manslaughter, is correct because her mental state was one of recklessness. Recklessness is generally defined as a high degree of negligence, (i.e., more than ordinary negligence) measured by an objective reasonable person standard, which creates a high and unreasonable degree of risk.
How well did you know this?
1
Not at all
2
3
4
5
Perfectly
17
Q
  1. A woman was married to a very wealthy man who had a generous life insurance policy. Deciding that she had had enough of married life, and desiring to spend more time with her boyfriend, she began to plan an early and unexpected demise for her husband. The woman devised a scheme whereby she would contaminate her husband’s guacamole with a poisonous substance called strychnine. To facilitate her plan, she enlisted the help of her boyfriend. At first, the boyfriend adamantly refused to go along with her scheme, but when the woman threatened to break off their relationship, he reluctantly agreed. The boyfriend, a pre-med student, got the strychnine from the pharmacy by telling them that he was using it for an experiment with rats. The boyfriend then gave the strychnine to the woman, who sprinkled it in her husband’s guacamole. The husband died from the poisoning. A statute in the jurisdiction provides: “Any intentional and premeditated killing or one occurring during the perpetration of a felony, is murder in the first degree. Murder in the second degree is murder with intent to cause serious bodily harm. Manslaughter is an unlawful killing due to recklessness.” Assault with intent to kill or to cause serious physical injury is a felony in the jurisdiction. There are no separate felonies relating to adulterating food or poisoning.The most serious crime for which the boyfriend can properly be convicted is(A) first-degree murder.(B) second-degree murder.(C) manslaughter.(D) no crime.
A
  1. (A) The boyfriend will be liable to the same extent as the woman, as a party to the crime. One is liable as an accomplice to the crime of another if he (1) gives assistance or encouragement or fails to perform a legal duty to prevent it (2) with the intent thereby to promote or facilitate commission of the crime. The boyfriend “agreed to assist” the woman “to facilitate her plan.” As an accomplice, he may be convicted to the same extent as the principal, the woman, for first-degree murder. Choice (A) is correct. Choices (B) and (C) are incorrect because the woman committed first-degree murder. Choice (D) is incorrect for the reasons stated above regarding accomplice liability.
How well did you know this?
1
Not at all
2
3
4
5
Perfectly
18
Q
  1. A registered pharmacist was falsely told by his best friend one day that the friend needed a dangerous drug for an experiment he was conducting with rats in his lab. Unbeknownst to the pharmacist, his friend used the drug to poison his girlfriend. The poison killed her. The pharmacist was aware that his best friend had a volatile relationship with his girlfriend.A statute in the jurisdiction provides: “Any intentional and premeditated killing or one occurring during the perpetration of a felony, is murder in the first degree. Murder in the second degree is murder with intent to cause serious bodily harm. Manslaughter is an unlawful killing due to recklessness.” Assault with intent to kill or to cause serious physical injury is a felony in the jurisdiction. There are no separate felonies relating to adulterating food or poisoning. Finally, it is a misdemeanor in the jurisdiction to distribute drugs without a prescription.The most serious crime for which the pharmacist can properly be convicted is(A) first-degree murder.(B) second-degree murder.(C) manslaughter.(D) selling drugs without a prescription.
A
  1. (D) The pharmacist gave the drug to the friend in violation of the state statute making it a misdemeanor to sell the drug without a prescription. The pharmacist may be convicted of selling drugs without a prescription. An individual is criminally liable as an accomplice if he gives assistance or encouragement or fails to act where he has a legal duty to oppose the crime of another, and fully intends to effectuate commission of the crime. No accomplice liability will arise under the facts because the pharmacist had no knowledge of the friend’s criminal intent when he sold the drug and, therefore, could not have formed the requisite mental state, so choice (C) is therefore incorrect. Choice (A) is incorrect because there was no intent on the defendant’s part to kill the woman. Choice (B) is incorrect because the pharmacist did not intend to commit any serious bodily harm.
How well did you know this?
1
Not at all
2
3
4
5
Perfectly
19
Q
  1. One evening, a defendant set fire to a homeowner’s occupied house. As a result of the blaze, the homeowner’s daughter was killed. The defendant was charged with felony murder on the first count and arson on the second count of the two-count indictment. The jury found the defendant guilty on the first count, but returned a not guilty verdict on the second.The defendant’s attorney’s motion to set aside the guilty verdict on the felony murder charge will most likely be(A) granted, because the guilty verdict is plain error that adversely affects the defendant’s constitutional rights.(B) granted, because the verdicts are legally inconsistent and should lead to an acquittal of both charges.(C) denied, because the verdicts do not amount to a reversible error.(D) denied, because the defendant’s proper remedy is to seek an appellate review for a non-constitutional error.
A
  1. (B) In order for one to be found guilty of murder under the felony-murder rule, he must also be found guilty of the underlying felony. Thus, as in the present example, if a defendant is found innocent of the underlying felony, he cannot be found guilty of felony-murder. Answer choices (A) and (C) are incorrect because constitutional protections are not triggered as an issue here. The issues of harmless error and reversible error arise where there has been some conduct in the criminal process that is violative of one’s constitutionally protected rights. Choice (D) is incorrect because the defendant cannot be legally convicted for felony murder if he is not guilty of the underlying felony. If the jury’s verdict cannot be supported by the law, then the judge must grant the motion to set aside the verdict.
How well did you know this?
1
Not at all
2
3
4
5
Perfectly
20
Q
  1. The night before his bar examination, the examinee’s next-door neighbor was having a party. The music from the neighbor’s home was so loud that the examinee couldn’t fall asleep. The examinee called the neighbor and asked her to please keep the noise down. The neighbor then abruptly hung up. Angered, the examinee went into his closet and got a gun. He went outside and fired a bullet through the neighbor’s living room window. Not intending to shoot anyone, the examinee fired his gun at such an angle that the bullet would hit the ceiling. He merely wanted to cause some damage to the neighbor’s home to relieve his angry rage. The bullet, however, ricocheted off the ceiling and struck a partygoer in the back, killing hr. The jurisdiction makes it a misdemeanor to discharge a firearm in public.The examinee will most likely be found guilty for which of the following crimes in connection to the death of the partygoer?(A) Murder.(B) Involuntary manslaughter.(C) Voluntary manslaughter.(D) Discharge of a firearm in public.
A
  1. (A) The examinee is guilty of depraved-heart murder. Firing a bullet into a room which that the defendant knows is occupied by severaL people constitutes the kind of reckless indifference to human rights that will make a criminal defendant liable for murder, even though the killing was unintentional. Choice (B) is incorrect because the defendant’s reckless indifference to human life goes far beyond the gross negligence that would give rise to a charge of involuntary manslaughter; rather it constitutes malice for the purposes of common law murder. Choice (B) is incorrect because voluntary manslaughter involves the intentional killing of another in the “heat of passion” brought about by adequate provocation. Provocation is adequate ifitwould cause a reasonable person would lose self-control. The defendant’s anger over the noisy party on the eve of his bar exam is not adequate provocation. Choice (D) is wrong. Though there is a law against the discharge of a firearm in public, and the examinee might be charged with violating it, it is not the charge he will face in relation to the partygoer’s death.
How well did you know this?
1
Not at all
2
3
4
5
Perfectly
21
Q
  1. A homeowner had a beautiful beach house overlooking the bay. Although the homeowner and his family lived in the beach house during the summer months, the house was left unoccupied for the remainder of the year. In order to protect the beach house from vandalism while it was vacant, the homeowner installed an automatic spring-gun device. The spring-gun was connected to the front door and calculated to automatically fire at chest level when the door was opened. No warnings were placed on the premises.A local man, knowing that the homeowner’s beach house was unoccupied, decided to burglarize the home one evening. When the local man forced open the front door and was about to enter the premises, the spring-gun automatically discharged, killing him instantly.If the homeowner is subsequently prosecuted and charged with the local man’s death, the most serious crime for which the homeowner will be found guilty is(A) voluntary manslaughter.(B) involuntary manslaughter.(C) murder.(D) assault with a deadly weapon.
A
  1. (C) An actor may never use direct or indirect force capable of causing death or serious bodily injury (including spring-guns and traps) to defend property unless a threat to the actor’s own safety justifies him to use such force as a means of self-defense. The fact that the homeowner set the spring-gun to fire at chest level upon opening the door suggests that he intended to kill (or at least cause great bodily harm to) any intruder. Either mental state is proper grounds for a murder conviction; therefore, choice (C) is the best answer. Choice (A) is wrong because there was no heat of passion brought about by adequate provocation. Choice (B) is incorrect because a homicide will be classified as murder where there is intent to cause serious bodily harm, which was the case here. Choice (D) is wrong because the assault resulted in the local man’s death, so the most serious charge is murder.
How well did you know this?
1
Not at all
2
3
4
5
Perfectly
22
Q
  1. A husband was sitting in his living room when his wife entered and asked what he’d like for dinner. The husband replied, “I’m not hungry. I’m too angry at our next-door neighbor.” The husband had had an argument with his neighbor earlier that day. The husband then said, “I’ve got this burning desire to go next door and beat him up.” As the husband was about to walk out the door, he turned to his wife and said, “You want to come along and watch?” The wife nodded, as if to say okay, and followed him next door.Moments later, the husband rang the doorbell, and the neighbor came to the door. After entering the home, the husband grabbed the nei’ghbor and began punching him in the face. Terribly frightened, the neighbor pleaded with the husband to stop. As the husband continued to hit him, the neighbor turned to the wife and said, “Please tell him to get off me.” The wife, who despised the neighbor, simply stood by and told her husband, “Do it, honey. . . do it.” The husband punched the neighbor repeatedly and afterward threatened to kill him.On a charge of battery, the wife should be found(A) not guilty, because the wife’s hands never made contact with the neighbor.(B) not guilty, because the wife’s mere presence and oral encouragement will not make her guilty as an accomplice.(C) guilty, because, with the intent to have the neighbor beaten, she shouted encouragement to her husband.(D) guilty, because she aided and abetted her husband through her mere presence, plus her intent to see the neighbor beaten.
A
  1. (C) The wife is guilty of battery as an accomplice. An individual is criminally liable as an accomplice if he gives assistance or encouragement or fails to act where he has a legal duty to oppose the crime of another, and fully intends to effectuate commission of the crime. Choice (A) is wrong because even though the wife’s hands never touched the neighbor, she can still be liable as an accomplice because she did encourage the battery. Choice (B) is incorrect because it contravenes the rule above. The wife’s encouragement, combined with her intent to effectuate the crime, will, indeed, make her guilty as an accomplice. Choice (C) is preferred over choice (D) because the wife “aided” and “abetted” her husband by shouting encouragement, not by her mere presence.
How well did you know this?
1
Not at all
2
3
4
5
Perfectly
23
Q
  1. In which of the following situations would the defendant not be guilty of homicide?(A) A defendant came into a bar looking for a fight. He walked up to a victim, tapped him on the shoulder and said, “You bother me. Get out of here.” The victim ignored him, and the defendant proceeded to punch the victim in the face and stab him in the arm with a knife. The victim thereupon took out a knife that had been concealed in his pocket and stabbed the defendant in the right arm. The defendant, fearful that the victim would stab him in the heart, took out a gun and shot the victim to death.(B) A defendant was home in bed with a fever one night, and all the lights in his house were off. A victim, who was scouting the neighborhood that night for a house to burglarize, broke into the defendant’s house through the basement window and went upstairs to the bedrooms to look for jewelry. The defendant, who was not aware that someone else was in the house, was startled when he saw the victim walk past his room toward the stairs leading to the outside doorway. The defendant pulled out a pistol from under his pillow when he saw the victim and shot him to death.(C) A defendant started a joke about the victim’s brother. When word got to the victim about the defendant’s joke, the victim became incensed. He rushed to the defendant’s home, broke open the door and found the defendant preparing dinner in the kitchen. He immediately said, “I’m going to kill you.” The defendant knew that the victim had been convicted of attempted murder several years ago, and he cringed when the victim took out a gun and pointed it at him. The defendant could have easily darted for the open front door and evaded the victim but, instead, he suddenly pulled a knife from the kitchen wall, lunged at the victim, and stabbed him to death. Unknown to the defendant, the victim’s gun was not loaded.(D) A defendant was a reporter for a newspaper and was sent on assignment to another state to cover the story of a mining disaster. He was sitting in his hotel room one evening, trying to get a message to his editor, when a victim knocked at the door. The defendant opened the door, and the victim announced a robbery. The defendant took a lamp from the night table and threw it at the victim. The victim was momentarily stunned, and the defendant then pulled a gun out of his own suitcase and shot the victim to death.
A
  1. (C) Choice (C) is the best answer. The defendant would have the strongest case successfully alleging the defense of self-defense. The facts in choice (C) are sufficient to indicate that the defendant has a reasonable belief that he was in immediate danger of deadly force or serious bodily harm. It is immaterial that the defendant did not know that the victim’s gun was not loaded. As a general rule, one who is not the aggressor in an encounter is justified in using a reasonable amount of force against his adversary when he reasonably believes (a) that he is in immediate danger of unlawful bodily harm from his adversary and (b) that the use of such force is necessary to avoid this danger. The defendant not only had such reasonable belief to justify the use of deadly force, but he also had no duty to retreat from his own home. Be aware of the problems involving the “duty to retreat” when deadly force is used in self-defense. The majority of jurisdictions hold that the defender (who was not the original aggressor) need not retreat, even though he may do so safely, before using deadly force upon an assailant whom he reasonably believes will kill him or do him serious bodily harm. However, even in the minority of jurisdictions that require retreat, the defender (1) need not retreat unless he knows he can do so in complete safety, and (2) he need not retreat from his home or place of business. Choice (A) is wrong because, even though he acted in defense of his life, he cannot assert self-defense as a justification for killing the victim because the defendant was the aggressor. If a person has a reasonable belief that he is in imminent danger of unlawful bodily harm, he may use that an amount of force which that is reasonably necessary to prevent such harm, unless he is the aggressor. Choice (B) is not the best answer. Even though discovering an intruder in his home would reasonably instill fear, perhaps even fear that he might be harmed, the facts do not say that the victim was about to use deadly force against the defendant. Rather, the defendant saw the victim walking away from his room toward the door. Similarly, choice (D) is wrong. Since the victim announced a robbery when he entered, but there is not evidence that suggests that the use of deadly force was imminent.
How well did you know this?
1
Not at all
2
3
4
5
Perfectly
24
Q
  1. A defendant was walking through a park when he decided to rob someone. He hid behind a tree, lying in wait for a victim to approach. Shortly thereafter, a girl was strolling in the park when the defendant suddenly jumped from his hiding place and accosted her. Although the defendant intended only to rob his victim, he punched her in the mouth and she fell to the ground. The defendant then grabbed her purse and fled. Unknown to the defendant, the girl suffered a fractured skull when her head struck the pavement. She subsequently died from her head injuries.Which of the following is the most serious crime for which the defendant can be found guilty?(A) Murder.(B) Felony murder.(C) Involuntary manslaughter.(D) Voluntary manslaughter.
A
  1. (B) As a general rule, one whose conduct brings about an unintended death in the commission or attempted commission of a felony is guilty of felony-murder. In many states, the felony-murder rule is limited in its application to serious felonies that must be dangerous to life. Exam Tip: These felonies can be remembered by the mnemonic BARRK: Burglary; Arson; Rape; Robbery; and Kidnapping. Since the girl’s death occurred during the commission of a robbery, the defendant would be guilty of felony-murder. Choice (A) is not the best answer because felony-murder is a specific type of murder. Choice (B) is the better choice because it is more specific. Choice (C) is an attractive, but incorrect, choice because it tempts the reader to classify the death as misdemeanor-manslaughter because the defendant punched the girl in the mouth, thus committing battery, a misdemeanor. However, the death occurred during a robbery (the defendant took the purse from the girl’s person by force), so he will be liable for the girl’s death under a felony-murder theory. Choice (D) is incorrect because the girl’s killing was unintentional.
How well did you know this?
1
Not at all
2
3
4
5
Perfectly
25
Q
  1. A state statute provides: “Whenever a person knows or should know that he (or she) is being arrested by a police officer, it is the duty of such person to refrain from using force or any weapon in resisting arrest.” Violation of the statute is made punishable by fine and/or imprisonment.One morning, there was a bank robbery in the state. That afternoon, a police officer arrested a suspect who he believed was involved in the crime. However, the police officer and the suspect have given different accounts concerning what happened next.According to the police officer, after the suspect was apprehended, he resisted arrest and hit the police officer in the mouth with his fist. The police officer, who was momentarily stunned, pulled out his nightstick and struck the suspect over the head with it.On the other hand, the suspect claimed that after he was arrested, he cursed at the policeman, whereupon the police officer began hitting the suspect with his nightstick. To avoid being hit again, the suspect hit the police officer with his fist, knocking him down. The suspect was charged with assault.The suspect should be found(A) not guilty, if the arrest was unlawful without probable cause and the jury believes the suspect’s account.(B) not guilty, if the arrest was lawful, provided that the jury believes the suspect’s account.(C) guilty, if the arrest was lawful, regardless which account the jury believes.(D) guilty, if the arrest was unlawful, regardless which account the jury believes.
A
  1. (B) One who is not the aggressor in an encounter is justified in using a reasonable amount of force against his adversary when he reasonably believes (a) that he is in immediate danger of unlawful bodily harm from his adversary and (b) that the use of such force is necessary to avoid this danger. The suspect is being charged with the crime of assault, not with violating the resisting arrest statute. As a result, self- defense may be a valid defense to assault. Therefore, choice (B) is correct. Choice (A) is incorrect. If the jury believes the suspect’s account, he will be found not guilty, irrespective of whether the arrest was lawful or not. Choices (C) and (D) are incorrect for the same reason: If the jury believes that the suspect acted to prevent being hit with the nightstick, the suspect will be found not guilty on the assault charge.
How well did you know this?
1
Not at all
2
3
4
5
Perfectly
26
Q
  1. A boyfriend and his girlfriend were attending a house party. While the party was in progress, a few teenagers from the neighborhood vandalized some of the cars parked outside the house. They broke the headlights and stole the battery from the boyfriend’s car.When the party ended, the boyfriend and his girlfriend left the house and got into his car. The boyfriend, who was about to drive his girlfriend home, was unaware of what had happened. He tried to start the car, but it wouldn’t turn on. Two police officers, who were parked outside the house, watched the boyfriend as he tried to start the car. They then approached the boyfriend and charged him with attempting to violate a local ordinance making it a misdemeanor to drive at night without headlights.The boyfriend’s best defense to the charge is(A) factual impossibility.(B) mistake of fact.(C) entrapment.(D) no requisite intent.
A
  1. (D) The boyfriend is being charged with the crime of attempt. Attempt is a specific intent crime. The boyfriend, having noticed neither the broken headlights nor the missing battery, did not possess the requisite state of mind required for commission of the crime, and so cannot be found guilty of attempt. Choice (A) is wrong because factual impossibility is no defense to attempt where the defendant intends a criminal act but cannot accomplish it because of facts unknown to him at the time of the act. Choice (B) is attractive because the boyfriend was mistaken about his presumption that his car was in the same working condition in which he left it. However, mistake of fact is a defense where it negates the existence of a mental state required to establish a material element of the crime. In otherwords, there would be no crime if the facts were such as the defendant thought them to be. Mistake of fact, is simply not in issue here. Choice (C) is easily eliminated because there was no action on the part of the police officers that would have influenced the boyfriend in any way.
How well did you know this?
1
Not at all
2
3
4
5
Perfectly
27
Q
  1. Two brothers who were in need of money approached their wealthy uncle’s housekeeper and asked her to poison him. The brothers would inherit the uncle’s estate when he died. The housekeeper agreed, but on the condition that they would pay her $10,000 from their inheritance. After the brothers agreed to her demand, the housekeeper decided to place some cyanide in the uncle’s tea one morning. As the housekeeper was preparing the tea, the uncle was visited by his personal physician. When the housekeeper was ready to serve the tea, the uncle asked her to fix some tea for his physician also. The housekeeper did so and then carefully set the tea on the table so that the uncle’s cup was facing him. However, when the physician reached over to get some sugar, he inadvertently took the uncle’s cup with the cyanide and drank it. Seconds later, the physician died from the poison.Which of the following crimes are the brothers guilty of?(A) Conspiracy to commit murder of their uncle only.(B) Conspiracy to commit murder of their uncle and the physician.(C) Conspiracy to commit murder of their uncle, and murder of the physician.(D) Solicitation, conspiracy to commit murder of their uncle, and murder of the physician.
A
  1. (C) The brothers are guilty of conspiracy to murder their uncle. They entered into an agreement to murder their uncle and had the specific intent to carry out their criminal objective. The defendants are also guilty of murder under an accomplice theory of liability, since they asked the housekeeper to kill their uncle. Choice (A) is incorrect because, as discussed, the brothers are also guilty of murder. Choice (B) is wrong because there was neither an agreement nor intent to murder the physician. Choice (D) is incorrect because solicitation merges into the conspiracy crime.
How well did you know this?
1
Not at all
2
3
4
5
Perfectly
28
Q
  1. Under which of the following situations would the defendant most likely be found guilty of murder?(A) A defendant conspired with two other men to rob a store. On the day of the robbery, the three participants drove to the store in a stolen van. In accordance with their plan, the defendant was to wait in the getaway van while the two men conducted their heist. While the two men were inside the store, the defendant decided to renounce his involvement and participation in the scheme and hurriedly drove away in the van. In his haste, the defendant failed to see a pedesthan jaywalking across the street, and struck and killed the pedestrian.(B) A defendant, accompanied by his girlfriend, was attending his law school graduation party. Both the defendant and his girlfriend became highly intoxicated during the party celebration. As the defendant and his girlfriend were leaving the party, the defendant handed his girlfriend the keys to his automobile and directed her to drive home. While driving, the girlfriend suddenly swerved the car across the median strip into the path of an oncoming car, killing the driver of the other vehicle.(C) A defendant, whose driver’s license had been revoked one week previously, was driving on an infrequently traveled country road one night. Knowing that his car brakes were defective, the defendant was traveling at 35 m.p.h. in a zone in which the speed limit was 25 m.p.h. when a victim suddenly darted in front of his car. In an effort to avoid striking the victim, the defendant applied his brakes to no avail. The car struck and killed the victim.(D) One afternoon a defendant was delivering pornographic materials to various adult bookstores in the community. In this jurisdiction, the delivery of pornographic materials constituted a misdemeanor. As the defendant was on his way to a book store for his final delivery of the day, he inadvertently struck a blind man who was crossing an intersection. Six months later, the blind man died as a result of injuries suffered in the accident.
A
  1. (A) Under the theory of felony-murder, the defendant in choice (A) would be found guilty of murder for causing the death of the pedestrian (during the commission or attempted commission of the robbery/burglary at the department store). It should be pointed out that the defendant’s “withdrawal” would be ineffective, so as not to exonerate him of criminal responsibility for the pedestrian’s death. A withdrawal is effective only if communicated to his confederates made in time for his companions to effectively abandon the conspiracy. Thus, notice is insufficient unless it is given to all of the other conspirators. Choice (B) is not the best answer. In order to be found guilty of murder, the defendant must have acted with malice. He must have had the intent to kill, or cause great bodily harm, have acted with reckless disregard for human life, or have caused the death during the commission of a dangerous felony. Because none of these apply here, the defendant is not guilty of murder. Choice (C) is also wrong because the requisite malice is not present. This is an attractive answer choice because the defendant was negligent in driving his car knowing that his brakes were defective. However, negligence, even gross negligence, does not rise to the level of reckless indifference. Choice (D) is incorrect. If the defendant is criminally liable for the homicide because it occurred during the commission of a crime, the crime would be a misdemeanor-manslaughter (involuntary manslaughter).
How well did you know this?
1
Not at all
2
3
4
5
Perfectly
29
Q
  1. A brother and his sister, who was 15 years old, had just left a movie theater late one evening and were walking toward a dimly lit parking lot to get to their car. As they reached the car, a defendant, who was visibly intoxicated, emerged from behind a trash can and approached them. The defendant knocked the brother to the ground and hit him over the head with a trash can, causing the brother to lose consciousness. The defendant then forced the sister into the car and raped her.The defendant is charged with assault with intent to commit rape, based on his attack on the brother. The defendant’s best defense would be which of the following statements?(A) Although the sister was only 15 years old, she appeared to be 16 years old to a reasonable man.(B) The defendant did not intend to rape the sister.(C) The defendant’s intoxication at the time negated the required general intent.(D) It is impossible to prove that the defendant was the perpetrator because the parking lot was dimly lit.
A
  1. (B) This question illustrates a number of Multistate principles: (1) answer the precise question asked; (2) read carefully; (3) select the BEST of the four answers; (4) if you cannot identify any answer as correct, at least eliminate the incorrect choices to increase your odds of choosing the best answer; and (5) in choosing an answer, note thata choice that is only possibly correct is preferred overone that is absolutely wrong. There are three important elements to the question asked: (a) a defense (b) to the crime of assault with intent to commit rape (c) based on an attack on the companion of a rape victim. Choice (A) is incorrect because this selection suggests an unsuccessful mistake of fact defense to the charge of statutory rape. Choice (C) is wrong because the crime involved in the question is a specific intent crime, and intoxication may be a defense only to a specific (not general) intent crime. In other words, choice (C) is an incorrect statement of law. Choice (D) is incorrect, as the mere fact that the lot was “dimly lit” will not preclude a guilty verdict. Consequently, only choice (B) applies, certainly an arguable and conceivably correct interpretation of the crime of assault with intent to rape.
How well did you know this?
1
Not at all
2
3
4
5
Perfectly
30
Q
  1. In which of the following situations is the defendant’s conduct most likely to make him criminally responsible for the victim’s death?(A) A defendant shot a victim in the head. The victim was then taken to a hospital for treatment of the wound. An earthquake later struck the city, causing the hospital to collapse. The victim was crushed to death in the rubble.(B) A defendant and a victim lived in the same apartment building. The defendant knew that the victim was having a love affair with a married woman. One day, the defendant learned that the victim was to be murdered by the married woman’s husband. Although the defendant had ample time to warn the victim, he failed to do so. That night, the victim was stabbed to death by the husband.(C) A victim, who was a gambler, was heavily in debt to the mob. The defendant, who was a mob enforcer, threatened to kill the victim if he didn’t pay up. Frightened, the victim boarded the next airplane flight out of town. The airplane crashed, and the victim was killed.(D) A defendant and a victim were driving to work together when the defendant, without provocation, stabbed the victim in the arm. The defendant then pushed him out of the car. The victim fell along the side of the street and fractured his ankle. Unable to move from the roadway, the victim was fatally crushed a half-hour later by a car driven by a drunk driver.
A
  1. (D) This question deals with defenses to murder. In order to impose criminal liability, the defendant’s conduct must be the “proximate” cause of the crime. Where an intervening cause in the form of an unforeseeable act of God follows a defendant’s criminal act and thereby causes death, the defendant may be relieved of liability. In choice (A), the earthquake caused the hospital to collapse, crushing the victim to death. As a superseding cause, the earthquake will cut off the defendant’s liability, so choice (A) is incorrect. In choice (C) the defendant will not be liable for the victim’s death due to lack of causation, so this answer choice is incorrect. Although the threat to kill the victim may have frightened him enough to board an airplane, the defendant’s act in itself did not proximately cause the victim’s death; the plane crash did. Choice (B) is incorrect because omission to act isa basis to impose criminal responsibility only where there exists a legal duty to act—generally established by contract, by statute, or by relationship. The defendant had no familial or business relationship with the victim such that a duty to warn existed. Therefore, he would not be responsible for the victim’s subsequent death. Choice (D) is correct by process of elimination; the defendant pushed the victim directly out of the car after stabbing him in the arm, thereby causing the fractured ankle. Because of the immobility of the ankle, the victim was unabLe to move from the roadway. Being run over subsequently by a drunken driverwas not only a direct intervening cause of the defendant’s initial conduct, but it was a foreseeable result. Therefore, the claim of causation will not be broken, and the defendant will be criminally responsible.
How well did you know this?
1
Not at all
2
3
4
5
Perfectly
31
Q
  1. A defendant and his college roommate went home together for the holiday vacation. The defendant was to go back to college by himself. The day that the defendant was to drive back to college, his roommate asked him if he would deliver a package to one of their classmates. The defendant agreed, and the roommate gave him a small package, which the defendant placed in his glove compartment.While driving back to school, the defendant was stopped for speeding. When the state trooper asked the defendant for his license and registration, he reached into the glove compartment. As he did, the package that his roommate had given him fell onto the floor. When it hit the floor, the box broke open and a plastic envelope containing a white substance slid out. Seeing that the substance appeared to be cocaine, the state trooper arrested the defendant for possession of illegal narcotics under a state statute making it a felony to willfully possess a controlled substance. The package did, in fact, contain cocaine.If the defendant is later brought to trial for this charge, his best defense is that(A) the defendant didn’t know what was in the package.(B) the roommate didn’t tell him he was carrying illegal narcotics.(C) the package didn’t belong to him. (D) the cocaine was illegally seized.
A
  1. (A) Many statutes defining conduct that is criminal will employ words or phrases indicating some type of bad-mind requirement: “intentionally” or “with intent to,““knowingly” or “with knowledge that . . . ,““fraudulently” or “with intent to defraud,”“willfully,” and so on. In our case, choice (A) is the best answer because if the defendant didn’t know what was in the package, she wouldn’t possess the required mental state for the statutory crime. Choice (B) is a correct statement of fact and, explains why the defendant did not know what was in the package, but it is not the best answer because this statement does not negate the mens rea of the crime. Choice (C) is also factually true, but is no defense to the crime of willful possession of a controlled dangerous substance. Choice (D) is incorrect. First, nothing in the facts say that the cocaine was illegally seized. Second, illegal seizure is not a defense, though it may result in the exclusion of the package at trial.
How well did you know this?
1
Not at all
2
3
4
5
Perfectly
32
Q
  1. A woman was in a public restroom at a mall. While the woman was at the sink, a man entered the restroom and told her not to make a sound. Frightened, the woman ran to an open window in the bathroom and jumped onto the fire escape. As she exited down the fire escape, the woman accidentally fell, bruising her legs.Which of the following crimes, if any, would the man most likely be convicted of?(A) Assault.(B) Battery.(C) Assault with intent to commit rape.(D) No crime.
A
  1. (D) In a majority of jurisdictions, assault includes both (1) attempted battery, and (2) the doing of an act which that places another person in reasonable apprehension of receiving a battery. In those jurisdictions where an assault is limited to an attempted battery, intent to commit a battery (i.e., an intent to cause physical injury to the victim) is required. The second type of criminal assault requires some overt act by the defendant to arouse a reasonable apprehension of bodily harm. Thus, threatening words alone, without any overt act to carry out the threat, or indecent proposals by a man to a woman, not accompanied by any attempt to carry them out without her consent, will not suffice. Choice (B) is incorrect because there was no harmful or offensive touching; there was no contact at all, so the man may not be convicted of battery. Choices (A) and (C) are incorrect because there was no attempted battery; the man did nothing to put the woman in reasonable apprehension of imminent harm.
How well did you know this?
1
Not at all
2
3
4
5
Perfectly
33
Q
  1. A defendant was charged with attempted rape of a victim. The crime allegedly occurred at a party at the defendant’s home. During the party, the defendant invited the victim into his bedroom to show her his tattoos. When she entered his bedroom, the defendant ripped off her blouse and threw her onto his bed. He then jumped on the victim and tried to pull off her skirt. When the victim began to scream, some of the guests rushed into the bedroom and pulled the defendant off the victim. At trial, the defendant testified that he wanted to have sexual intercourse with the victim but he believed that she was consenting. The defendant further testified that he had consumed a pint of whiskey earlier in the evening and was intoxicated at the time the incident occurred.If the jury believes that the victim did not consent but also believes that the defendant, in his intoxicated state, honestly believed that she was consenting, the defendant should be found(A) guilty, because consent is determined by the objective manifestations of the victim and not the subjective beliefs of the defendant.(B) guilty, because voluntary intoxication is no defense.(C) not guilty, because he honestly believed that she was consenting.(D) not guilty, because his belief that she was consenting was reasonable.
A
  1. (C) Voluntary intoxication may be a valid defense for a specific intent crime if it negates the requisite mental state. Attempt is a specific intent crime. Choice (C) is correct because if the jury believes that the defendant thought the victim consented, then they cannot find that he had the requisite mens rea. Choice (B) is wrong, but attractive, because it is a true statement of law; with respect to general intent crimes. If the defendant were charged with the crime of rape, then choice (B) would be correct, because intoxication is not a valid defense for the “general-intent” crime of rape. Choice (A) contains a true statement, but it is incorrect because the defendant’s guilt of the attempt crime does turn on his subjective belief. Choice (D) is incorrect because reasonableness is irrelevant here.
How well did you know this?
1
Not at all
2
3
4
5
Perfectly
34
Q
  1. A man is on trial for rape. The alleged victim testified that she went out to dinner with the man. Afterward, he invited her to his apartment for coffee. Upon entering the apartment, he violently assaulted her. Although she tried to resist, he overpowered and raped her.The man testified that during dinner, he and the victim drank two bottles of Champagne. When they returned to his apartment, he was so intoxicated that he honestly believed that she consented to the intercourse.The jury determined that the victim did not consent to the intercourse. The jury also found that the man, as a result of his intoxication, honestly but unreasonably believed that she was consenting. As a consequence, the defendant should be found(A) not guilty, because he honestly believed that the victim consented.(B) not guilty, because his intoxication negated his criminal intent.(C) guilty, because rape is a general intent crime. (D) guilty, because she did not consent, and his belief that she was consenting was unreasonable.
A
  1. (D) This question involves a two-step approach. First, rape is a general intent crime. Second, mistake of fact can be a valid defense for a general intent crime if it is reasonable. Note that mistake can be a valid defense for a specific intent crime (such as larceny or burglary) whether it is reasonable, or unreasonable, as long as it is honest. Choice (A) is incorrect. Even if the jury believes that the man was honestLy mistaken about the victim’s consent, his mistake was unreasonable, so it will not provide a valid defense to the general intent, the crime of rape. Choice (B) is wrong because, as discussed, rape is a general intent crime. Choice (C) does not address the reasonableness of the man’s mistake. In other words, if the man’s mistake had been reasonable, then he would not be guilty of rape, despite the fact that it is a “general intent” crime.
How well did you know this?
1
Not at all
2
3
4
5
Perfectly
35
Q
  1. A defendant was playing cards with a group of friends at their weekly poker game. Earlier in the evening, one of the players handed the defendant a gun and told him it was unloaded. As a joke, the defendant pointed the gun at one of his friends and pulled the trigger. Unknown to the defendant, the gun was loaded and it discharged. The bullet grazed the friend’s shoulder, slightly wounding him.The defendant was charged with assault with intent to kill. In this jurisdiction, battery is a lesser included offense of assault. At trial, the defendant requested that the court instruct the jury that if his mistake was honest, whether reasonable or unreasonable, it would be a valid defense to assault and the lesser included offense of battery. Conversely, the prosecution wanted the court to instruct the jury that in order for the defendant’s mistake to be a valid defense for either crime, it must be reasonable.Regarding the jury instructions, which of the following statements is most accurate?(A) The defendant is correct with respect to assault, and the prosecution is correct with respect to battery.(B) The defendant is correct with respect to battery, and the prosecution is correct with respect to assault.(C) The prosecution is correct with respect to both the battery and assault charges.(D) The defendant is correct with respect to both the battery and assault charges.
A
  1. (A) With respect to the defense of mistake of fact, it is necessary to distinguish between specific intent and general intent crimes. Any mistake of fact, reasonable orunreasonable, is a defense to a specific intent crime. On the other hand, in order to be a defense for a general intent crime, the defendant’s mistake must be reasonable (namely, the type of mistake that a reasonable person would have made under the circumstances). This question requires that the reader be able to differentiate between assault (a specific intent crime) and battery (a general intent crime). As a consequence, the defendant is correct with respect to assault, and the prosecution is correct with respect to battery. A mistake, even an unreasonable one, negates the specific intent required for an assault conviction, but with respect to the battery, mistake can only be a defense if the mistake was reasonable. Forthe foregoing reasons, choices (B), (C), and (D) are incorrect.
How well did you know this?
1
Not at all
2
3
4
5
Perfectly
36
Q
  1. A wife was unhappily married to her husband, an alcoholic. When the husband got drunk, he would become abusive and beat the wife. During the course of their marriage, the beatings became more violent and more frequent. Unable to endure the physical abuse any longer, the wife hired a hit man to kill her husband. One night while the husband was asleep, the hit man entered the home with a key given to him by the wife and shot the husband to death. Afterward, the wife was prosecuted for murder as an accomplice.At trial, the prosecution presented the facts outlined above. Thereafter, the defense sought to have the judge instruct the jury that the wife acted in self- defense under the battered women’s syndrome.Based on the given facts, should the judge give such an instruction to the jury?(A) No, because the wife’s belief in the necessity of deadly force in self-defense was unreasonable.(B) No, because even though the wife was the victim of spousal abuse, she could have avoided the danger by safely retreating.(C) Yes, because, on these facts, a reasonable jury could conclude that the wife acted in self- defense by using necessary force to protect herself from the husband’s constant violent attacks.(D) Yes, because a criminal defendant’s Sixth Amendment right to a jury trial prohibits a court from refusing to submit affirmative defenses to the jury.
A
  1. (A) A person is privileged to use deadly force in self-defense if (a) she reasonably believes that she is in immediate danger of death or serious bodily injury, and (b) the use of such force is necessary to avoid this danger. The given facts indicate that the wife was not in immediate danger of unlawful bodily harm. In fact, the husband was asleep when she arranged to have the hit man kill him. The judge should not instruct the jury on self-defense because the wife has failed to show that she was in immediate danger of serious bodily injury when the murder occurred. Choice (B) is wrong. Not only does the retreat doctrine not apply in one’s own home, it does not apply here because, as discussed, the wife was not in danger of immediate harm. Choice (C) is wrong because it relies on a legally incorrect presumption that self defense may be asserted against intermittent attacks, when it is, in fact, limited to situations where one must use force to protect herself oneself from immediate danger of death or great bodiLy harm. Choice (D) discusses an issue that is not relevant on these facts.
How well did you know this?
1
Not at all
2
3
4
5
Perfectly
37
Q
  1. A woman and a defendant entered into an arrangement where the woman promised to pay the defendant $10,000 to act as a surrogate mother. In return, the defendant agreed to be implanted with the woman’s embryo and carry the baby to term. The woman paid the defendant the $10,000 upfront.During the seventh month of the pregnancy, the defendant changed her mind and decided to keep the child herself. The defendant moved out of state and gave birth to the baby, which she refuses to turn over to the woman.The defendant is guilty of(A) no crime.(B) embezzlement.(C) kidnapping.(D) false pretenses.
A
  1. (A) Sometimes the best way to answer Multistate questions is by process of elimination, especially when the correct answer choice isn’t obvious. Choice (B) is wrong because embezzlement covers the misappropriation of either personal or real pro perty. Embezzlement is broader than larceny, which, at common law, was limited to the theft of personal property. Nonetheless, the defendant is not guilty of embezzlement because we are dealing with a child, not personal or reaL property. Choice (D) is incorrect because the original English false pretenses statute covered only “money, goods, wares, or merchandise,” and thus was limited to tangible personal property and money. By the same token, the defendant is not guilty of kidnapping, which is the forcible abduction of a person. Since we are not deaLing with abduction by force, choice (C) is incorrect. Therefore, by process of elimination, choice (A) is the best answer.
How well did you know this?
1
Not at all
2
3
4
5
Perfectly
38
Q
  1. A man, a woman, and their son went to their neighbor’s house. The man intended to take back some tools that he believed were his and that the neighbor was keeping unlawfully. The woman believed that the tools were the man’s, and she intended to help the man take them. When the son learned that the man and the woman were going to break into the neighbor’s home, he decided to accompany them. The son planned to find some items inside that might be worth taking.Arriving at the neighbor’s home, the man opened the front door, which was closed but unlocked. Upon entering, the son went to the neighbor’s upstairs bedroom and found a watch, which he took. In the meantime, the man and the woman went to the garage and began rummaging through the neighbor’s tools. The man found the tools, which he seized. The three of them then left the neighbor’s home.In this jurisdiction, burglary is defined as the breaking and entering of any structure with the intent to commit a felony therein.Which, if any, individuals should be found guilty of conspiracy?(A) The man, the woman, and the son.(B) The man and the woman.(C) The woman and the son.(D) None.
A
  1. (D) The Common Law definition of conspiracy requires (1) an agreement between two or more persons (which constitutes the act), and (2) an intent to achieve a criminal or unlawful objective. To constitute conspiracy at common Law, there must be a combination of two or more guilty persons. Based on these facts, the “plurality” requirement is not satisfied, because the man and the woman lacked the intent to achieve a criminal objective. Both believed the man was the true owner of the tools. Therefore, neither the man nor the woman should be found guilty of conspiracy. The son intended to commit a crime when he decided to accompany the man and the woman in breaking in to the home, but he never entered into an agreement with the man or the woman to commit a theft inside the home. Since he has no co-conspirator, the son should not be found guilty of conspiracy. For the foregoing reasons, answer choices (A), (B), and (C) are incorrect.
How well did you know this?
1
Not at all
2
3
4
5
Perfectly
39
Q
  1. A driver drove his car into the full-service area ofa gas station. He told the attendant, “Ten gallons,please.” The attendant went ahead and pumped10 gallons of gas into the driver’s tank. When theattendant approached the driver for payment, thedriver drove off without paying.The driver is guilty of(A) larceny.(B) larceny by trick.(C) false pretenses.(D) embezzlement.
A
  1. (B) The distinction between obtaining possession and obtaining title is the principal dividing line between larceny by trick and the crime of false pretenses. The crime of false pretenses requires that the defendant, by his lies, obtain title to the victim’s property. If he obtains possession without title by means of his lies, his crime is larceny by trick. In Hufstetler v. State, 37 Ala. App. 71 (1953), the defendant’s conviction for larceny by trick was affirmed where he (the driver) suddenly drove off without paying for the gasoline. In this particular situation, the court held that the defendant got possession but not title because the fraud vitiated the (gas station) owner’s consent. Because the fraud (i.e., the deception, here, the driver’s implication that he would pay) is said to vitiate consent, the fraud is the reason that the attendant pumped the gas. Thus, the driver has obtained possession, not ownership of the gasoline. Because both the crimes of false pretenses and embezzlement are crimes against ownership, choices (C) and (D) are incorrect. Choice (A) is not larceny because it was not a simple “trespassory taking”; instead, the driver told a lie to get the attendant to pump the gas.
How well did you know this?
1
Not at all
2
3
4
5
Perfectly
40
Q
  1. A boyfriend stole a diamond necklace that he gave his girlfriend as a birthday present. At the time the boyfriend gave the necklace to his girlfriend, she did not know that it was stolen. Three weeks later, while the boyfriend and his girlfriend were lying in bed, she whispered in his ear, “Gee, darling, I really love the diamond necklace you gave me. . . it must have cost a fortune.” The boyfriend responded, “Honey, the necklace didn’t cost me anything. . . I stole it.” Startled by her boyfriend’s confession, the girlfriend broke down and started crying. Moments later, however, after regaining her composure, the girlfriend decided to keep the necklace.The girlfriend is guilty of(A) receiving stolen property.(B) larceny.(C) larceny by trick.(D) no crime.
A
  1. (D) This is another question that illustrates the helpfulness of eliminating the incorrect answers. Choice (A) is the most attractive answer, as the boyfriend did give his girlfriend a stolen necklace, and she did, in a literal sense “receive” it. However, choice (A) is incorrect because the girlfriend lacked the requisite mens rea for the crime of receiving stolen property. She would have to have known that the necklace was stolen at the time she received it. Receiving stolen property is statutorily defined, but in most states (1) the property must be received; (2) it must have been previously stolen; (3) the person receiving the property must know it was stolen; and (4) the receiver must intend to deprive the owner of his or her property. Choice (B) is wrong. The crime of larceny requires the trespassory taking and carrying away of the personal property of another with the intent to permanently deprive the owner. The mental state of intent to steal must concur in time with the act of taking and carrying away. In this question, the boyfriend, not the girlfriend, performed the act of stealing the diamond necklace. The girlfriend decided to keep it three weeks Later. Therefore, she cannot be guilty of common law larceny. At most, the girlfriend is an accessory after the fact. Since the girlfriend did not obtain possession of the neckLace by lying to someone, she cannot be guilty of larceny by trick, so choice (C) is wrong. By process of elimination, choice (D) is the best answer.
How well did you know this?
1
Not at all
2
3
4
5
Perfectly
41
Q
  1. One afternoon, a woman was having lunch with her boss when the boss excused herself to go to the bathroom. As the boss stood up to leave the table, her wallet fell out of her pocketbook onto the floor. The boss was unaware of what occurred and proceeded to the bathroom. The woman, however, saw the wallet fall. Intending to steal it, the woman picked up the wallet and placed it in her pocket. Before the boss returned to the table, the woman had a change of heart and decided to give the wallet back. Thereupon, the woman told her boss what had happened and handed her the wallet when she returned from the bathroom.The woman is guilty of which, if any, crime?(A) No crime.(B) Larceny.(C) Embezzlement.(D) False pretenses.
A
  1. (B) Commission of the crime of larceny requires a taking (caption) and carrying away (asportation) of another’s property. A taking occurs when the offender secures dominion over the property, while a carrying away requires some slight movement of the property. Once the woman picked up the wallet (with the intent to steal) and placed it in her pocket (sufficient asportation), she committed the crime of larceny despite the fact that she later returned the property. The woman later had a “change of heart” and returned the wallet to her boss, but that would not constitute a valid defense. For this reason, choice (A) is incorrect. Both choices (C) and (D) are wrong because the crimes of false pretenses and embezzlement are both crimes against ownership, and the woman never had the right to possess the wallet.
How well did you know this?
1
Not at all
2
3
4
5
Perfectly
42
Q
  1. A defendant entered a department store and took the elevator to the eighth-floor toy department. He went there intending to purchase a new game. The defendant, who only had $8.00 in his possession, saw that the game he wanted was selling for $10.00 Realizing that he did not have enough money to pay for the game, he ripped the $10.00 price tag off the box. While no one was looking, the defendant then took the $7.00 sticker from another game and placed it on the box. He then purchased the game for $7.00 and walked out of the store.Which one of the following crimes may the defendant be convicted of?(A) False pretenses.(B) Larceny.(C) Deceit.(D) Conversion.
A
  1. (A) Choice (A) is correct because the crime of false pretenses requires that the defendant by his lies obtain title to the victim’s property. Choice (B) is incorrect. If one obtains possession without title by his lies, his crime is larceny. At common Law, larceny consists of (1) trespassory (2) taking and (3) carrying away of the (4) personal property (5) of another (6) with intent to steal it. In this regard, larceny by trick is simply one way of committing the crime of larceny; it is not a crime separate from larceny. Answer choices (C) and (D) are easily eliminated because both refer to intentional torts.
How well did you know this?
1
Not at all
2
3
4
5
Perfectly
43
Q
  1. An owner took his car to an auto mechanic to have the oil changed. When he returned to pick up his car later in the day, the mechanic told him the charge was $1 The owner objected and said the charge was excessively high. The mechanic indicated that the amount was reasonable and showed the owner a price listing substantiating the charge. As the two men were arguing, the mechanic excused himself to answer the telephone in the rear of his shop. While the mechanic was on the phone, the owner got into his car and drove off without paying the servicing charges.If the owner is subsequently charged with larceny, he should be found(A) not guilty, because the car was his own property.(B) not guilty, if the jury finds the servicing charge to be excessively high.(C) guilty, if the jurisdiction has a statute making theft of services a crime.(D) guilty, because he took the car without the mechanic’s consent.
A
  1. (C) At common law, larceny was limited to the taking (and carrying away) of tangible personal property. Modern statutes in most jurisdictions have broadened the scope of larceny to include the theft of Labor or services or the use of property. Consequently, choice (C) is correct, and the owner is guilty of larceny if the jurisdiction has a statute making theft of services a crime. Remember to apply the common law unless the question provides a statutory definition. Choice (A) is incorrect because the owner is being charged with larceny of the services, not his own car. Choice (B) is wrong. Even if the service charge was excessively high, the owner did not have the right to drive off without paying. Choice (D) presents a true statement, but, as discussed above, at common law, larceny is limited to tangible personal property. The owner can only be found guilty of larceny of the mechanic’s services in a jurisdiction that has statutorily defined larceny to include the theft of services; so choice (C) is preferred over choice (D).
How well did you know this?
1
Not at all
2
3
4
5
Perfectly
44
Q
  1. In most states, the division of homicide into degrees is distinguished according to which of the following?(A) the causal relationship between the defendant’s act and the resulting death.(B) the attendant circumstances surrounding the death.(C) the nature of the act causing the death.(D) the defendant’s state of mind at the time the killing was committed.
A
  1. (D) It is important that students understand the common law definition of murder as distinct from statutory definitions of murder. This important concept is frequently misunderstood. At common law, there are no degrees of murder. “Murder is the unlawful killing of a human being with malice aforethought.” This is the common law rule. Malice can be found where there Is: (1) intent to kilL, (2) intent to inflict great bodily harm, (3) reckless indifference to human life, or (4) a killing committed in furtherance of a dangerous felony (felony murder). At common law, there are no degrees of murder. Degrees of murder are statutorily defined. Since the common law is the default rule, when approaching a question about murder, you needn’t discuss consider degrees of murder, unless the question prompts you to do so. In most jurisdictions that distinguish between first-and second-degree murder, first-degree murder will include the premeditated variety, as well as felony murder. Everything else is second-degree. Thus, choice (D) is the best answer. Choice (A) is wrong. Although causation is always required in homicide cases, the causal relationship is irrelevant to the distinction between first-and second-degree murder. Answer choice (B) is easily eliminated, since “the attendant circumstances” is so vague as to have no meaning at all. Choice (C) is incorrect because the key distinction is the defendant’s mental state, not her actions.
How well did you know this?
1
Not at all
2
3
4
5
Perfectly
45
Q
  1. A law school professor was hired to teach a bar review course. The professor taught the course in three cities. After conducting the sessions, the professor submitted an expense report to the owner of the bar review course. Her expense voucher included a $225 travel expenditure to a fourth city where the professor had attended a symposium unrelated to the bar review course.The professor also submitted the $225 travel expenditure to the administrator for the faculty at the law school where she teaches. The dean of the law school had previously approved the professor’s appearance at the symposium. The owner paid the professor for the symposium-related expenditure but indicated that the expense item was improperly billed to his company.The professor was indicted for false pretenses. In her defense, she contends that the double billing was the result of a bookkeeping error. Her defense should be(A) valid, if her mistake was reasonable.(B) valid, if she didn’t know that the billing to the bar review company included the symposium expenditure.(C) invalid, if her bookkeeping error was unreasonable.(D) invalid, because white-collar crime imposes absolute criminal liability.
A
  1. (B) In order to answer this question correctly, the reader must know the elements of the crime of false pretenses. False pretenses involves: (1) a false representation of a material present or past fact (2) which causes the victim (3) to pass title to (4) his property to the wrongdoer, (5) who (a) knows his representation to be false, and (b) intends thereby to defraud the victim. Answer choice (B) is the best answer here because the defendant must intend to defraud in order to be guilty of false pretenses. If the professor did not know that the bill to the bar review company included the expenditure for the symposium, then she didn’t have the requisite meris rea, and her defense would be valid. Choices (A) and (C) are wrong for the same reason. Mistake of fact to a specific intent crime, even if unreasonable, will be a valid defense, so long as it negates the required mental state. Choice (D) is easily eliminated, as there is no strict liabiLity for specific intent crimes.
How well did you know this?
1
Not at all
2
3
4
5
Perfectly
46
Q
  1. Late one night, co-defendants broke into a warehouse and proceeded to load the large crates of appliances onto their truck. As they were leaving, one of the co-defendants inadvertently threw his cigarette butt into a refuse pile of old cardboard boxes and papers. Soon afterward, the refuse ignited into a small fire. Although the co-defendants had time to douse the fire without any danger to themselves, neither did so. Instead, they climbed into the truck and fled. Meanwhile, the fire quickly spread, engulfing the entire warehouse in flames.At common law, the co-defendants should be found guilty of(A) burglary and arson.(B) larceny and arson.(C) larceny only.(D) burglary, larceny, and arson.
A
  1. (C) At common law, the defendants could only be convicted of larceny, which is the taking and carrying away of the personal property of another by trespass with intent to permanently deprive that person of his interest in the property. The defendants could not be convicted of burglary or arson since, at common law, both offenses required commission in the dwelling house of another. Choice (A) is wrong because it includes burglary, and that is incorrect for the reason stated above. Choice (B) is incorrect because it includes arson, and that is wrong for the reason stated above. Choice (D) is incorrect for the reasons stated above.
How well did you know this?
1
Not at all
2
3
4
5
Perfectly
47
Q
  1. Two men were told by the defendant that a medical devices company was now using the old warehouse on the river for storing its supply of defibrillators. Late one night, the two men broke into the warehouse and proceeded to load the large crates of defibrillators onto their truck. As they were leaving, one of the men inadvertently threw his cigarette butt into a refuse pile of old cardboard boxes and papers. Soon afterward, the refuse ignited into a small fire. Although the men had time to douse the fire without any danger to themselves, they did not do so. Instead, they fled. Meanwhile, the fire quickly spread, engulfing the entire warehouse in flames.If the defendant is later charged as a co-conspirator, in all likelihood he would be held responsible as(A) an accessory before the fact.(B) a principal in the second degree.(C) an accomplice.(D) not respbnsible.
A
  1. (D) Since the defendant only furnished factual information concerning the contents of the warehouse without knowledge or participation in the planned scheme to commit the burglary, he cannot be held as a co-conspirator, accessory before the fact, or principal in the second degree. Choice (A) is incorrect for the reason stated above. Choice (B) is incorrect because a principal in the second degree is typically an accessory. Choice (C) is incorrect for the reasons stated above.
How well did you know this?
1
Not at all
2
3
4
5
Perfectly
48
Q
  1. A defendant decided to rob a bar because he knew that the bar was now utilizing a new security system that was faulty and that nighttime was the best time to break in.Late one night, the defendant broke into the bar and proceeded to load large kegs of beer onto his truck. As he was leaving, the defendant inadvertently threw his cigarette butt into a trash can behind the bar. Soon afterward, the trash ignited into a small fire. Although the defendant had time to douse the fire without any danger to himself, he did not do so. Instead, he climbed into the truck and drove away. Although the fire spread beyond the trashcan, the fire department responded quickly and was able to put it out. The floor behind the bar was charred. The statutes for both burglary and arson in this jurisdiction include residential and business structures.The defendant should be convicted of which of the following crimes?(A) Arson, burglary, and larceny.(B) Arson and burglary.(C) Larceny and burglary.(D) Arson, robbery, and burglary.
A
  1. (A) Choice (A) is correct because modern statutes have broadened the property, which, if burned and/or burglarized, include buildings. ALthough the defendant did not intend originally to set fire to the pub, an arson was nevertheless committed by his failure to douse the fire without endangering himself. Charring is sufficient for arson. Keep in mind, however, that blackening is not. In addition, the defendant should also be convicted of larceny because it was a trespassory taking and carrying away of the property of another with the intent to permanently deprive the victim of the property. Choice (B) is incorrect because the defendant is also guilty of larceny, and that makes choice (A) a more complete answer. Choice (C) is incorrect because the defendant is also guilty of arson for the reason stated above. Choice (D) is incorrect because there is no robbery here, as the defendant did not commit larceny from a person with the use of force or threat of force.
How well did you know this?
1
Not at all
2
3
4
5
Perfectly
49
Q
  1. A state has a statute defining burglary as “the breaking and entering without privilege of any building or occupied structure with the intent to commit a felony therein.” Late one night, the defendant broke into a warehouse located in the largest óity in the state. He carried with him an incendiary device with which he intended to commit arson. After breaking a window and putting his leg through the glass, the defendant was immediately arrested by a security guard.The defendant should be found guilty for which of the following crimes under modem law?(A) Burglary.(B) Burglary and attempted arson.(C) Burglary, attempted arson, and robbery.(D) Criminal damage to property.
A
  1. (B) In accordance with the state burglary statute, the defendant is guilty of burglary because he (1) broke into and (2) entered (3) the warehouse (4) with the intent to commit a feLony therein. The defendant should also be found guilty of attempted arson because (1) he intended to commit arson and (2) engaged in an act constituting a “substantial step” in the commission of the crime — which consisted of the defendant’s breaking into the building with an incendiary device in his possession. The Model Penal Code sets forth several categories of conduct which that are sufficient as a matter of law to be corroborative of the actor’s criminal purpose. Model Penal Code Section 5.O1(2)(O hoLds that it is a “substantial step” to be in “possession of materials to be employed in the commission of the crime, which are specially designed for such unlawful use or which serve no lawful purpose of the actor under the circumstances.” Choice (A) is incorrect because the defendant is also guilty of attempted arson, as stated above. Choice (C) is incorrect because there was no taking from a person by force or threat of force. Choice (D) is incorrect because this would be a misdemeanor, and the defendant is guilty of far more serious crimes, as stated above.
How well did you know this?
1
Not at all
2
3
4
5
Perfectly
50
Q
  1. A state has a statute defining burglary as “the breaking and entering without privilege of any building or occupied structure with the intent to commit a felony therein.” Late one night, a burglar broke into a jewelry store located in the state. She was immediately arrested by the night security guard. When the burglar was apprehended by the security guard, she offered him $500 to let her go. The security guard agreed. The burglar then handed him the money and took with her the jewelry she intended to steal.If the security guard is later arrested, he should be found guilty for which, if any, of the following crimes?(A) Conspiracy to commit burglary.(B) Accessory before the fact to burglary(C) Accessory after the fact to burglary.(D) No crime.
A
  1. (C) Here, the defendant shouLd be found guilty as an accessory afterthe factto burglary. According to LaFave, there are three basic requirements which that must be met to constitute one as an accessory after the fact: (1) a completed felony had theretofore been committed by another; (2) he knew of the commission of the felony by the other person; and (3) he gave aid to the feLon personally for the purpose of hindering the felon’s apprehension, conviction, or punishment. Choice (A) is incor rect because there was no specific intent on behalf of the defendant to commit the underLying crime of burgLary. Choice (B) is incorrect because you don’t have facts that the burglar committed another burglary. Choice (D) is incorrect for the reasons stated above.
How well did you know this?
1
Not at all
2
3
4
5
Perfectly
51
Q
  1. In which situation would the defendant most likely be guilty of murder?(A) As a practical joke, the defendant throws a firecracker in a crowded movie theater. A victim, who has a weak heart, becomes frightened and thinks someone is firing a gun. The victim suffers a heart attack and dies.(B) The defendant hates her husband after learning that he is having an affair with her best friend. She also knows that her husband owes a man a large amount of money, which he refuses to pay. Before the husband is about to leave for work, the defendant looks out the window and sees the man hiding outside with a gun. The defendant decides not to warn the husband, who then walks outside and is shot and killed by the man.(C) The defendant and his friend attended a party together. At the party, the friend became extremely inebriated. The friend asked the defendant if she could borrow his car to drive to the liquor store to buy more booze for the party. Although the defendant was aware of the friend’s intoxicated condition, he allowed her to drive his car to the liquor store. The friend drove through a red light and collided with the victim’s car. The victim was killed in the accident.(D) The defendant keeps a loaded gun in his house for protection. One day, the defendant leaves the gun on the kitchen table and leaves for work. That same day, a robber breaks into the defendant’s home, sees the gun, and steals it. The robber then uses the gun to rob a convenience store. During the robbery, the robber shoots and kills the proprietor with the defendant’s gun.
A
  1. (B) For criminal liability to be based upon a failure to act, it must first be found that there is a legal dutyto act. The situations which that impose such a duty include:(1) duty based upon the relationship of the parties; (2) duty based upon statute; (3) duty based upon contract; (4) duty based upon voluntary assumption of care; (5) duty based upon creation of peril; (6) duty to control conduct of others; and (7) duty of landowner. Regarding duty based upon relationship, the common law imposed an affirmative duty upon a husband to aid his imperiled wife. InterestingLy enough, the common law is silent on a wife’s legal duty. LaFave states, however, that “it would seem that a wife owes a similar duty to act to safeguard her husband.” Criminal Law, footnote 11, pg. 204. Thus, choice (B) is correct. Choices (A) and (C) are incorrect. At most, the defendant would be guiLty of involuntary manslaughter, as the conduct in both situations would not rise to the level of depraved-heart murder. Choice (D) is incorrect. The defendant would not be guilty of any crime committed by the robber, as his conduct of leaving the gun on his kitchen table was not criminal.
How well did you know this?
1
Not at all
2
3
4
5
Perfectly
52
Q
  1. A passenger on a subway car placed her pocketbook on the seat next to her and began to zip up her jacket. Defendant was standing in front of the passenger and holding on to the hand rail. Seeing the pocketbook unattended, he suddenly grabbed it and tried to run away. As he did so, the passenger became very frightened and fell backward. She struck her head against the window and was knocked unconscious. In the commotion, Defendant dropped the pocketbook and hurriedly exited the subway car empty-handed.Defendant should be found guilty ofa. larceny.b. attempted larceny. c. assault.d. robbery.
A
  1. (A) The distinction between larceny and robbery is tested on each and every Multistate exam. Robbery requires that the taking be done by means of violence or intimidation. Larceny from the person or presence of the victim is not robbery without the added element of force or violence. The line between robbery and larceny from the person (between violence and lack of violence) is not always easy to draw. The “snatching” cases, for instance, have given rise to some dispute. LaFave points out that “the great weight of authority supports the view that there is notsufficient force to constitute robbery when the thief snatches property from the owner’s grasp so suddenly that the owner cannot offer any resistance to the taking.” Criminal Law, pg. 696. Choices (C) and (D) are incorrect. This is larceny, despite the fact that the victim became frightened after the taking. Choice (B) is wrong because sufficient asportation occurred when the Defendant took the pocketbook off the seat (even if it was only a slight distance).
How well did you know this?
1
Not at all
2
3
4
5
Perfectly
53
Q
  1. A state has the following homicide statute in effect:“Whoever, purposely and with premeditated malice, or in the perpetration of, or attempt to perpetrate a rape, arson, robbery, burglary, or any other felony dangerous to human life, kills any human being, is guilty of murder in the first degree, and, on conviction, shall suffer death or be imprisoned in the state prison for life; Whoever maliciously but without premeditation kills any human being, is guilty of murder in the second degree, and, on conviction, shall be imprisoned in the state prison for life; Whoever unlawfully kills any human being without malice, express or implied, either voluntarily upon a sudden heat or involuntarily, but in the commission of some unlawful act, is guilty of manslaughter, and on conviction, shall be imprisoned in the state prison not less than two years nor more than 20 years.”A man is having an affair with a married woman. The husband finds out about his wife’s relationship with the man. The next day, the husband sees the man walking down the street. Enraged by the man’s relationship with his wife, the husband pulls out a gun and shoots the man, killing him.The husband is subsequently arrested and prosecuted under the homicide statute for killing the man. At trial, the husband’s attorney attempts to introduce evidence that at the time the husband shot the victim, he was intoxicated.Upon objection by the prosecuting attorney, the evidence is(A) admissible.(B) admissible, but the jury must be cautioned that it does not pertain to manslaughter.(C) admissible, but the jury must be cautioned that it does not pertain to felony murder or seconddegree murder.(D) not admissible.
A
  1. (B) This is another popular Multistate Criminal Law question dealing with voluntary intoxication as a defense for homicide. Always remember that voluntary intoxication will never mitigate murder to manslaughter. It may reduce first-degree murder to second-degree murder if the intoxication negates the defendant’s premeditation, deliberation, or intent. Furthermore, choice (B) is correct because voluntary intoxication is no defense to manslaughter. Choice (A) is incorrect because it is not as strong as choice (B), which speaks directly to the issue in the question. Choice (C) is incorrect because intoxication may pertain to felony-murder, even though it will not reduce a second-degree murder to manslaughter. In this regard, intoxication may be a defense for the underlying felony in a felony-murder prosecution, such as robbery, where it negates the intent to steal. Choice (D) is incorrect for the reasons stated above.
How well did you know this?
1
Not at all
2
3
4
5
Perfectly
54
Q
  1. Having just stolen a car, a defendant was operating the car within the posted speed limit when the steering suddenly malfunctioned. The car swerved out of control, jumped the curb, and crashed into a home. The defendant was arrested and charged with the separate crimes of larceny and malicious destruction of property.During the trial the judge instructed the jury that the theft of the car was sufficient evidence to satisfS’ the malice requirement for the malicious destruction of property charge. The defendant’s attorney has filed a motion objecting to the judge’s jury instruction.The judge’s instruction was(A) correct, because malice can be inferred from the intent to steal.(B) correct, because malicious destruction of property is a general intent crime.(C) incorrect, because larceny is not a lesser included offense of malicious destruction of property.(D) incorrect, because malice requires a showing of recklessness.
A
  1. (D) Larceny and malicious destruction of propertyare separate criminal offenses. Choice (A) is incorrect. The mere theft of the auto does not satisfy the malice requirement for the destruction of property offense. Choices (B) and (C) are incorrect, as they do not answer the call of the question as to whether the judge’s instruction to the jury was proper. Choice (D) is correct because reckless conduct satisfies the malice requirement.
How well did you know this?
1
Not at all
2
3
4
5
Perfectly
55
Q
  1. A defendant went to a convenience store intending to rob it. The defendant had a gun inside his coat pocket. When the defendant entered the store, the owner saw that he had his hand in his coat pocket. Although the owner did not actually see the gun, he noticed a bulge in the defendant’s coat pocket.Paranoid because of a rash of recent robberies, the owner said, “Please don’t hurt me 11 do anything you want.” The owner then fainted and fell to the floor. The defendant walked behind the counter and opened the cash register. He took the money from the register and left the store.If the defendant is charged with robbery, he should be found(A) guilty, because the owner was placed in fear.(B) guilty, because the defendant entered the store with a gun in his possession.(C) not guilty, because the defendant did not make any threat of force.(D) not guilty, because the defendant did not take any money from the victim’s person.
A
  1. (C) Robbery at common law consists of the same six elements as larceny, namely, (1) a trespassory (2) taking and (3) carrying away of the (4) personal property (5) of another (6) with intent to steal it, plus the two added elements of (7) the taking must be accomplished by force, violence, or intimidation and (8) the taking must be from the victim’s person or presence. With respect to element 7, robbery requires that the taking be done by means of force, violence, or intimidation. The most common sortoffearin robbery, of course, isthefearengendered bythe robber’sthreat of immediate bodily injury or death, as where the robber points a gun, loaded or unloaded, at the owner, with a threat to shoot unless the latter hands over his property. In this question, the defendant did not actually threaten the owner nor did he brandish his gun. Therefore, choices (A) and (B) are incorrect. Choice (D) is incorrect because a defendant does not have to actually take money from a victim in order to be found guilty of robbery.
How well did you know this?
1
Not at all
2
3
4
5
Perfectly
56
Q
  1. A wealthy retiree had amassed a fortune after selling his pharmaceutical company. The retiree and his wife lived in an exquisitely decorated penthouse apartment overlooking a park. One night, the retiree and his wife were returning home from dinner when they were accosted by a man outside their apartment building. The man brandished a gun and told the retiree and his wife to take him to their apartment. Upon entering the apartment, the man blindfolded the retiree and his wife and tied them up with duct tape to chairs in the living room. The man then stole the retiree’s wallet and assorted jewelry from the bedroom. After the man exited the apartment, the retiree and his wife tried desperately to free themselves from their constraints. As the retiree was struggling, he suffered a heart attack and died.The man should be found guilty of(A) burglary.(B) robbery and burglary.(C) robbery and murder.(D) burglary, robbery, and murder.
A
  1. (D) First, the man is guilty of burglary, which at common law was defined as the (1) breaking (2) and entering (3) of the dwelling house (4) of another (5) at nighttime (6) with intent to commit a felony therein. In the present case, students must assume that the man intended to commit a larceny or felony when he led the victims into the apartment at gunpoinL Second, the man is guilty of robbery because there was a violent taking of the victims’ personal property. Although robbery requires presence, “the robber takes property from the victim’s presence if he locks or ties the victim up in one room of a building and then helps himself to valuables located in another room.” LaFave, pg. 780. Third, the man is guilty of felony-murder because the retiree’s death was proximately related to the robbery and burglary. Therefore, choices (A) and (B) are incorrect. Choice (C) is wrong because burglary and robbery are separate criminal offenses.
How well did you know this?
1
Not at all
2
3
4
5
Perfectly
57
Q
  1. A husband and wife owned and operated a grocery store. The grocery store was situated in the first floor of the building in which the husband and wife occupied a second-floor apartment. Late one evening, the defendant was walking past the grocery store when he decided to steal the money he heard had been stashed in a cigar box inside the store.The defendant furtively walked to the rear of the building. He then proceeded to open the gate to the fenced-in back yard. Once inside the back yard, the defendant attempted to pry open the back window of the grocery store. Awakened by the barking of his watchdog, the husband went out onto his second- floor back porch and saw the defendant below. The husband yelled at the defendant. Startled, the defendant turned to run when he noticed a large package lying outside the rear door to the store. The defendant picked up the package, which contained baked bread that had been delivered earlier in the evening, and the defendant then ran off.Which of the following crimes will the defendant most likely be convicted of?(A) Larceny.(B) Burglary.(C) Larceny and attempted burglary.(D) Larceny and burglary.
A
  1. (C) The defendant wouLd be guilty of larceny, which at common law may be defined as the (1) trespassory (2) taking and (3) carrying away of the (4) personal property (5) of another (6) with intent to steal it. In the present example, the defendant committed larceny by taking (caption) and carrying away (asportation) the package containing the baked bread. With respect to the crime of burglary, students should note that, at common law, burglary consisted of (1) breaking and (2) entering of (3) a dweLling house (4) of another (5) in the nighttime (6) with the intent to commit a felony therein. Here, the defendant would be guilty of attempted burgLary (rather than the completed crime) because he did not gain entry into the grocery store’s structure. According to LaFave, “a place of business used onLy during the day will not so qualify, but if it is attached to a residence it will.” Choice (A) is incorrect because the defendant is also guilty of attempted burglary. Choice (B) is incorrect because the defendant is also guilty of larceny. Choice (D) is incorrect because the defendant did not complete the crime of burglary for the reasons stated above.
How well did you know this?
1
Not at all
2
3
4
5
Perfectly
58
Q
  1. The victim owned and operated a small convenience store that was situated on the first floor of the building in which the victim occupied a second-floor apartment. Late one evening, the defendant was walking past the convenience store when he decided to break into the store to steal beer and cigarettes.The defendant threw a brick through the window of the convenience store and quickly went inside. Awakened by the alarm, the victim went down the stairs and into the convenience store. Seeing the victim, the defendant turned and began to run.Seeing the defendant running away with a case of beer and a carton of cigarettes, the victim ran after the defendant but tripped and fell, breaking his neck, which resulted in his immediate death.If the defendant is subsequently charged with the victim’s death, he will most likely be found(A) guilty of felony murder.(B) guilty of involuntary manslaughter.(C) guilty of voluntary manslaughter.(D) not guilty, because it was unforeseeable that a death would occur under the circumstances.
A
  1. (A) In accordance with the felony murder rule, at common law, one whose conduct brought about an unintended death in the commission or attempted commission of a serious or inherently dangerous felony was guilty of murder. Choice (B) is incorrect because the defendant is guilty of felony murder, as stated above. Choice (C) is incorrect because a voluntary manslaughter is an intentional killing brought about by a reasonable provocation, and there are no facts to support it here. Choice (0) is incorrect because the victim’s death was foreseeable.
How well did you know this?
1
Not at all
2
3
4
5
Perfectly
59
Q
  1. A defendant and his friend were down on their luck. They decided they would make some extra cash by robbing the local liquor store. The friend didn’t like guns and decided to use a toy gun, which looked like a real gun, but only shot water. The defendant was aware that the gun was not real.Their plan fixed, they walked into the liquor store late one afternoon when few customers were in the store. The friend produced the toy pistol and pointed it at the customers and the clerk. The defendant ordered everyone to the floor, and the clerk began to rummage behind the counter for the key to the register. While his head was ducked, the clerk pulled a shotgun from behind the counter and fired it at the defendant and his friend. The defendant dived to the side, but his friend was shot and killed.If the defendant is subsequently charged with his friend’s death he should be founda. guilty of felony murder.b. guilty of voluntary manslaughter.c. guilty, because a felon may be convicted of the murder of a co-felon.d. not guilty, because the clerk was justified in killing the friend.
A
  1. (D) The Redline limitation on the felony—murder doctrine holds that a felon is not liable for the death, which the felon did not intend, of a co-felon participating in criminal activity. According to LaFave, Criminal Law, 2Ed, pg. 629, “it is now generally accepted that there is no felony-murder liability when one of the felons is shot and killed by the victim, a police officer, or a bystander.” Although this is contrary to the common law rule, Redline is now the prevailing view in a majority of jurisdictions. Choice (A) is incorrect for the reasons stated above. Choice (B) is incorrect because a voluntary homicide is an intentional killing based on reasonable provocation. Choice (C) is incorrect because it reflects the common law rule.
How well did you know this?
1
Not at all
2
3
4
5
Perfectly
60
Q
  1. Under which one of the following situations would the defendant(s) most likely be found guilty of larceny?(A) A customer took his watch to a jeweler for repairs. The defendant inspected the watch and informed the customer that the watch needed a minor adjustment, which he would perform that afternoon. The customer gave the watch to the defendant and told him that he would return the following day to pick it up. As the defendant was repairing the watch, he discovered that it was an extremely valuable antique. He then substituted a less-expensive look-alike for the watch, which the customer picked up the next day.(B) As two defendants were walking down the street, they noticed a victim park his metallic gold sports car and enter the pool hail. When they approached the car, one of the defendants observed that the victim had left the keys in the ignition. Knowing that the victim would be hustling pool all evening, the defendants hopped into the sports car and drove off, intending to return the car later that evening.(C) During a craps game in the back room of a bar, the defendant lost $150 to the victim. The defendant left the bar after losing his money and returned an hour later with a gun in his possession. Honestly believing that the $150 still belonged to him, the defendant confronted the victim in the back room and demanded the return of his money. Frightened, the victim handed the money back to the defendant.(D) One afternoon, the defendant noticed the victim riding his racing bike in the park. The defendant, who always wanted to own a racing bike, saw his opportunity when the victim left his bike unattended to participate in a touch football game. The defendant jumped on the bike and quickly rode away. Later that evening, the defendant called the victim and demanded $200 for the return of the bike. The victim agreed to the defendant’s demand. The following day, the victim paid the defendant the money, and the defendant, in turn, returned the bike.
A

6o. (D) Choice (D) is correct. Choice (A) is incorrect because the defendant did not commit larceny when he fraudulently converted the watch which that he was lawfully in possession of. Therefore, the defendant would be guilty of embezzlement, not larceny. Choice (B) is wrong because one who takes another’s property, intending at the time he takes it to use it temporarily and then return it unconditionally within a reasonable time (and having the substantial ability to do so), lacks the intent to steal required for larceny. Choice (C) is wrong because one may take the property of another honestly, but mistakenly, believing that it is his own property. In such an event, the defendant lacks the intent to steal required for larceny, even though his mistaken but honest belief was unreasonable. A similar result was reached in People v. Rosen, 78 R2d 727 (1938), where the defendant used a pistol to recapture money he had lost by illegal gambling, honestly believing the money still belonged to him; conviction of larceny held reversed for lack of intent to steal. Consequently, choice (D) is correct, since it is no defense to larceny that the taker intended to return it (i.e., the stolen property), only if he should receive a reward for its return.

How well did you know this?
1
Not at all
2
3
4
5
Perfectly
61
Q
  1. A state has the following statute in effect:“No person shall sell, barter, furnish, or give to a minor under 16 years of age an air gun, rifle, shotgun, pistol, or other firearm; or being the owner or having charge or control thereof, knowingly permit it to be used by a minor under such age. Whoever violates this statute shall be fined not more than $1,500 (one thousand five hundred dollars), or imprisoned not more than 45 days, or both.”A mother purchased an air rifle, which she gave to her son. The mother, a police officer, who was familiar with firearms, trained her son in the systematic practice of care in the use of the air rifle. One afternoon, the son, who was 15 years of age, was playing with his friends. The three boys took turns firing the air rifle, which discharged small pellets, at various targets in the son’s back yard. As one of the friends, who was 16 years old, was using the air rifle, he fired a shot over the other friend’s head, intending to frighten him. The pellet missed the other friend, but struck a next-door neighbor in the eye, severely injuring her.The mother is subsequently charged with violating the state statute. As her defense, the mother claims that she erroneously believed the statute prohibited firearms to be given or sold to minors under 15 years of age.If the mother’s mistaken belief is honest, it should(A) result in her acquittal, because she didn’t have the necessary mental state required for the crime.(B) result in her acquittal, because the friend is 16 years of age or older.(C) not prevent her conviction, because mistake of law is no defense.(D) not prevent her conviction, because mistake of fact is no defense.
A

6i. (C) As a general rule, ignorance or mistake as to a matter of fact or law is a defense if it negates a mental state required to establish a material element of the crime. H ow- ever, LaFave states that a “quite different kind of mistake of law, whereby the defendant believes that his conduct is not proscribed by the criminal law, is generally not a defense.” Criminal Law, pg. 356. Choices (A) and (B) are incorrect because the statute may be violated in either of two ways. First, anyone who gives, sells, or furnishes an air gun (or firearm) to a minor is guilty of violating the statute. Second, anyone who is the owner or in control of a firearm and who knowingly permits it to be used by a minor is also in violation of the statute (which imposes strict criminal liability). In the present case, the mother violated the first section of the statute by giving the air rifle to her son, who was underthe statutory age. Choice (D) is incorrect because a mistake of fact, which is not present in our question, can be a defense.

How well did you know this?
1
Not at all
2
3
4
5
Perfectly
62
Q
  1. One morning in a Laundromat, a defendant approached a man and said, “If you don’t pay me $500 by July 2,, I’ll beat you to a pulp.” A week later, on July 2, the defendant met the man at a local bar and demanded the money. The man handed the defendant the $500. After receiving the money, the defendant then punched the man in the stomach and hurriedly left the bar.Under modem statutory law, the defendant will most likely be found guilty of which of the following crimes?(A) Extortion and battery.(B) Extortion and robbery.(C) Assault and battery.(D) Assault and robbery.
A
  1. (A) Statutory extortion (or blackmail) covers threats to do future bodilyharm. The crime of robbery by intimidation requires that the threat be to do immediate harm. Since the defendant threatened the man with future bodily harm, the former would be found guilty of extortion, not robbery. In addition, the defendant should be found guilty of criminal battery, which may be defined as the unlawful application of force. Assault, on the other hand, does not require such physical contact. Therefore choices (B), (C), and (D) are incorrect.
How well did you know this?
1
Not at all
2
3
4
5
Perfectly
63
Q
  1. A woman entered a jewelry store and asked the store’s owner if he had any bracelets with turquoise and mother-of-pearl inlay. The owner answered affirmatively and showed the woman two display trays of bracelets. As the woman was looking at a few of the pieces, the telephone in the store began to ring. The owner excused himself and walked to the rear of the store where he answered the telephone call. While the owner was speaking on the phone, the woman placed one of the bracelets in her pocketbook and walked a few feet toward the front door of the store. She was about to leave the store, without paying for the bracelet, when she suddenly noticed one of the other employees. Thinking that the employee had seen her place the bracelet in her pocketbook, the woman walked back to the counter and returned the bracelet to the display tray. In fact, the employee had seen the woman take the bracelet but decided not to say anything after she put it back.If the woman is subsequently prosecuted for larceny of the bracelet, she will most likely be found(A) guilty, because it is not relevant that she returned the bracelet to the display tray.(B) guilty, because the employee had actually seen her place the bracelet in her pocketbook.(C) not guilty, because she returned the bracelet to the display tray.(D) not guilty, because she didn’t leave the store with the bracelet in her possession.
A
  1. (A) Commission of the crime of Larceny requires a taking (caption) and carrying away (asportation) of another’s property. A taking occurs when the offender secures dominion over the property, while carrying away requires some slight movement of the property. Therefore choices (C) and (D) are incorrect. Choice (B) is incorrect. The fact that the employee did or did not see her place the bracelet in her pocketbook is not relevant as to whether the woman would be guilty of larceny.
How well did you know this?
1
Not at all
2
3
4
5
Perfectly
64
Q
  1. A man was at a hotel bar having a drink when the defendant approached him and sat down next to the man. The man was attracted to the defendant and immediately engaged her in conversation. After having a few drinks together, the man invited the defendant back to his room for a nightcap. Sensing that the man was wealthy, the defendant agreed to accompany him back to his hotel room.When they got back to the room, the man excused himself to go to the bathroom. While he was inside the bathroom, the defendant took some narcotics from her purse and placed them in the man’s drink. The defendant realized that the narcotics were a strong sedative and hoped that they would cause the man to pass out. She then planned to steal his wallet and jewelry. Upon returning from the bathroom, the man proceeded to finish his drink. Shortly thereafter, the man passed out. While he was unconscious, the defendant took his watch off his wrist and pulled the wallet out of his pants. She took $300 in cash from the wallet. As she was leaving, the defendant noticed that the man had a diamond earring in his left ear. She then ripped out the earring from his ear and stole that, as well. The man suffered a cut in his earlobe when the defendant snatched the earring.Which of the following crimes should the defendant be convicted of?(A) Larceny.(B) Robbery.(C) Larceny and robbery.(D) Battery and larceny.
A
  1. (B) This question covers two important elements regarding robbery. At common law, robbery consists of all six elements of larceny: (1) trespassory (2) taking and (3) carrying away (4) the personal property (5) of another (6) with intent to steal, plus two additional requirements: (7) the taking be accomplished by force, violence, or intimidation, and (8) the taking must be from the victim’s person or presence. First, one may commit robbery by rendering his victim helpless by administering intoxicating liquors or drugs to produce unconsciousness as a means of force. LaFave, Criminal Law, pg. 782. Second, choices (A), (C), and (D) are wrong because larceny is a “Lesser included” crime and merges into robbery. A defendant cannot be guiLty of both larceny and robbery for the same criminal transaction.
How well did you know this?
1
Not at all
2
3
4
5
Perfectly
65
Q
  1. A defendant shot and killed a victim at close range with a revolver. After the shooting death, the defendant was arrested and charged with first-degree murder. In this jurisdiction, first-degree murder is defined as “knowingly or willfully causing the death of another human being.”Which of the following situations, if believed by the jury, would most likely result in the defendant’s acquittal of the first-degree murder charge?(A) The victim, who was the defendant’s brother, suffered from an incurable case of cancer and asked the defendant to kill him to put an end to his pain and suffering.(B) The killing was not done with premeditation or deliberation.(C) The defendant intended to kill himself, but the bullet grazed his head and struck and killed the victim.(D) The defendant intended to kill another man, but the victim unknowingly stepped in front of the other man and was struck and killed by the bullet.
A
  1. (C) According to LaFave, “A is guilty of murder if he is actually the agent of B’s death, notwithstanding the fact that he acted at B’s request — as where A shoots and kills B upon B’s insistence that he wants to die now rather than continue to suffer from a serious illness.” Criminal Law, p. 650. Choice (A) is incorrect, since mercy killing would not result in the defendant’s acquittal of the first-degree murder charge. Choice (B) is also incorrect because the lack of premeditation and deliberation (subjective mental states determined from the defendant’s conduct in light of the surrounding circumstances) does not preclude the fact that the defendant’s conduct in firing the gun at the victim could still have been knowing and/or willful. Choice (D) is incorrect because the defendant’s intent to kill the other man, the intended victim, will be transferred to the victim, the actual victim, under the doctrine of transferred intent. By process of elimination, choice (C) is correct. Suicide is not murder under the statute, since the defendant would be required to have knowingly or willfully caused the death of another human being, not of himself. By attempting suicide, the defendant did not knowingly or willfully cause the death of the victim.
How well did you know this?
1
Not at all
2
3
4
5
Perfectly
66
Q
  1. A state has a statute prohibiting the sale of cigarettes to any person under the age of 17. Violation of the statute is a misdemeanor punishable by a fine not less than $500 or more than $5,000 The state courts have interpreted this statute as a public welfare offense requiring no particular mental state for its violation. As such, this is a strict liability crime whereby culpability is imposed on a defendant for doing the proscribed act.The defendant, a recovering alcoholic, worked as a cashier at a drug store. Late one night while the defendant was working alone, he got the urge to have a drink. He opened a bottle of rum that the store sold and soon began drinking. A short time later, the defendant became inebriated and passed out. Not long thereafter, a 15-year-old girl entered the store to purchase a package of cigarettes. She took a box off the shelf and went to the cashier’s counter to pay for them. Seeing the defendant lying on the floor, unconscious, the girl left the exact price for the cigarettes on the countertop and left the store.If the defendant is prosecuted for violating the state statute, he should be found(A) guilty, because the offense does not require any mental state.(B) guilty, because the defendant’s intoxication was voluntaiy.(C) not guilty, because the defendant was unconscious.(D) not guilty, because the defendant’s employer is vicariously liable for the violation that occurred.
A
  1. (C) This Multistate question deals with statutory interpretation. Here, it is necessary to carefully read the facts to see that the statute makes it a crime to sell cigarettes to anyone under the age of 17. In the present example, the defendant did not violate the statute because he didn’t sell the cigarettes to the girl, so we know the answer must be choice (C) or choice (D). Note that choice (D) is wrong because if the defendant is not criminally liable, then no vicarious liability will be imposed on the owner of the drug store. Choice (A) is incorrect because although the statute is a strict liability crime, the defendant would still have to fulfill the actus reus requirement and because he was unconscious, he could not do so. Choice (B) is incorrect for the reasons stated above.
How well did you know this?
1
Not at all
2
3
4
5
Perfectly
67
Q
  1. A state has a statute prohibiting the sale of guns and ammunition to any person under the age of 17. Violation of the statute is a felony punishable by a fine of $5,000 and imprisonment for up to two years. The state courts have interpreted this statute as a public welfare offense requiring no particular mental state for its violation. As such, this is a strict liability crime whereby culpability is imposed on a defendant for doing the proscribed act.A defendant, a 15-year-old, entered the store to purchase a shotgun as a present for her father. She took a shotgun off the shelf and went to the cashier’s counter to pay for it. Not seeing the cashier anywhere, the defendant left the exact price for the shotgun on the countertop and left the store.If the defendant is prosecuted for violating the state statute, she should be found(A) guilty, because she purchased the shotgun while under the statutory age.(B) guilty, because the statute imposes absolute criminal liability.(C) not guilty, provided she was unaware that the statute prohibited the sale of weapons to a person in her age group.(D) not guilty, provided that the legislative intent was not to make the purchase of shotguns a criminal offense.
A
  1. (D) The statute is interpreted to make the sale, not the purchase, of firearms (to minors) a criminal offense. As such, only sellers of firearms and ammunition may be convicted under the statute. Choice (A) is incorrect because, as stated above, only the sale of firearms is prohibited under the statute. Choice (B) is incorrect for the reasons already stated. Choice (C) is incorrect, because, even if this were true, ignorance of the law is not a defense.
How well did you know this?
1
Not at all
2
3
4
5
Perfectly
68
Q
  1. Late one night, a defendant, who had a long history of drug-related arrests, was attending a party at a house. During the party, the defendant approached an undercover narcotics agent and offered to sell him some drugs. The undercover agent purchased the drugs from the defendant. Immediately thereafter, the undercover agent arrested the defendant and charged him with conspiracy to sell narcotics and sale of narcotics. He was convicted of both crimes and given consecutive seven-year sentences.On appeal, the defendant’s best argument is which of the following?(A) There was no true agreement between him and the undercover agent and, hence, noconspiracy.(B) There was no true agreement between him and the undercover agent and, hence, no sale.(C) He cannot be convicted of both the sale of narcotics and conspiracy because each offense is essentially the same crime.(D) He cannot be convicted of both the sale of narcotics and conspiracy because both crimes arose from the same criminal transaction.
A
  1. (A) The agreement is all-important in conspiracy. In order to have a conspiracy, there must be an agreement between two or more persons to engage in a criminal act. Since the buyer was an undercover poLice officer, he never intended to really purchase the narcotics; he feigned agreement because he wanted to trap the defendant. Therefore, no agreement (or “meeting of the minds”) existed. Choice (A) is preferred over choice (B) because in a conspiracy, you need the requisite plurality of two or more persons entering into an agreement. On the other hand, the defendant can be convicted of sale of narcotics because sale does not require an agreement between two or more persons. Choices (C) and (D) are incorrect, as double jeopardy would not prevent conviction for conspiracy and the sale of narcotics.
How well did you know this?
1
Not at all
2
3
4
5
Perfectly
69
Q
  1. A husband came home early from work one day. He walked to the bedroom and, upon opening the door, saw his wife and another man in bed together. Visibly upset, the husband told the other man to leave. As the man was reaching for his pants, he pulled out a gun, fearing that the husband was about to attack him. When the husband saw the gun, he jumped on the man and angrily twisted his neck, breaking it and killing him.The husband is guilty for which, if any, of the following crimes?(A) Murder.(B) Voluntary manslaughter.(C) Involuntary manslaughter.(D) No crime.
A
  1. (D) In this question, the facts clearly indicate that the other man was the aggressor. When the husband caught the other man in bed with his wife, the husband did not threaten the other man with bodily harm or injury. It was only after the other man brandished his weapon that the husband acted in self-defense and killed him. Therefore choices (A) and (C) are incorrect. This type of question appears frequently on the Multistate because many students will skim the facts and go for choice (B), since they have been trained for that response anytime they see an apparent “heat of passion” killing.
How well did you know this?
1
Not at all
2
3
4
5
Perfectly
70
Q
  1. While on a camping trip, a defendant became intoxicated and decided to take a walk late at night. He was so drunk he could not find his way back to the campsite. He did, however, come across a secluded mountain cabin, in which he decided to take shelter for the night. Since the door to the cabin was locked, he broke a window and entered the structure. Once inside, he fell asleep on the sofa. When he awoke the next morning he was hungry. The defendant then found some food in the refrigerator, which he cooked and ate. Before leaving the cabin, the defendant looked around to see if there was anything worth stealing. He opened the door to the bedroom and found a gold watch on the nightstand. He placed the watch in his pocket and left the cabin. The defendant was later arrested and charged with burglary.The defendant’s strongest defense would be that(A) he was drunk when he broke into the cabin.(B) the crime was not completed until the morning hours.(C) he did not have the requisite intent at the time of the breaking and entering.(D) the cabin was empty when he entered it.
A
  1. (C) The common law definition of burglary requires that there be a (1) breaking and (2) entering of (3) a dwelling house (4) of another (5) in the nighttime (6) with the intent to commit a felony therein. Although the defendant originally broke into the cabin at night, his breaking and entering was not accompanied by the requisite felonious intent. Thus, choice (C) is the best answer. Since burglary is a specific intent crime, intoxication is a defense to the crime if it negates a required element of the crime, and this is so, whether the intoxication is voluntary or involuntary. This makes choice (A) an attractive answer, but choice (A) is incorrect because there are no facts that say that the defendant’s intoxication prevented him from forming the requisite intent, but merely that he could not find his way back to the campsite. Choice (B) is not the best choice. The breaking and entering took place in the nighttime. If the defendant had the intent to commit a felony at the time, then the crime of burglary would be complete at the breaking and entering. Choice (D) is wrong because whether the dwelling house is vacant or occupied is irrelevant to the crime of burglary.
How well did you know this?
1
Not at all
2
3
4
5
Perfectly
71
Q
  1. A defendant and his friend were drinking heavily one night at the friend’s house. They both became intoxicated. After several hours, both the friend and the defendant passed out. In the middle of the night, the defendant awoke with a headache. He went down to the kitchen and poured himself another shot of whiskey. The defendant also felt hungry and decided to make himself some eggs. After fixing his snack, he inadvertently forgot to turn off the stove. A gas leak occurred, causing a fire that burned down the house.If the defendant is charged with arson, he will be found(A) guilty, because the defendant was reckless.(B) guilty, because the defendant was criminally negligent.(C) not guilty, because the defendant did not commit a felony.(D) not guilty, because arson requires the presence of malice.
A
  1. (D) The defendant would not be guilty of arson under the common law, in that arson requires the presence of “malice.” A defendant will have the requisite intent of malice when that defendant acts intentionally or with reckless disregard of an obvious or known risk. A person acts recklessly with respect to a material element of an offense when he consciously disregards a substantial and unjustifiable riskthat the material element exists or will result from his conduct. The risk must be of such a nature and degree that, considering the nature and purpose of the actor’s conduct and the circumstances known to him, its disregard involves a gross deviation from the standard of conduct that a law-abiding person would observe in the actor’s situation. The facts indicate that several hours passed after the drinking and that the defendant “inadvertently” forgot to turn off the stove. Even if the Defendant had not been drinking at all that night, his behavior does not appear to involve a “conscious disregarding” of the risk, nor is ita “gross deviation from the standard of conduct.” In other words, people do leave the stove on accidentally in everyday life. This is not the kind of legal reckless disregard of an obvious or known risk that malice is intended to capture. Therefore, answer choice (D) is correct.
72
Q
  1. An elderly woman entered a convenience store and tried to cash her Social Security check. The owner told the woman that he was sorry but it was his policy not to cash Social Security checks. Terribly upset, the woman reached into her pocketbook and said, “Listen, buddy, I’ve got a bomb in here, and if you don’t give me the money, I’m going to blow up this joint.” Actually, the woman did not have a bomb in her possession. The owner, who was not deceived by her threat, felt sorry for the woman and agreed to cash the check. The woman, who walked with the assistance of a cane, took the money and limped out of the store. After she left, the owner noticed that the woman had not endorsed the check.The woman has committed which, if any, of the following crimes?(A) No crime.(B) Robbery.(C) Attempted robbery.(D) False pretenses.
A
  1. (C) As noted earlier, the crime of attempt consists of: (1) an intentto do an act or bring about certain consequences which that would, in law, amount to a crime; and (2) an act in furtherance of that intentwhich, as it is most commonly put, goes beyond mere preparation. Robbery, on the other hand, consists of all six elements of larceny—a (1) trespassory (2) taking and (3) carrying away of the (4) personal property (5) of another (6) with intent to steal it—plus two additional requirements: (7) that the property be taken from the person or presence of the other and (8) that the taking be accomplished by means of force, violence, or intimidation. In the present case, the woman will be guilty of attempted robbery because she acted with the requisite intent to commit a robbery and performed an overt act (or “substantial step”) in furtherance of that goal. Her act of reaching into her pocketbook, and her statement accompanying this action would constitute an “act toward the commission” of the crime. Choice (C) is a better answer than choice (B) because the owner was not intimidated or deceived by her ruse. Although the woman is not guilty of a completed robbery, she, nevertheless, may be convicted of attempt. Therefore, choice (A) is incorrect. Choice (D) is incorrect because a false representation of a material fact did not cause the owner to pass title to the money to the woman.
73
Q
  1. A homeowner was holding his annual party at his home. His parties attracted a wide variety of diverse individuals. On the night in question, a defendant was loafing at the local bar when a number of the invited guests came in to purchase spirits for the party. The defendant decided to crash the party. He arrived at the homeowner’s house around midnight when the party was in full swing. He went in through the front door, which was unlocked, and easily mingled with the other partygoers.During the party, the defendant wandered through the house and came upon a baseball, signed by a famous baseball player, in a glass case. Knowing the ball to be very valuable, the defendant put it under his jacket and made his way down the stairs and out the front door.If the defendant is subsequently charged with burglary, he will probably be found(A) guilty, because he crashed the party.(B) guilty, because he intended to steal the baseball.(C) not guilty, because he crashed the party only to have a good time.(D) not guilty, because the front door was unlocked.
A
  1. (C) At common law, burglary consisted of breaking and entering the dwelling house of another at nighttime, with intent of committing a felony. The defendant did not intend to commit a felony at the time he entered the house. He did not formulate his criminal intent (e.g., to steal the baseball) until after he had entered the house. Choice (B) is incorrect because the intent was formed after the entrance to the house. Choices (A) and (D) are incorrect because both choices only address how the defendant entered the house and do not address whether or not there was any intent to commit a felony inside the home. Choice (C) is the best choice because it addresses the issue of his mental state when he entered. Since this choice negates an element of the crime, it is a better choice because it forecloses the possibility of liability for burglary.
74
Q
  1. A defendant went to a party at his friend’s house. While he was there, the defendant drank a lot of beers and countless shots. The defendant eventually stumbled into the bathroom where he spotted a purse lying on the floor under the sink. Wondering vaguely if it might contain more beer, he opened it up and found his friend’s wallet. He grabbed some money from the wallet and left the bathroom.As he was descending the stairs with the money in his hand, his friend spotted him. She was initially concerned he might tumble down the stairs but upon seeing the money in his hand, she asked him where he got it. The defendant responded, “Ha! Your money or your life!” Not taking him seriously, the friend let him pass by. It wasn’t until later that she realized that the defendant had taken the money from her wallet.The defendant should be charged with which of the following crimes?(A) Larceny.(B) Larceny and burglary.(C) Larceny and attempted extortion.(D) Robbery and extortion.
A
  1. (A) First, the defendant is guilty of larceny, not robbery. Robbery requires that the taking be done by means of force, violence, or intimidation. Larceny from the person or presence of the victim is not robbery without this added element of force or fear. The most common sort of fear in robbery is the fear, engendered by the robber’s intentional threat, of immediate bodily injury or death to the property owner himself (as where the robber points a gun, loaded or unloaded, at the owner with a threat to shoot unless the latter hands over his property). In this exampLe, the defendant never actually threatened the friend with bodily injury. The fact pattern tells you that the friend was never afraid, and the defendant’s statement itself makes it clear that it was not intended as a real threat. As a result, choice (D) is wrong because no robbery took place. Choice (C) is also incorrect because practically all extortion statutes cover the demand for (or acquisition oO money. Here, however, the defendant never extorted or demanded money (though “he had the idea” of doing so). Choice (B) is incorrect because the elements of burglary are not present here.
75
Q
  1. The defendant was an off-duty police officer who frequently worked security at concerts on his free time. One night, he was working backstage security for one of his favorite bands when he noticed a groupie sneak in the back door, pick up a guitar, and begin to walk out with it. He drew his service revolver and exclaimed, “Stop or I’ll shoot!” The groupie looked at him and then fled. The defendant fired at the fleeing groupie, fatally wounding her. Use of force was not authorized by the concert promotion company that employed him.On these facts, if the defendant is charged with murder, he should be found(A) guilty.(B) not guilty, because the killing constituted justifiable homicide.(C) not guilty, because the killing constituted excusable homicide.(D) not guilty, because the defendant warned the groupie before he fired his gun.
A
  1. (A) The defendant would be guiLty of killing the groupie, since a person may not use deadly force in the defense of personal property. A person may use deadly force in the defense of property generally only in conjunction with another privileged use of force, (i.e., self-defense or defense of another individual). Furthermore, the modern view is that deadly force may be used only to prevent the commission of dangerous feLonies (e.g., those involving a substantial risk of death or serious bodily harm). As a consequence, choices (B), (C), and (D) are incorrect.
76
Q
  1. A woman had been spreading rumors around town that a defendant was a shoplifter. After the defendant learned about these rumors, she was furious and decided that she would teach the woman to keep her mouth shut. Late that night, the defendant went over to the woman’s house with the intent to punch her in the mouth. When the defendant arrived at the woman’s home, she peered in the bedroom window and saw the woman asleep in bed. The defendant proceeded to pry open the bedroom window and climbed inside. The defendant then grabbed the woman and punched her in the face and hurriedly fled from the home. The force of the blow fractured the woman’s jaw.If the defendant is subsequently prosecuted for burglary, she will most likely be found(A) guilty, because she broke into the woman’s home at night with the intent to commit a felony therein.(B) guilty, because the woman suffered a broken jaw as a result of the blow inflicted by the defendant.(C) not guilty, because the defendant did not intend to seriously injure the woman.(D) not guilty, because the defendant did not intend to commit a felony at the time of the breaking and entry.
A
  1. (D) At common law, burglary required the breaking and entering of the dwelling house of another at nighttime with intent to commit a felony therein. Note: Assault and battery were two separate common law misdemeanors. Consequently, the defendant could not be found guilty of burglary (at common Law) because she intended to commit a misdemeanor only. Therefore, choices (A) and (B) are incorrect. Choice (C) is not the best answer because it is a correct statement of fact. On the MBE, a correct statement of law will generally be a stronger answer than a correct statement of fact.
77
Q
  1. In which of the following situations would the actions of the defendant(s) constitute a burglary?(A) A defendant, seeking shelter from a severe snowstorm, broke into a house. When he awoke the next morning, the defendant noticed a transistor radio, which he put into his pocket before fleeing the house.(B) A defendant was walking past a home when he saw the homeowner sitting in a chair near the window. Angered at the homeowner for stealing his former girlfriend, the defendant picked up a large brick that was lying nearby and hurled it through the window, intending to kill the homeowner.(C) Late one afternoon, two defendants were walking down the street when they noticed an open window at a doctor’s office. The defendants crawled through the window, broke open his medicine cabinet, and stole various drugs.(D) Late one evening as a guest was registering ata hotel, a defendant noticed that the guest wasa guy who owed him money from a bet. Afterthe guest had retired to his room, the defendantbroke into his room and threatened him regarding the money he was owed.
A
  1. (B) At common law, burglary consisted of (1) breaking and (2) entering (3) a dwelling house (4) of another (5) in the nighttime (6) with the intent to commit a felony therein. Choice (B) is correct because hurling the rock through the window of the home would constitute an effective breaking and entering. Moreover, since the defendant intended to kill the homeowner, requirement (6) would also be met. Choice (A) is incorrect because the defendant did not have the intent to commit a felony at the time he broke into the house. Choice (C) is incorrect because at common law, the burglary had to be of a dwelling, not a place of business. Finally, choice (D) is incorrect because the defendant only intended to commit an assault upon breaking and entering the hotel room, which was a misdemeanor at common law.
78
Q
  1. A man was on a date when he realized that he didn’t have enough money to pay the bill. The man excused himself and went to the men’s room. The man decided to set fire to the waste paper in a trash container. He hoped that the fire would serve as a diversion so he and his date could run out of the restaurant without paying. He set fire to the waste paper and then went back to his table. As he anxiously waited, the maitre d’ grabbed the man and told him that he had seen what occurred in the bathroom. The maitre d’, who had been seated in a bathroom stall, saw the man, through a crack in the door, set the fire. The maitre d’ extinguished the fire, which had charred a portion of the ceiling. This jurisdiction defines arson as the malicious burning of any structure.If charged with arson, the man should be found(A) guilty, because he was reckless as to the restaurant’s being damaged by fire.(B) guilty, because he set the fire for the purpose of committing an unlawful act.(C) not guilty, because he did not intend to burn down the restaurant.(D) not guilty, because the fire only charred a portion of the ceiling.
A
  1. (A) At common law, arson was defined as the malicious burning of the dwelling of another. A defendant will have the requisite intent of malice when that defendant acts intentionally or with reckless disregard of an obvious or known risk. Choice (A) is correct. The man was reckless in his act of damaging the restaurant. Starting a fire in a wastebasket and then leaving it unsupervised is intentional on the part of the defendant (even if his own desire was to only cause a distraction), and, in any event, would also constitute a reckless disregard of an obvious or known risk. The conduct would be sufficient for the crime of arson, inasmuch as a piece of the ceiling was charred. The requisite degree of “burning” for arson is charring, (i.e., more than blackening by smoke). Choice (C) is incorrect because reckless burning without additional intent is sufficient for arson. Choice (D) is incorrect because charring is enough for arson. Finally, choice (B) is incorrect, because there is no requirement that the defendant have a purpose of committing an unlawful act, as long as the defendant acts with maLice.
79
Q
  1. A defendant was charged with felony murder as a result of his setting fire to the victim’s house. The victim was upstairs sleeping when the fire was set, and he died in his bedroom during the fire.If the defendant can prove the facts to support his defense, which of the following assertions would LEAST likely remove liability for felony murder?(A) The defendant did not intend to kill the victim.(B) The defendant was insane when he set the fire.(C) The defendant was coerced by another to set the fire.(D) The victim died of a heart attack before the fire spread to his bedroom.
A
  1. (A) Choice (A) would least likely relieve the defendant of liability for felony murder. The common law felony murder rule provides that one whose conduct brought about an unintended death in the commission of a felony was guilty of murder. For a defendant to be found guiLty of felony murder, he need not have the intent to kill the victim. Thus, choice (A) would not be a good defense. Choices (B) and (C) state the respective defenses of insanity and duress to the underlying felony (here, arson). Thus, if the defendant is not found guilty of the underlying felony, he cannot be convicted of felony murder. Choice (D) would be helpful to the defendant because that fact would create the absence of the causation factor, which is generally a requirement in felony murder cases.
80
Q
  1. A husband and a wife were chronic alcoholics. One afternoon, the husband drank a fifth of bourbon and was drunk when his wife returned home from work. When the wife saw her husband’s condition, she got very angry because they had planned to go out to dinner and celebrate their wedding anniversary. While the husband was passed out on the living room couch, the wife decided to fix herself a martini. After drinking two martinis, the wife became extremely inebriated. A short while later, the wife began preparing a third martini and tried to slice a lemon for the drink. As she did so, the knife slipped, and she cut her hand severely. With blood gushing from the wound, the wife called her husband to help her. He awoke momentarily, stood up, but fell back on the couch and passed out. He failed to render any assistance, and the wife bled to death.If the husband is subsequently charged with manslaughter, he will be found(A) guilty, because he owed his wife a duty to assist her.(B) guilty, because criminal negligent conduct cannot be negated by voluntary intoxication.(C) not guilty, because the wife caused her own injury.(D) not guilty, because he was physically unable to assist her.
A

8o. (D) Generally, one has no legal duty to aid another person in peril. For criminal liability to be based upon a failure to act, it must first be found that there is a legal duty to act. Such an affirmative act is placed upon persons standing in certain personal relationships to the persons—upon parents to aid their small children, upon husbands to aid their wives, upon ships’ captains to aid their crews, upon masters to aid their servants. Choice (A) states this general rule; however, it is not the best answer. LaFave points out that “[j]ust as one cannot be criminally liable on account of a bodily movement which is involuntary, so one cannot be criminally liable for failing to do an act which he is physically incapable of performing. “Criminal Law p. 209. In our question, the husband is physically unable to assist his wife, so he will not be guilty of manslaughter for failing to aid her from bleeding to death. Choice (D) is correct. Choice (B) is incorrect. This would be invoLuntary manslaughter, which is a general intent crime and, as such, voluntary intoxication is not a defense. Choice (C) is incorrect. The defendant need not have caused the victim’s peril if there is a Legal duty to act based on the relationship of the parties.

81
Q
  1. “An assault is an unlawful attempt, with or without present ability, to commit a battery on the person of another. It is a misdemeanor punishable by imprisonment of up to six months.”A defendant was a lawyer who worked in a law firm located in the state. The defendant, who had a reputation of being a prankster, decided to play a practical joke on the senior partner of the law firm. The defendant donned a cowboy mask and entered the senior partner’s office. As the senior partner was talking on the phone with his back to the door, the defendant pulled out a toy cap gun and said, “This is a stickup . . . give me your wallet or I’ll shoot.” When the senior partner turned around to face the defendant, he pulled the trigger of the gun. This caused the toy pistol to make a loud noise.Although the senior partner was not frightened, he fired the defendant because of the incident, citing unprofessional conduct. The defendant was subsequently charged with criminal assault under the above mentioned statute. At trial, the defendant testified that he was only playing a practical joke on the senior partner and was trying to scare him as a harmless prank.If the jury believes the defendant, they should find him(A) guilty, because he intended to frighten the senior partner.(B) guilty, because the statute does not require the present ability to commit a battery.(C) not guilty, because the defendant did not intend to cause physical injury to the senior partner.(D) not guilty, because the facts state that the senior partner was not frightened.
A
  1. (C) The key words in this question are that the defendant “was playing a practical joke on the senior partner and was trying to scare him.” The mental state described by these words falls short of that necessary under the given statute. The statute requires for assault an unlawful “attempt” — i.e., an intent plus an act to commit an unlawful application of force (i.e., a battery). If the defendant did not intend to commit any application of force on the senior partner, which is what the facts state, then, if the jury believes him, he will be not guilty due to lack of intent to cause physical injury. Therefore, choice (C) is correct. Without a specific intent to unlawfully apply force, the defendant cannot be guilty, so choices (A) and (B) are incorrect. Choice (D) is incorrect, since fright is not required for criminal assault under the given statute.
82
Q
  1. A senior associate in a law firm was conducting spot checks of other associates as they left the office to determine who had taken some documents that were missing from the firm’s library. A young associate was leaving his office for the day when he was stopped by the defendant, who asked him to open his briefcase. The senior associate explained that some files from the law office had been stolen. Startled, the young associate refused to permit the senior associate to look in his briefcase. The young associate said that he had personal papers in his briefcase and he didn’t want the senior associate to see them. Having his suspicions aroused, the senior associate raised his clenched fist and exclaimed, “If you don’t hand over that briefcase, I’m going to punch you in the mouth.” The young associate, who, unknown to the senior associate, suffered from a serious heart condition, had a seizure and died as a result of fright produced by the threatened attack.Which of the following is the most serious crime for which the senior associate should be found guilty?(A) Assault.(B) Voluntary manslaughtçr.(C) Involuntary manslaughter.(D) Murder.
A
  1. (C) As a general rule, whenever an intentional battery or assault results in an unintended death, the defendant is guilty of involuntary manslaughter. LaFave notes that a criminal assault, like a criminal battery, is an unlawful act malum in se. Therefore, if the defendant approaches close to another person intending to strike him but not to kill him, and the latter, who, unknown to the defendant, possesses a weak heart, has a heart seizure and dies as a result of fright produced by the threatened attack, the defendant is guilty of manslaughter, though he never touched the victim. See Regina v. Dugal, 4 Que. L.R. 350 (Q.B. 1878), referred to by LaFave in Criminal Law, pg. 601. Choice (A) is incorrect, even though this would be an assault. However, on the bar exam, it is always wise to read the call of the question closely, and it asks for the most serious crime for which the defendant can be convicted. Choice (B) is incorrect, as voluntary manslaughter is an intentional killing with provocation. Choice (D) is incorrect because there is no intent to kill here.
83
Q
  1. A first-year law student worked as a part-time law clerk at a prestigious law firm. He was permitted to borrow law books and legal treatises to take home for his own personal use. One afternoon, the law student was preparing a brief in the firm’s law library when he noticed a book lying on the conference table. Making sure no one was watching him, he took the book and placed it in his briefcase. He planned to take the book home that night with the intention of not returning it.If the law student is subsequently charged with larceny of the book, which of the following claims, if true, would LEAST aid him in his defense?(A) The law firm had failed to give him his salary for two weeks, and he took the book as security for the debt.(B) He intended to return the book after reading it.(C) The law firm had given him possession of the book.(D) The law firm had given him custody of the book.
A
  1. (D) Students must be aware of the distinction between larceny and embezzlement. Whenever an employer hands his property to his employee, the employee acquires only custody (as the empLoyer retains “constructive possession” of the property), so that the employee who misappropriates the property is guilty of larceny. However, whenever the employee acquires possession of his employer’s property and misappropriates it, he is guilty of embezzlement. In sum, misappropriating employees who have possession of their employer’s property are guilty of embezzlement; those with custody are guilty of larceny. For this reason, if the law student had possession when he misappropriated the book, his crime would be embezzlement, not larceny. Therefore, choice (0) would LEAST aid him in his defense for the crime of larceny. Choice (A) is incorrect because claim of right is a valid defense for larceny, and this would aid the defendant in his defense. Choice (B) is incorrect because, if the defendant did not have the intent to permanently deprive, then he would not be guilty of larceny and, therefore, that would be heLpful to his defense. Choice (C) is incorrect because, if the defendant had possession of the book, the taking could not be trespassory and, therefore, could not be larceny. As this would be very helpful to the defendant’s defense, it is incorrect.
84
Q
  1. A defendant was going out to dinner, and she parked her car at a parking lot across the street from the restaurant. The parking attendant directed the defendant to leave her keys in the car, and he handed her a receipt as she left. Following dinner, the defendant went back to the parking lot and handed her receipt to the attendant who informed her that the parking charge would be $14. Visibly irate, she told the attendant that $14 was much too high for a two-hour parking fee. The parking attendant responded that unless she paid the $14, he could not return her car. She adamantly refused to pay the $14, and her friend, with whom she was having dinner, drove the defendant home in her car.Later that same night, the defendant decided to regain possession of her car with a spare set of keys. She returned to the lot and drove her car home.If the defendant is subsequently charged with larceny, she will most likely be found(A) guilty, because she failed to pay the $14 before regaining possession of her car.(B) guilty, because she was under a binding contractual obligation to pay the parking fee.(C) not guilty, because the $14 charge was excessively high.(D) not guilty, because the defendant cannot be charged with larceny of her own motor vehicle.
A
  1. (A) At common law, larceny may be defined as the (1) trespassory (2) taking and (3) carrying away of the (4) personal property (5) of another (6) with intent to steal it. Note that common law larceny was limited to the taking of tangible personal property. Scott and LaFave in their hornbook, Criminal Law, note that modern statutes in all jurisdictions have broadened the scope of larceny to include such intangible personal property as written instruments embodying choices in action or other intangible rights. However, in the absence of a specific statutory provision, it has been held not to be larceny to make use of the factory, or of the labor and services, of another. In this regard, sometimes the property that X owns is in the lawful possession of Y, who has a pledge or lien interest in the property to secure a debt that X owes Y. From X’s viewpoint, such property is considered the “property of another” for purposes of larceny, so that if X takes it from Y’s possession with intent to deprive him of his pledge or lien interest therein, X is guilty of larceny. Choice (B) is incorrect because contractual obligations would not be relevant in a criminal proceeding. Choice (C) is incorrect because of the reasons stated above. Choice (D) is incorrect for the reasons stated above.
85
Q
  1. An aspiring actress, whose auditions had not been going well, was forced to pawn her mother’s pearls to pay rent and buy food. She finally got good news when her agent told her that she had landed a part in a big movie.Delighted, the actress went down to the pawnshop to retrieve her mother’s pearls. When she arrived, the clerk told her that she needed to have cash in order to retrieve the necklace. As she had not been paid yet for the movie, the actress called her agent and told him her dilemma. The agent told her he would come down to the pawnshop and fix everything. When the agent arrived, the actress was still arguing with the clerk. The agent pulled the actress aside and told her that he would distract the clerk while the actress snatched the necklace. The agent then pretended to be interested in an antique lighter, and the actress snatched her necklace off the counter and left the store.The agent is subsequently prosecuted for larceny. The defendant could most appropriately be found(A) guilty of larceny, but not conspiracy to commit larceny.(B) guilty of conspiracy to commit larceny, but not larceny.(C) guilty of both larceny and conspiracy to commit larceny.(D) not guilty of either larceny or conspiracy to commit larceny.
A
  1. (C) One source of continuing confusion for law students (and lawyers, as well) is whether the doctrines concerning complicity and conspiracy are essentially the same, so that liability as a conspirator and as an accomplice may be based upon essentially the same facts. Is one who is a member of a conspiracy of necessity a party to any crime committed in the course of the conspiracy? On the other hand, is one who qualifies as an accomplice to a crime of necessity part of a conspiracy to commit that crime? According to LaFave, both of these questions must be answered in the negative. In the present case, the defendant should be found guilty of conspiracy (to commit larceny), which is an agreement between two or more persons, which constitutes the act and an intent to do either an unlawful act or a lawful act by unlawful means. Since the agent suggested that the actress recover her necklace (without paying for it) he would be found guilty of conspiracy to commit larceny. On the other hand, should the defendant also be found guilty of the crime of larceny itself? The answer is yes. Generally, one is liable as an accomplice to the crime of another if he (a) gave assistance or encouragement or failed to perform a legal duty to prevent it (b) with the intent thereby to promote or facilitate commission of the crime. Several terms have been employed by courts and state legislatures in describing the kinds of acts which that will suffice for accomplice liability. LaFave notes that the most common are “aid,”“assist,”“cause,”“command,”“counsel,”“encourage,”“hire,”“induce,” and “procure.” Clearly, in the case at bar, the agent aided and assisted the actress in recovering her necklace (without paying for it) to be criminally liable for larceny under the state statute. Therefore, choices (A), (B), and (0) are incorrect.
86
Q
  1. A man was at a bar drinking beer when he started conversing with a woman who was seated at the next barstool. During the course of their conversation, the woman told the man that she was just laid off her job and desperately needed money to pay her rent. The man, a practical joker, pointed to a nearby coat rack and said, “Hey, see that fur coat there. Why don’t you take it?” The man then told the woman that he would cause a disturbance to distract attention while she ran out of the bar with the coat. Believing that it was a good idea, the woman agreed to take part in the scheme.Thereupon, the man lit a matchbook and threw it on top of the bar. He then yelled, “The bar’s on fire, help!” When everyone turned to look at the man, the woman ran to the back of the bar, took the fur coat and scurried outside unnoticed. Just as the woman left the bar and was running down the sidewalk, she was apprehended and arrested by a police officer. Later, the man confessed that the fur coat that the woman took really belonged to him.With respect to the man’s and the woman’s criminal liability, which of the following is most correct?(A) The man and the woman are guilty of conspiracy and larceny.(B) The man and the woman are guilty of larceny.(C) The woman is guilty of larceny.(D) The man and the woman are not guilty of either conspiracy or larceny.
A
  1. (D) Most important, the woman is not guilty of larceny because there was no trespassory taking. Simply stated, trespassory means the taking (or caption) must be without the owner’s permission. The man gave the woman permission to take his fur coat, so choices (A), (B), and (C) are incorrect. Moreover, no conspiracy occurred because the man obviously did not manifest an intention to steal his own coat.
87
Q
  1. A baseball card dealer agreed to sell an extremely rare card to a collector. The collector lived in another state. As part of their agreement, a courier was hired to pick up the card from the dealer and transport it to the collector. The courier was then to get the purchase money from the collector and bring it back to the dealer.But rather than entrusting the genuine card to the courier, the dealer gave him a counterfeit replica that was valueless. After picking up the envelope containing the replica, the courier left the dealer’s home and embarked on his trip. The police, however, received a tip concerning the scheme and intercepted the courier’s car before he left the city limits. Thereafter, the dealer was arrested and charged with attempting to obtain property by false pretenses.If the dealer contends that the scheme had not progressed far enough to constitute an attempt, it will(A) be to his advantage to claim that the courier was his accomplice and was participating in the scheme to receive a share of the proceeds.(B) be to his advantage to claim that the courier knew nothing of the scheme but was simply hired by the dealer to make the delivery.(C) be to his advantage to claim that the courier knew nothing of the scheme but was simply hired by the collector to make the delivery.(D) make no difference whether the courier was the dealer’s accomplice or was an innocent agent.
A
  1. (D) The elements of attempt are: (1) an intent to do an act or bring about certain consequences which that would, in law, amount to a crime, and (2) an act in furtherance of that intent which that goes beyond mere preparation. In the case at hand, the dealer, the defendant, possessed the requisite state of mind, inasmuch as he intended to defraud the victim. Next, by handing the counterfeit replica of the card to the courier to carry out his scheme would constitute a substa ntial step toward the commission of the offense. Choices (A), (B), and (C) are simply diversions that have no legal relevancy.
88
Q
  1. After a wife found out about her husband’s infidelity, she decided to have him killed. The defendant approached a hit man to solicit his assistance in carrying out the planned murder. Although the wife believed that he was a hit man, he was, in fact, an undercover agent.The wife told the undercovr agent she would pay him $20,000 if he accepted the job and an additional $20,000 after the killing was completed. Upon hearing the wife’s proposal, the undercover agent told the wife he would kill her husband. However, the undercover agent secretly intended not to go through with the plan, and he merely feigned agreement because he wished to trap the wife. The wife told the undercover agent that she would deliver the first payment the next day. However, before making the initial payment, the wife learned that the hit man was really an undercover agent. Fearful that she might be prosecuted for planning her husband’s murder, the wife contacted the police and renounced her participation in the criminal endeavor.The defendant will most likely be found(A) guilty of solicitation.(B) guilty of conspiracy to commit murder.(C) guilty of solicitation and conspiracy to commit murder.(D) not guilty of either solicitation or conspiracy to commit murder.
A
  1. (A) For the crime of solicitation to be completed, it is necessary only that the actor, with intent that another person commit a crime, have enticed, advised, incited, ordered, or otherwise encouraged that person to commit a crime. The crime solicited need not be committed. LaFave notes that it is nota defense to a solicitation charge that, unknown to the solicitor, the person solicited could not commit the crime. Criminal Law, pg. 422. Similarly, it is also no defense that the person solicited is an undercover agent and under no circumstances would have committed the crime solicited. It is important to point out that the defendant is not guilty of conspiracy because an essential element of the crime is an agreement (for an unlawful purpose) between two or more persons. Choices (B) and (C) are incorrect because there was no conspiracy as discussed above. Choice (D) is also incorrect because the defendant is guilty of solicitation as stated above.
89
Q
  1. A defendant was at a bar when she saw her exboyfriend who had recently dumped her. The defendant walked away and spotted a good friend of hers. She approached her friend and told him she would give him $50 to beat up her ex-boyfriend. The friend agreed but said he would wait until the ex-boyfriend left the bar. While the defendant was eavesdropping on her ex-boyfriend in the hopes he would say something nice about her, she overheard him having a conversation on his cellphone. During the course of that conversation, the defendant realized that her ex-boyfriend was now a federal agent. Afraid she might be prosecuted for a federal offense, the defendant told her friend not to bother with the planned attack.If the defendant is subsequently charged with the statutory offense of conspiring to commit an assault on a federal officer, the defendant will probably be(A) convicted, because her knowledge that her exboyfriend was a federal agent is not a material element of the crime.(B) convicted, because withdrawal is never recognized as an affirmative defense to conspiracy.(C) acquitted, because she was unaware that her ex-boyfriend was now a federal agent.(D) acquitted, because her abandonment of the plan aborted the conspiratorial objective.
A
  1. (A) According to the holding in United States v. Feola, 420 U.S. 671 (1975), the U.S. Supreme Court held that with respect to the federal statutory offense of conspiracy to commit an assault on a federal officer, defendant’s knowledge that the intended victim was a federal officer was held not to be a material element of the crime. Choice (B) is incorrect because, although it is a true statement and the correct result, choice (A) addresses the specific issue in this question more fully. Choice (C) is incorrect because, as stated above, her knowledge of her ex-boyfriend’s status as a federal agent is not a material element. Choice (D) is incorrect because the defendant would not be able to withdraw from liability for the conspiracy itself, merely the crimes committed in furtherance of the conspiracy which took place after the withdrawal.
90
Q
  1. This jurisdiction has the following bribery statute in effect:“Any person who offers or gives a thing of value to a government officeholder in exchange for official action is guilty of bribery.”A real estate developer owned a large parcel of land in the suburbs. Although the developer wanted to build an office building on the property, the land was zoned residential. Due to the residential zoning, the developer could not pursue his planned development unless he received a variance from the building commission.The developer held a meeting with a member of the building commission to solicit his approval in securing a zoning variance. To do so, the developer gave the commission member $10,000 in exchange for his support in approving the zoning variance. Thereupon, the commission member voted to approve the variance, thus making it possible for the developer to commence construction of the office building.The developer was subsequently prosecuted for conspiracy to commit bribery. During the course of the trial, the commission member testified that he faked the agreement with the developer and would have approved the zoning variance regardless of whether the developer gave him any money. Furthermore, in his defense, the developer presented evidence that the other six members of the building commission voted affirmatively to approve the variance.If the jury believed that the commission member would have approved the variance even had he not received the $10,000, the developer should be found(A) guilty, because the commission member’s agreement to accept the $10,000 was sufficient to form a conspiratorial objective.(B) guilty, because he gave the commission member the $10,000 in exchange for his approval of the zoning variance.(C) not guilty, because the commission member did not receive a thing of value, since he would have approved the variance regardless of receiving any payment from the developer.(D) not guilty, because there was no true agreement between the parties.
A
  1. (D) At common law, conspiracy required two elements: (1) the defendant must possess the specific intent to commit the “target” offense, and (2) there must be an agreement between two or more individuals to commit the criminal offense. The facts state that the commission member faked the agreement and would have approved the variance regardless of the payoff. Thus, if the jury believed the commission member’s testimony, there would not be a true agreement between the two parties. Therefore, choices (A) and (B) are incorrect. Choice (C) is incorrect because the statute doesn’t require that a thing of value actually be received in order for a defendant to be found guilty.
91
Q
  1. A defendant entered a bar and ordered a beer. The bartender politely told the defendant he would have to wait until two other patrons were served. The defendant became irate and pulled out a small penknife. Although the defendant intended only to frighten the bartender, he accidentally nicked the bartender’s arm with the penknife. The bartender’s arm became infected, and he died from gangrene two months later.The most serious crime that the defendant can be convicted of isa. battery.b. involuntary manslaughter. c. assault with a deadly weapon. d. murder.
A
  1. (B) Choice (D) is incorrect. Whenever the crimes of assault or battery result in an unintended death, the defendant is guilty of involuntary manslaughter. In the present case, the defendant committed a (criminal) battery by nicking the bartender with his penknife. Criminal battery is simply the unlawful application of force to the person of another. Because battery was a misdemeanor at common law, the unintentional death resulting from that battery two months later (due to gangrene) is involuntary manslaughter. Choices (A) and (C) are incorrect. While an argument could certainLy be made that the defendant is guilty of battery and assault with a deadly weapon, involuntary manslaughter is the most serious crime for which the defendant could be found guilty and is, therefore, the strongest answer.
92
Q
  1. Under which of the following situations would the defendant not be guilty of the attempted crime at common law?(A) A defendant wanted to receive the proceeds from her insurance policy on her home. After a careful examination of her policy, she decided that the best way for her to collect would be to stage a cooking accident in her own home and have the structure bum down. She decided that bacon would be the best way to do it because all of the extra grease in the pan could easily catch on fire. After cooking the bacon, the defendant left the flame on and left her home. Unbeknownst to the defendant, she left the window open and a breeze blew out the pilot. When the defendant returned home, she found that there was no fire. She was arrested and charged with attempted arson.(B) A defendant was dancing with his girlfriend at a local club when she collapsed in his arms. The defendant then carried his girlfriend to his car and engaged in sexual intercourse with her. Although the defendant believed that his girlfriend was intoxicated, she had, in fact, died of a heart attack while dancing. The defendant is charged with attempted rape.(C) After a preliminary examination, a defendant informed a patient that she was two months pregnant. Although the defendant knew that abortions were illegal in this jurisdiction, he agreed to perform an abortion on the patient, anyway. During the operation, it was discovered that the patient was not pregnant. The defendant is charged with attempt to perform an illegal abortion.(D) A defendant called his friend to obtain some marijuana. The friend told the defendant that he had some for sale. Believing that his friend had some high-quality marijuana, the defendant purchased an ounce of the substance, which he did not know was actually oregano. While the defendant was smoking the oregano on the street corner, a police officer arrested him and charged him with attempted illegal use of a controlled substance.
A
  1. (A) Choice (A) is correct because it is an example of a legal impossibility. At common law, arson is defined as the malicious burning of the dwelling house of another. Here, the defendant attempted to burn down her own home and, therefore, could not commit arson at common law. Conversely, choices (B), (C), and (D) are examples of factual impossibility, which do not constitute a defense to attempt.
93
Q
  1. In which of the following situations would criminal liability LEAST likely be imposed on the defendant?(A) A defendant is an operator of a telephone answering service with positive knowledge that one of his clients was using his service to facilitate the illegal distribution of cocaine.(B) A defendant is a service station attendant who knew that the buyer of gasoline was using his product to make explosives for illegal use.(C) A defendant is a hotel registration clerk who knew that one of his regular guests was using her room for purposes of prostitution.(D) A defendant is the owner of a car that he permits a friend, whom he knows to have been drinking that night, to drive. As a consequence, the friend is involved in an accident that causes a victim’s death.
A
  1. (D) Generally, it may be said that accomplice Liability exists when the accompLice intentionally encourages or assists, in the sense that his purpose is to encourage or assist, another in the commission of a crime as to which the accomplice has the requisite mental state. As a consequence, the defendants in choices (A), (B), and (C) would incur accomplice liability, since their actions or non-action served to promote or facilitate criminal activity. Choice (0) is, therefore, correct, since under the principle of accomplice liability, one does not become an accomplice by an intentional act of assistance or encouragement merely because he knows that such an act might facilitate a crime. This is not to say, however, that an owner (of an automobile), by permitting an intoxicated person to drive his car, will necessarily escape liability. The owner could very well be found guilty of criminal negligence involuntary man-slaughter without being declared an accomplice of the intoxicated driver.
94
Q
  1. A boyfriend and his girlfriend broke into a house late at night with intent to steal a stereo system. Although they believed that the owner was away on a business trip, in fact he was sleeping in an upstairs bedroom. While they were inside the house, the girlfriend announced that she had changed her mind and urged her boyfriend to leave.The owner, who was awakened by the noise downstairs, descended the staircase to investigate. Upon seeing the owner, the girlfriend again urged her boyfriend to flee. Instead, the boyfriend attacked the owner and tied him up with a rope. Thereupon, the boyfriend and his girlfriend left with the owner’s stereo equipment. After they left, the owner choked to death on the ropes while trying to free himself.The boyfriend and his girlfriend were charged with murder but were acquitted. Thereafter, the girlfriend was apprehended and prosecuted for felony murder.Which of the following is the girlfriend’s best argument for acquittal?(A) The acquittal of the girlfriend and her boyfriend for murder precludes any subsequent prosecution under the doctrine of res judicata.(B) The owner’s suicidal effort to free himself was a supervening cause of death.(C) Since the girlfriend changed her mind, she cannot be found guilty of burglary.(D) The girlfriend withdrew from the commission of the underlying felony of burglary.
A
  1. (D) The key to this question is recognizing that the girlfriend is being prosecuted forfelony-murder, not murder. Choice (A) is incorrect because even though the girlfriend was acquitted of the crime of murder, she could still be prosecuted for felony-murder. This is true because felony-murder is a distinct and separate form of murder, in that the crime requires proof of a fact that murder does not (namely, commission of the underlying felony). Choice (A) is incorrect because res judicata, the doctrine of claim preclusion, bars the retrial of the same cause of action. Exam Tip: As a general rule, the res judicata doctrine is applicable in civil, not criminal, cases. Choice (B) is wrong because the owner’s effort to free himself would clearly be foreseeable. By the same token, choice (C) is incorrect, inasmuch as changing one’s mind is never sufficient grounds for withdrawal from a crime. By process of elimination, choice (D) is the strongest answer.
95
Q
  1. A defendant incurred heavy gambling losses to a bookie. Short of cash, the defendant sought help from his uncle. When the uncle refused to lend his irresponsible nephew any money, the defendant decided to get even. One morning, as the housekeeper was preparing his uncle’s breakfast, the defendant stealthily entered the kitchen and, unnoticed, dropped arsenic into the teapot from which his uncle was always served his tea. The defendant then hurriedly left the house and went to play golf. Shortly thereafter, the housekeeper served the uncle the poisoned tea, thus causing his unfortunate death.The defendant’s culpability for his uncle’s death would most probably be as(A) an accessory before the fact.(B) a principal in the first degree.(C) a principal in the second degree.(D) an accomplice.
A
  1. (B) A principal in the first degree may simply be defined as the criminal actor. He is the one who, with the requisite mental state, engages in the act which that causes the criminal result. One who uses an intermediary to commit a crime is not ordinarily a principal in the first degree. But do note the situation where the crime is accomplished by the use of an innocent party (as in the present example with the housekeeper) or irresponsible agent, as where the defendant causes a child or mentally incompetent to engage in conduct. In such a case, the intermediary is regarded as a mere instrument, and the originating actor is still the principal in the first degree. Choices (A) and (C) are incorrect because the defendant’s liability is more than a mere accessory, as stated above. Choice (0) is incorrect for the reasons stated above.
96
Q
  1. A nephew was short of cash and was waiting for his elderly aunt to die. His aunt was currently in the hospital on a ventilator. Her condition was terminal, and the nephew could not wait to get his hands on his inheritance. One night, his girlfriend demanded that he buy her an engagement ring or she would break up with him. Out of time, the nephew went to the hospital and broke into the cabinet where the drugs were kept. Unbeknownst to the defendant, a nurse in the hospital, the nephew injected 1,000 mg’s of a powerful muscle relaxant into the IV bag bound for his aunt’s room. Ten mg’s is the prescribed dosage of the muscle relaxant.The defendant, unaware of the IV’s lethal capability, attached the new bag to the aunt’s IV drip. The aunt’s death was swift, and attempts to revive her were futile. The nephew celebrated his inheritance with a bottle of Champagne with his girlfriend. Unfortunately for the nephew, his aunt had changed her will some months prior to this, and all the aunt’s money was left to her dog.If the defendant is prosecuted for administering the IV, she will most likely be found(A) guilty, as a principal in the first degree.(B) guilty, as a principal in the second degree.(C) guilty, as an accessory after the fact.(D) not guilty.
A
  1. (D) Choice (D) is correct because an innocent agent, without a criminal state of mind, would not be held responsible where she merely left an instrument which that caused the criminal result. If an actor leaves poison for another who later drinks it, he is a first-degree principal, as is the person whose unwitting agent acts for him in his absence. Although it has been said that a principal in the first degree must be present at the commission of the offense, this is not literally so. Choices (A), (B), and (C) are all incorrect, as the defendant is not guilty of any crime, for the reasons stated above.
97
Q
  1. One evening, a husband received a telephone call from an unidentified source who informed him that his wife was having an affair with his friend. As a result, the husband drove to his friend’s house with a loaded shotgun. Believing that his friend was in an upstairs bedroom, the husband fired the shotgun through the dining room window, as he only wanted to teach him a lesson. Unbeknownst to the husband, his friend’s wife was in the dining room and suffered a minor gunshot wound.The husband should be found(A) guilty of attempted murder of his friend’s wife only.(B) guilty of attempted murder of his friend only. (C) guilty of attempted murder of both his friend and his friend’s wife.(D) not guilty of attempted murder of either his friend or his friend’s wife.
A
  1. (D) First, it is necessary to determine upon the available facts whether the necessary elements of an attempt are present. That is, looking at all of the facts, may it be said that there existed both (1) the requisite mental state of a specific intent to commit a murder, and (2) an act in furtherance of that intent beyond mere preparation. Since the facts do not indicate that the husband intended to kill the friend or the wife (in fact, the husband believed that the friend was in an upstairs bedroom), the requisite mens reals lacking, and the husband would not be guilty of attempted murder. Therefore choices (A), (B), and (C) are incorrect. If the wife died from the gunshot, however, the husband would probably be guilty of “depraved-heart” murder.
98
Q
  1. A defendant worked as a patent attorney in a law firm. Late one evening, she left the office and walked outside to her car. While in the parking lot, she noticed a gold watch lying on the ground. When the defendant picked it up, she immediately identified the watch as belonging to one of the other lawyers in her law firm. She took it home with her, intending to return it the next day. The following morning, however, the defendant decided to keep the watch, and did so.The defendant has committed(A) larceny only.(B) embezzlement only.(C) both larceny and embezzlement.(D) neither larceny nor embezzlement.
A
  1. (D) This question deals with a very tricky hornbook area, namely, finders of lost property. According to LaFave, the owner of lost property has “constructive possession” of it as long as actual possession is vacant; therefore, a finderwho misappropriates property may be viewed as taking it by a trespass from the owner’s possession. Such conduct constitutes larceny if, at the time of the finding, the finder 1) intends to steal and 2) either knows who the true owner is or has reason to believe that he can ascertain the owner’s identity. Criminal Law, pp. 711—12. Nevertheless, it is not larceny for the finder to pick up the property with knowledge of its ownership, intending to return it to the owner, even though, later, the finder converts it to his her own use. Ibid. p. 712. This situation fits the facts in our question. The defendant found the gold watch, with knowledge of its ownership, but with the intent to return it. Her intent to keep the watch did not concur with the taking. No larceny has been committed. Therefore, choices (A) and (C) are incorrect. Choice (B) is incorrect because “constructive possession” of the watch was still in the owner. EmbezzLement requires a fraudulent conversion of the personal property of another by one in lawful possession. By process of elimination, choice (D) is the best answer.
99
Q
  1. It was raining hard one evening as a cab driver was looking for one last passenger. A passenger flagged him down and directed him to take her home. The cab driver recognized the passenger as a former girlfriend, and the passenger invited him into her house when they arrived.Unknown to the cab driver, the passenger was an undercover police officer who worked in the sex crimes unit. When they were inside the passenger’s house, the cab driver made sexual advances toward the passenger, who responded by kissing him and holding his hand. The cab driver was about to kiss the passenger again, when she got up to answer the phone in the kitchen.If the cab driver is subsequently arrested and charged with the attempted rape of the passenger, he should be found(A) not guilty.(B) not guilty, solely because he was entrapped. (C) not guilty, if he raises the proper alibi defense. (D) not guilty, if he was predisposed to commit thecrime.
A
  1. (A) Students should recognize that “guilty” is not an alternative. What this question is really asking is, “Why is he not guilty?” The reason is that the victim consented to the defendant’s actions, and there is a question as to whether the defendant committed enough acts to result in an attempt. If the elements of the crime are not present, the issue of entrapment is not even reached. Choice (B) is incorrect because it reads “solely because,” and it would not be the only reason he is not guilty. Choice (D) is intended to confuse you into thinking that the point of the question is to test your ability to define entrapment. If entrapment were the issue, and if the cab driver was predisposed, then he would be “guilty.” If you were to assume that choices (D) and (B) were both correct (which, in fact, they are not), then choice (B) would be preferred as a statement of law, as opposed to the statement of fact in choice (D). In addition, choice (C) is incorrect because there is no alibi defense under the given facts.
100
Q
  1. The defendant was on a date when the victim invited him up to her apartment for coffee. The defendant and the victim began to kiss on the couch. The defendant lifted the victim’s shirt, and the victim gently pushed the defendant’s hand away and then left the couch to make coffee.While the victim was making coffee, the defendant picked up her solid gold ring that she had carelessly left on the living room table. Intending to give it back to her with a diamond in it, he put it into his pocket and got ready to leave.Before the defendant went home, he stopped into a bar. There he met a patron who offered to sell him a two-month-old plasma TV for $200. The patron said that he would take only cash and that he also had some unboxed silverware and jewehy he could sell the defendant from his van, which was parked outside. The patron said, “You better decide fast, because I’ve got to get out of town right away.” The defendant, who knew that the TV was worth $1,000, gave the patron $200, and the patron gave the defendant the TV, which was not in a box. The patron did not tell the defendant he had just stolen the television from a house down the street.Based on the above facts, for which of the following crimes should the defendant be convicted?(A) Battery upon the victim, and larceny of the ring.(B) Larceny of the ring.(C) Embezzlement of the ring.(D) Receiving stolen property.
A
  1. (D) Even though the patron did not tell the defendant that the TV was stolen, there are enough circumstances present to indicate to the defendant that this was the case. Even if one considers choice (D) as only possibly correct, the other three alternatives are wrong. Choice (B) is incorrect because there was no larceny of the ring, since the defendant intended to return it. As a consequence of larceny being incorrect, you need not even considerthe battery upon the victim as presented in choice (A). There was obviously consent to the battery. In addition, there was no embezzlement of the ring, since the defendant was not in lawful possession of it; thus, choice (C) is incorrect.
101
Q
  1. The defendant is on trial for rape of the victim. On the night of the offense, the defendant drove the victim home and asked to come inside for a nightcap. The victim refused, saying she had to be up early in the morning. The defendant then forced the victim upstairs at gunpoint and raped her.During the trial, the defendant took the witness stand and testified that the victim gave him a ride in her car that evening and forced him to go to her house.The defendant’s testimony may be used in a subsequent trial as evidence that the defendant committed the crime of(A) misprision of felony.(B) misprision.(C) peijury.(D) compounding a felony.
A
  1. (C) The correct answer is choice (C), as the defendant would be guilty only of perjury in the subsequent trial. This question is essentially testing you on whether you know what a misprision of felony or a general misprision is. A misprision of felony is an old English common law crime. Someone who is guilty of misprision of felony is someone who failed to report a felony. Exceptions were made for family members of the felon and people who could possibly incriminate themselves by reporting the felony. The defendant here is not guilty of those crimes. Therefore, choices (A) and (B) are incorrect. Choice (D) is incorrect. Compounding a felony is essentially an agreement between a victim and defendant that the victim will not prosecute the defendant in exchange for some monetary gain.
102
Q
  1. Late one evening while the homeowners were away on vacation, Bob entered their home through a broken basement window. Bob knew that the homeowners were collectors of antique weapons. After ransacking the house, he found the prize collection of guns. Bob wrapped the guns in two of the expensive rugs in the home and hurriedly left the house.Upon returning from their vacation, the homeowners notified the police of the theft. During the investigation, a detective received a tip that the guns could be found in an old abandoned warehouse on the riverfront. When the police entered the warehouse, they found Sam with the guns. Upon questioning, Sam told the police that he had planned to dispose of the guns through a fence. With the homeowners’ consent, the police authorized Sam to deliver the guns to the fence and sell them. As soon as the fence paid Sam and took possession of the guns, the police arrested both Sam and the fence.With which of the following crimes should Sam be convicted?(A) Larceny.(B) Possession of stolen property.(C) Burglary and receiving stolen property.(D) Burglary and possession of stolen goods.
A
  1. (B) Choice (B) is the only correct choice, since Sam was merely in possession of the stolen guns when the police raided the warehouse. The burglary was committed by someone else. From the given facts, we may not infer that Sam participated in the burglary. Therefore choices (A), (C), and (0) are incorrect.
103
Q
  1. A defendant stole a car and, while he was driving, the brakes suddenly failed, and the car veered out of control. The car jumped the sidewalk and crashed into a home, causing extensive damage to the dwelling.The defendant was arrested and charged with larceny and the separate crime of malicious destruction of property. At trial, the prosecution and the defense both stipulated that the malfunctioning of the brakes caused the car to veer out of control and damage the home.Assume that the defendant is convicted of larceny for the theft of the car. With respect to the second charge of malicious destruction of property, he should be found(A) not guilty, because the malice requirement is not satisfied, since the destruction resulted from the car’s malfunctioning.(B) not guilty, because malicious destruction of property is a lesser included offense of larceny.(C) guilty, because malice can be inferred from the defendant’s intent to steal.(D) guilty, because malicious destruction of property is a general intent crime.
A
  1. (A) Malicious destruction of property is a separate criminal offense, apart from larceny. Therefore, choice (B) is wrong. Choice (C) is incorrect. Certainly, the defendant was guilty of larceny for stealing the vehicle, but the theft by itself will not satisfy the malice requirement for destruction of property. To be guilty of the crime of malicious destruction of property, there must be evidence presented that the defendant acted maliciously (thus satisfying the mens rea requirement). Choice (D) is incorrect because “general intent” crimes still require a mens rea whether it be malice, criminal negligence, or recklessness, instead of a “specific” intent (e.g., intent to steal). Here, the defendant did not maliciously destroy the property; the damage to the home resulted from the car’s brakes malfunctioning.
104
Q
  1. The defendant was caught in a thunderstorm while walking down the street. As the defendant was about to open an umbrella that she was carrying, a stranger to the defendant came up to her, snatched the umbrella out of the defendant’s hand, and said, “You thief! That’s my umbrella.” Enraged by being accosted in such a manner, the defendant grabbed for the umbrella with one hand and pushed the stranger with the other. The stranger hung on to the umbrella but fell over backward onto the sidewalk, which was wet and slippery. When the defendant saw the stranger hit the ground, she calmed down, decided the umbrella was not worth all the commotion, and walked off. As it turned out, the umbrella did in fact belong to the stranger, and the defendant had picked it up by mistake when she was left a restaurant earlier that day.A few moments later, the stranger got up in a daze and stepped into the gutter, where he was struck by a car that was passing another car on the right, in violation of a state law. The stranger died in the hospital two hours later.Which of the following is the most serious crime for which the defendant could be found guilty?(A) Battery.(B) Larceny.(C) Involuntary manslaughter.(D) No crime.
A
  1. (A) Criminal battery is defined as the unlawful application of force to the person of another, which results in bodily harm, or an offensive touching. Battery is a “general intent” crime where the offensive contact results from the defendant’s negligent or reckless conduct. On the other hand, criminal offenses of a “specific intent” nature require that the defendant intends to injure the victim. One of the defenses to criminal battery is defense of property. One whose lawful possession of property is threatened by the unlawful conduct of another, and who has no time to resort to the law for its protection, may take reasonable steps to protect the property. In the present situation, the defendant obviously was not the rightful owner of the umbrella when she tried to regain possession from the stranger. Nonetheless, the question of whether the defendant was privileged to use reasonable force becomes moot because she actually used excessive force under the circumstances. The facts state that “the defendant grabbed for the umbrella with one hand and pushed Wesley the stranger with the other.” Thus, even if the defendant was privileged to grab the umbrella, she was not privileged to push the stranger. That’s why she remains liable for the criminal offense of battery. In this regard, LaFave states that “one may not use more than reasonable force or the amount of force that reasonably appears necessary to prevent the threatened interference with the property.” For that reason, choice (A) is a more preferable answer than choice (D). See Criminal Law, pp. 399—400. Choice (B) is incorrect because the defendant did not have the intent to commit larceny when she mistakenly grabbed the wrong umbrella. Choice (C) is incorrect because the defendant’s recklessness did not result in the stranger’s death. Also, the stranger’s death did not stem from the defendant’s battery (misdemeanor manslaughter), which would be another theory by which to argue for involuntary manslaughter. Choice (D) is incorrect for the reasons stated above.
105
Q
  1. A victim was leaving his favorite local watering hole and contemplating a late night cheeseburger when he tripped over his own feet and fell on the sidewalk. A few moments later, the victim got up in a daze and stepped into the street, where he was struck by a car driven by the defendant, who was speeding, in violation of a regulation of state law. The victim died in the hospital two hours later.Which of the following is the most serious crime for which the defendant should be found guilty?(A) Speeding.(B) Involuntary manslaughter.(C) Voluntary manslaughter.(D) Murder.
A
  1. (B) Involuntary manslaughter consists of two types: (1) criminal-negligence manslaughter, which requires conduct creating an unreasonable and high degree of risk of death or serious bodily injury (i.e., more than ordinary tort negligence); and (2) unlawful act manslaughter, where the death-causing conduct occurs during the commission or attempted commission of an unlawful act (generally a malum in se misdemeanor) involving a danger of death or serious bodily injury. LaFave, Criminal Law, p. 594. Malum in se crimes generally include morality offenses and serious traffic offenses, as well as criminal assault or intentional battery. By unlawfully passing another car on the right, the defendant committed a traffic violation (i.e., a misdemeanor) that resulted in the victim’s death. The defendant may be found guilty of involuntary manslaughter based on the fact that his unlawful act directly and proximately caused the victim’s death. Therefore, choice (B) is correct. Choice (A) is insufficient, since the motor vehicle violation will merge into misdemeanor manslaughter. Choice (C) is incorrect, since voluntary manslaughter always involves an intentional killing. Choice (D) is incorrect because the defendant’s conduct was not of a high enough degree to constitute depraved or wanton recklessness sufficient for depraved-heart murder.
106
Q
  1. The defendant was walking down the street when he saw a woman struggling with a man over a briefcase. Unbeknownst to the defendant, the woman had just stolen the briefcase from the man on the street. Believing the woman to be the victim of an attack, the defendant intervened and punched the man until the woman was able to get away with the briefcase. Confused as to why he wasn’t being hailed as a hero by the woman, the defendant eventually realized that he had been an unwitting accomplice to the woman’s theft. The defendant apologized profusely to the man and went home.According to the alter ego rule, which of the following statements is correct with respect to the amount of force that the defendant was entitled to use in the woman’s defense?(A) Since the defendant did not stand in any personal relationship with the woman, he was not justified in using force in her defense.(B) Not knowing the true facts, the defendant was not justified in using force to protect the woman because the man was privileged to recapture his briefcase.(C) The defendant was justified in using reasonable force in the woman’s defense, since he reasonably believed she was in immediate danger of unlawful bodily harm from the man.(D) The defendant was justified in using reasonable force in the woman’s defense, since his belief that she was in immediate danger of unlawful bodily harm from the man was both objectively and subjectively reasonable.
A

io6. (B) Generally, with respect to the defense of others, a defendant is justified in defending another person with reasonable force only if he reasonably believes the victim had a right to use such force. That is the majority rule — and that is the rule to apply unless the question instructs otherwise. Here, however, the reader is asked to apply the “alter ego” rule. Under this modern rule, the defendant is viewed as “standing in the shoes” of the person defended, making the right to defend another extensive with the other’s right to defend himself. Thus, the defender who intervenes to protect a perceived victim against a perceived assailant takes the risk that the victim is not, in fact, privileged to defend himself in the manner he employs. Choice (A) is not the best choice. Some jurisdictions limit this defense to situations where a special relationship exists between the defendant and the victim, but application of this minority rule is not prompted by the question. Choices (C) and (0) are incorrect because both choices incorrectly apply the common law reasonable belief standard.

107
Q
  1. This jurisdiction has the following criminal statute in effect:“A person is not responsible for criminal conduct if at the time of such conduct, as a result of mental disease or defect, he lacks substantial capacity to appreciate the wrongfulness of his conduct, or to conform his conduct to the requirements of law.”One afternoon, a defendant was babysitting his five-year-old nephew. As they were playing catch outside, the defendant threw the ball over his nephew’s head, and it rolled into the street. Instinctively, the nephew ran after the ball but tripped over the gutter and fell in the street. When he tripped, the nephew severely sprained his ankle and couldn’t stand up. Moments later, a large garbage truck was backing up and ran over the nephew, killing him. Although the defendant saw his nephew’s predicament, he made no effort to rescue him.Subsequently, the defendant was charged with involuntary manslaughter. At trial, the defendant testified that he was so shocked when he saw his nephew fall near the garbage truck that he froze and was unable to move until it was too late. Following the defendant’s testimony, an expert witness testified for the defense that reactions of the sort described by the defendant are not unusual when a truly shocking event occurs.If the jury believes the testimony of the defendant and his expert witness, the defendant’s best defense is which of the following?(A) The defendant was suffering from temporary insanity.(B) The defendant lacked the requisite mental state required for the commission of the crime.(C) The defendant’s failure to act was not voluntary.(D) The defendant’s criminal liability was superseded by that of the truck driver.
A
  1. (C) In Criminal Law, students must distinguish between actus reus and mens rea. The statute given in the facts is similar to the A.L.I. Model Penal Code test for insanity. As a defense, it will negate the mens rea of the crime committed. The defendant was charged with involuntary manslaughter, a crime requiring criminal negligence. Choices (A) and (B) address the statutory language, in that the defendant could not “conform his conduct (volitionally) to the requirements of law” because he was so shocked that he froze. Although a good argument, both choices use the defense of insanity to negate the mens rea of the crime. Choice (C) is even stronger, however, because an actus reus must have occurred before a proper defense is even relevant. A person may be criminally liable for his omission to act when (1) there is a legal duty to act under the circumstances, and (2) he can physically perform the act. LaFave, Criminal Law, p. 182. If the defendant’s failure to act, where he was babysitting his nephew and had a legal duty to act (based on either relationship or contract), was not deemed to be voluntary, due to the fact that he was so shocked that he was unable to move, then he cannot be criminally responsible for lack of an actus reus. This argument is more basic than the other alternatives because it negates an element of the crime itself rather than using a defense to the crime; therefore, choice (C) is the defendant’s bestdefense and the correct answer. Choice (0) is incorrect for the reasons stated above.
108
Q
  1. Under which of the following fact situations should the defendant be found NOT guilty of the crime committed?(A) A defendant was the treasurer of an electronics company. After remodeling his private residence, the defendant owes an outstanding balance of $25,000 to his contractor. Although he had a $100,000 certificate of deposit, the defendant didn’t want to cash it because he would incur a penalty for early withdrawal. Consequently, the defendant, without authorization, withdrew $25,000 from the company account to pay his contractor. At the time he made this withdrawal, the defendant honestly intended to repay the money by cashing in his certificate of deposit the following week when it matured. The defendant is charged with embezzlement.(B) A defendant was a salesman for a cellular phone company. He honestly believed that the company owed him $10,000 in sales commissions that he had earned. The company president disputed owing the defendant any outstanding commissions and refused to pay him anything. After a heated argument with the president, the defendant was fired from his job. Thereafter, still believing that he was entitled to the $10,000, the defendant barged into the president’s office and, at gunpoint, demanded the money. Under the threat of being shot, the president handed over $10,000 to the defendant. Subsequently, the defendant is charged with robbery.(C) A defendant was a new season ticket holder at basketball games. For the first game of the season, the defendant entered the arena to see his team play their rivals. As he was walking to his seat, the defendant saw a vendor selling yearbooks. When the vendor turned his head to make a sale, the defendant took a yearbook without paying. Thinking that he had stolen the yearbook, the defendant hurriedly walked away. After he had taken a few steps, the defendant for the first time saw a sign that read “All Season Ticket Holders Entitled to a Free Yearbook.” Unknown to the defendant at the time, if he had presented his season ticker stub to the vendor he would have received a free yearbook. The defendant is charged with larceny.(D) A defendant, who was unemployed, opened a charge account at a department store by lying on a credit application that she was employed and earning a yearly salary of $20,000. Using her store credit card, the defendant purchased $1,000 in store merchandise. When the bills came due, she failed to make any payments because she was insolvent. At the time the defendant purchased the items, she honestly thought that she would have a job and be in a position to pay for the goods once the bills came due. The defendant is charged with false pretenses.
A

io8. (B) In choice (A), the defendant will be guilty of embezzlement. In Commonwealth v. Tuckerman, 76 Mass. 173 (1857), a corporate treasurer took company money to spend for his own purposes. Even though he intended to restore the money later and had sufficient funds to do so, he was convicted of embezzlement. Choice (C) is also incorrect. The defendant will be guilty of larceny because at the time the defendant took the yearbook from the vendor, he intended to steal it. The defendant had no claim of right to the yearbook, since, at the time of misappropriation, he was unaware his season ticket stub entitled him to a copy. Choice (D) is incorrect, as well. Here, the defendant will be guilty of false pretenses. At the time she applied for the credit card, she falsely represented her salary and employment status, thereby inducing the department store to issue a card which that would not otherwise have been given to an insolvent applicant. Her honest belief that employment would come in the future is no defense to the fake representation of her present employment status. Choice (B) is correct. The defendant will not be guilty of robbery. Under the claim of right defense, one may take the property of another honestly, but mistakenly, believing it is his own property. LaFave, Criminal Law, p. 538. There is no larceny because there is no intent to steal.

109
Q
  1. A wife was notified by an airline that her husband’s plane had crashed. All passengers aboard were reported lost at sea and presumably drowned. The wife, after making diligent inquiries in good faith, became convinced that her husband was dead. Three years later she re-married. A few months after her re-marriage, the newspaper announced that her husband had been found on a desert island and was rescued. The wife was then prosecuted under the following state bigamy statute.“Whoever, being married, shall marry any other person during the life of the former spouse shall be guilty of a felony: provided, that nothing in this Act shall extend to any person marrying a second time whose spouse shall have been continually absent from such person for a period of seven years last past, and shall not have been known by such person to be living within that time.”On the charge of bigamy, the wife should be found(A) guilty.(B) not guilty, because of the wife’s mistake of fact regarding her husband’s death.(C) not guilty, because of the wife’s mistake of law regarding her husband’s death.(D) not guilty, because the wife did not have the requisite mens rea to be held criminally liable.
A
  1. (A) The prevailing American view is that bigamy is an offense of absolute liability. Even in the minority jurisdictions, a defendant remarrying under a bona fide mistake of fact may be guilty if the belief in the spouse’s death was based on lack of due diligence in determining the facts. Therefore, choices (B), (C), and (0) are incorrect.
110
Q
  1. In which of the following situations will the defendant’s defense of necessity most likely exculpate him of criminal liability?(A) A defendant was marooned on a desert island, along with a victim and another sailor after their boat capsized during a violent storm at sea. The three sailors had gone without food for two weeks and were on the verge of dying of starvation. Each of the sailors, however, still had a small amount of water left in their canteens with which to subsist. One night, the defendant took the water from the victim’s canteen and poured it into his own canteen while the victim slept. The defendant did this in the hope that the extra water would make it possible for him to live a couple of more days so that he would be able to walk across the island to look for help. The next day, the victim died from starvation and dehydration. Two days later, the defendant and the other sailor were rescued. The defendant never walked across the island because he was too weak from body weight loss.(B) A defendant and a victim were on a hiking expedition together. They were longtime hiking buddies who had climbed many steep and precipitous mountains in the past. As the defendant and the victim were climbing a dangerously steep incline, their ropes suddenly became entangled. As a result of the entanglement, the victim fell 50 feet before he was able to regain his balance and footing. However, the victim’s fall caused the defendant to lose his balance, and he was in jeopardy of falling off the mountain. In order to avoid falling, the defendant cut the safety rope that held the victim. This caused the victim to fall 400 feet to his death.(C) A defendant, who was intoxicated, was driving his car home along a heavily traveled highway. In his inebriated condition, the defendant was swerving his vehicle across the highway. Suddenly, a school bus loaded with children made a wrong turn and headed directly into the path of the defendant’s auto. In order to avoid crashing head-on with the school bus, the defendant turned his vehicle onto the sidewalk and struck a victim, a paraplegic, who was confined to a wheelchair. The defendant’s auto crushed the victim to death.(D) A defendant, a bank teller, was on duty when two robbers held up the bank. She was taken hostage and ordered to drive their getaway car. As they were being pursued by the police, the defendant was commanded to drive excessively fast to avoid being captured. While driving at a speed of nearly 100 M.P.H., the defendant approached a busy intersection and saw the victim, a blind person, with a guide dog walking across the street. She slowed down to avoid hitting the pedestrian with the dog. As she did so, one of the robbers stuck a gun against her head and threatened to kill her if she didn’t speed through the intersection. Afraid she would be murdered, the defendant accelerated into the path of the victim, killing him and his guide dog.
A
  1. (C) In situations where the pressure of natural physical forces confronts an individual with a choice of two evils, either to violate the criminal law and produce a harmful result, orto comply with the law and produce a potentially greater harm, the defense of necessity is available to justify the former course of action. LaFave, Criminal Law, p.441. For the defense of necessity to operate, the defendant must have acted with the intention of avoiding the greater harm. In choice (A), the defense of necessity would not be justified, since the defendant still had some water of hi.s own when he took all of the victim’s remaining water, causing his death. The possibility of walking across the island would not outweigh taking a human life. Choice CD) is incorrect because it is an example of duress, not necessity. The defendant is being threatened by human forces, not natural forces. Choice (B) is incorrect. Necessity would not be justified under the facts because the victim had regained his balance and footing when the defendant cut the safety rope, killing him. In this case, killing another to save oneself is murder. Choice (C) is the best answer. The defendant’s intoxication is irrelevant because he did not cause the dangerous situation; rather, the school bus made a wrong turn and headed directly toward the defendant. Killing one person to save many would be justified under these facts.
111
Q
  1. A husband and wife divorced after 17 years of marriage. They had one son, aged 10. As part of the divorce decree, the wife was given custody of the son, while the husband was entitled to weekend visitation.Thereafter, the husband accepted a new job in a neighboring state. Before relocating, the husband met with an attorney to seek his advice about how he could gain full custody of his son. The attorney told the husband that his new state did not give full faith and credit to divorce proceedings in his former state of residence. As a consequence, the attorney advised the husband that he could take the son to live with him and not be in violation of the law. This was erroneous legal advice, and his new state, in fact, did honor and give full faith and credit to other states’ divorce decrees.When his next scheduled visitation took place, the husband picked up his son at his ex-wife’s home. Instead of returning his son, he took him to live with him in his new state. After refusing to return his son to his ex-wife, the husband was subsequently arrested and charged with kidnapping. The applicable statute is defined as “knowingly abducting a person and moving him or her to another location.”Should the husband be found guilty of kidnapping?(A) Yes, because he unlawfully transported his son to another state, in violation of the divorce decree.(B) Yes, because mistake of law is no defense.(C) No, because he lacked the requisite state of mind.(D) No, because he received erroneous legal advice.
A

iii. (C) The kidnapping statute is defined as “knowingly abducting a person.” Based upon the advice given him by his lawyer, the husband believed that he could lawfully take his son to the new state and not be in violation of the law. As a consequence, he did not act knowingly, which would negate the mental state or mens rea requirement for the commission of this crime. Therefore choices (A) and (B) are incorrect. Choice (C) is better than choice (D) because it eliminates an element of the crime. Remember, anytime you can eliminate an element of a crime, that will be a better explanation as to why the defendant is not guilty than to say a defense exists (e.g., intoxication or mistake).

112
Q
  1. Two men agreed to burglarize a home. While they were planning the burglary, the two men learned that the home had a sophisticated alarm system that needed to be disarmed. One of the men told the other that he knew an alarm specialist who could help disarm the security system.One of the men then approached the alarm specialist and asked if he would assist them in disarming the home’s alarm system. The alarm specialist said that he didn’t want to participate in the crime but told the man how he could disarm the system himself. The two men thereafter went to the home to commit the burglary.When they arrived at the home, they saw a vicious guard dog patrolling the fenced-in area of the home. Deciding it would be too risky to confront the dog, the two men abandoned their planned burglary.Which of the following is the most accurate statement regarding the criminal liability of the two men and the alarm specialist?(A) The two men are guilty of conspiracy.(B) The two men and the alarm specialist are guilty of conspiracy.(C) The two men are guilty of conspiracy and the two men and the alarm specialist are guilty of attempted burglary.(D) The two men and the alarm specialist are guilty of both conspiracy and attempted burglary.
A
  1. (B) Clearly, the two men have entered into an agreement to burglarize the home and are guilty of conspiracy. The more difficult question is whether the alarm specialist can become a party to the crime of conspiracy, even in the absence of any agreement on his part. According to LaFave, it is possible for a person to become a member of a con sp i racy where he knew of the conspfracy and intentionally gave aid to the conspiratorial objective even though he did not enter into the agreement himself. Cri m nal Law, pg. 534. ConsequentLy, the alarm specialist is guilty of conspiracy because he aided the conspiratorial objective by telling the two men how they could disarm the home’s alarm system with the knowledge that they were planning to perpetrate a burglary. Therefore choice (A) is incorrect. The next issue is which, if any, of the defendants are guilty of attempt. At common law, attempt requires an act that constitutes a “substantial step” in the commission or attempted commission of a crime. The two men did not perform a “substantial step” merely by going to the home without any entry. Therefore choices (C) and (D) are incorrect.
113
Q
  1. A defendant worked as a short-order cook at a restaurant. After work, the defendant went out drinking with one of the other cooks, and they were commiserating about their lack of money. They decided that the only way out of their rut was to rob a local bank.They then agreed to carry out a bank heist on the following Friday. In accordance with their plan, the cook on Monday purchased two ski masks to be used in the robbery. On Wednesday, after learning that the cook had bought the masks, the defendant got cold feet and told the cook he was renouncing his involvement in the robbery scheme. On Friday, the cook carried out the robbery without the defendant’s participation. This jurisdiction requires an overt act for the crime of conspiracy.Is the defendant guilty of conspiracy?(A) Yes, because the cook and the defendant did enter into an agreement to commit the bank robbery.(B) Yes, because the purchase of the ski masks was a sufficient overt act in the furtherance of the crime.(C) Yes, because the defendant did not thwart the cook from committing the robbery.(D) No, because the defendant’s withdrawal was effective.
A
  1. (B) In this jurisdiction, conspiracy requires an overt act. Common law conspiracy did not require an overt act; a conspiracy was punishable even though no act was done beyond the mere making of the agreement. Many states, however, do require an overt act for conspiracy. According to LaFave, the Supreme Court “currently regards the overt act merely as evidence of the offense.” Criminal Law, pg. 548. The function of the overt act in a conspiracy prosecution is simply to manifest “that the conspiracy is at work.” Consequently, the purchase of the ski masks (to be used in the robbery) would clearly constitute an overt act. Thus, choice (B) is correct. Choice (A) is incorrect because this jurisdiction requires more than just an agreement by two or more individuals with the intent to achieve a criminal objective. Choices (C) and CD) are wrong because the traditional common law rule is that withdrawal is not a defense to conspiracy because the crime is complete with the agreement. Moreover, in jurisdictions which that have added an overt act requirement, withdrawal is effective only if it occurs before the overt act has been committed. Since the overt act of buying the ski masks occurred before the attempted withdrawal, the defendant would be guilty of conspiracy.
114
Q
  1. A defendant was wearing a black leather jacket when he entered a deli for lunch. He placed his jacket on a coat rack located in the front of the deli. After his meal, the defendant picked up a similar black leather jacket, believing it to be his own. The jacket he took, however, belonged to another customer. The defendant left the deli and walked a short distance before realizing he had taken the wrong jacket by mistake. He then returned to the deli and placed the jacket back on the coat rack. He found his own jacket, which had been partially hidden under a stack of other coats.If the defendant were arrested and charged with larceny of the other customer’s jacket, he will most likely be acquitted because(A) there was a mistake of fact.(B) he returned the jacket after discovering his mistake.(C) he lacked the requisite state of mind.(D) there was no fraudulent conversion.
A
  1. (C) Larceny at common law may be defined as the (1) trespassory (2) taking and (3) carrying away of the (4) personal property (5) of another (6) with intent to steal it. The crime of larceny requires an intent to steal, (i.e., an intent to deprive the owner of the possession of her property permanently or for an unreasonable length of time). The defendant did not possess an intent to steal (i.e., no animus furandi). As a result, choice (C) is correct. Choice (A) is very appealing, as it is a defense which that would apply on these facts. However, an answer choice that cuts off culpability to a defendant by knocking out an element of the crime charged is always a stronger answer than a valid defense. Choice (B) is incorrect because returning the property once a larceny is complete is not a valid defense. Finally, choice (D) is incorrect because a fraudulent conversion is not an element of larceny, but, rather, embezzlement.
115
Q
  1. In which of the following situations would the defendant’s intoxication NOT be able to negate his criminal culpability?(A) A defendant had consumed a fifth of bourbon. Later that same day, he approached a victim and, brandishing a knife, told her to accompany him or he would stab her. He led the victim to his car and then ordered her to disrobe. As the victim was removing her pantyhose, she kicked the defendant in the head, temporarily dazing him. The victim then safely ran from the car. The defendant is arrested and charged with the crime of assault with the intent to commit rape.(B) A defendant attended a wedding reception at a hotel, where he drank several vodka daiquiris. Following the reception, the defendant engaged in a violent argument with the hotel’s parking lot attendant. The defendant took a tire iron from his car and threw it at the attendant. The tire iron missed the attendant and hit a victim as he was entering the hotel. The defendant is arrested and charged with assault with the intent to commit battery.(C) A defendant had been drinking liquor all evening at a bar with three of his buddies. An undercover detective overheard the defendant and his buddies plot to rob the bar after closing hours. When the defendant attempted to draw a gun from his coat, he was quickly disarmed and placed under arrest by the detective. The defendant is charged with the crime of conspiracy to commit robbery.(D) At his law school graduation party, a defendant drank two six-packs of beer. Around midnight, the defendant was approached by a girl, who asked him to drive her home. Although the girl was only 15 years old, she had the appearance of a woman in her mid-to-late twenties. The defendant, who had had his eye on the girl all night, quickly agreed, and he showed her the way to his car. Once inside, they engaged in sexual intercourse. The age of consent in this jurisdiction is 17 years old for females. The defendant is subsequently arrested and charged with statutory rape.
A
  1. (D) Students should note the distinction between specific and general intent crimes. According to LaFave in his Criminal Law hornbook, it is sometimes stated that intoxication can negate a specific intent which that the crime in question may require (meaning some intent, in addition to the intent to do the physical act which that the crime requires), but it cannot negate a crime’s general intent. In this regard, the crimes of (a) assault with intent to commit rape, (b) assault with intent to commit battery, and (c) conspiracy are all specific intent crimes. Therefore, intoxication may serve as a defense to the specific intent crimes enumerated in choices (A), (B), and (C). However, intoxication will not negate the criminal culpability of the defendant in choice (D) because statutory rape is a crime which that imposes “absolute” criminal liability (i.e., no available defense).
116
Q
  1. A defendant had been drinking at a bar for three hours and was visibly intoxicated. A man entered the bar and sat down next to the defendant. After ordering a beer, the man turned to the defendant and said, “Hey buddy, you’re sure an ugly looking dude.” The defendant ignored the man’s insult and turned to walk away. The man then pushed the defendant against the bar and said, “Your face makes me sick to my stomach.” The defendant then pulled out a razor and slashed the man’s throat, killing him.If the defendant is prosecuted for the man’s murder, he will most likely be founda. guilty, because his intoxication was voluntary. b. guilty, because he was under a duty to retreat. c. not guilty, because of his intoxication. d. not guilty, because there is no duty to retreat in a public place.
A
  1. (A) The majority of jurisdictions hold that, while voluntary intoxication may be so great as to negate premeditation and deliberation, this fact serves only to reduce the homicide from first-degree to second-degree murder. Therefore, choice (C) is incorrect. Choice (D) is incorrect. A basic tenet of criminal law is that one may not use more force, in self-defense, than is reasonably necessary. Choice (B) is incorrect, since one who can safely retreat need not do so before using non-deadly force. Thus, the question of retreat is a problem only when deadly force is employed in self-defense. The prevailing view is that the defender who was not the original aggressor need not retreat, even though he can do so safely, before using deadly force upon an assailant whom he reasonably believes will kill him or inflict serious bodily harm.
117
Q
  1. In which case would the defendant’s intoxication defense most likely negate his criminal intent?(A) A defendant is charged with raping a victim. At trial, the defendant testifies that he was so inebriated that he was unable to understand that the victim did not consent to his conduct.(B) A victim was horseback riding when she was approached by a defendant, who rode up from behind and struck her horse with his riding crop, causing the horse to bolt and throw the victim. On trial for battery, the defendant testifled that he was drunk and only fooling around and did not intend to injure the victim.(C) While intoxicated, a defendant wandered into a victim’s barn, lit a match, and began looking for some whiskey that he thought was hidden there. Angered at not finding any liquor, the defendant threw the match into a bale of hay, which quickly ignited, thus causing the destruction of the victim’s barn. The defendant is charged with arson.(D) A defendant is charged with assault with intent to commit rape. While on trial, the defendant testified that he was intoxicated to such an extent that he did not remember striking the victim.
A
  1. (D) Intoxication can negate a “specific intent” crime, but it cannot negate a “generalintent” crime. So-called “specific intent” crimes require two elements: (1) an actusreus (or the criminal act), and (2) the mens rea (or “guilty mind”). Common law larceny, for exampLe, requires the taking and carrying away of the property of another,but it must also be shown thatthere was an intenttostealthe property. Common lawburglary requires a breaking and entry into the dwelling of another, but it must alsobe established that the defendant acted with the intent to commitafelonytherein.On the other hand, so-called “general intent” crimes require only an actus reus. Inthis regard, the crimes of (1) rape, (2) battery, and (3) arson are commonly referredto as “general intent” crimes. These crimes are completed by the criminal act without regard to the defendant’s intention. The crime of rape, for example, is committed by the act of non-consensual sexual intercourse, irrespective of the defendant’sintention. A key Multistate testing area deals with the distinction between crimesof rape and assault with intent to commit rape. Although rape is a “general intent”crime, assault with intent to commit rape is a “specific intent” crime. Similarly,while battery is a “general intent” crime, assault with intent to commit battery is a“specific intent” crime. Therefore, choices (A), (B), and (C) are incorrect.
118
Q
  1. A state enacted a statute making it illegal to knowingly sell, purchase, or in anyway distribute any form of tobacco to a minor. Violation of the statute was a misdemeanor punishable by a $500 fine and up to 30 days in jail. After the statute’s enactment, a defendant sold a pack of cigarettes to a girl who was 17 years of age. Before selling the product to the girl, the defendant carefully examined the girl’s driver’s license, which indicated that she was, in fact, 17 years old. The defendant nevertheless made the sale because he erroneously believed the age of majority to be 17. The defendant is subsequently charged with violation of the statute, and his mistake is honestly held to have been made.Such a mistake should(A) not prevent his conviction, because mistake of the law is no defense.(B) not prevent his conviction, because the crime imposes absolute criminal liability.(C) result in his acquittal, because he took reasonable steps to ascertain the girl’s age.(D) result in his acquittal, because he did not possess the requisite mens rea.
A
  1. (D) Many statutes defining conduct which that is criminal employ words or phrasesindicating some type of mens rea (or guilty mind) requirement: (e.g., “intentionally”; “knowingly”; “purposely”; or “fraudulently.”). In this regard, LaFave pointsout that such crimes require “subjective fault” — actually a guilty mind of some sort.For example, the statutory crime of receiving stolen property is generally worded interms of receiving stolen property “knowing the property to be stolen.” Such wording requires that the defendant, to be guilty, must know in his own mind (i.e., subjectively) that the property he receives is stolen. Similarly, in the present example,since the defendant did not (subjectively) know that he was selling tobacco to aminor, he did not possess the requisite mens rea. Therefore, choices (A), (B), and(C) are incorrect.
119
Q
  1. Two brothers broke into the home of a bank’s president and kidnapped the president’s wife. The brothers took the wife hostage, pointed a gun at her head, and demanded that she drive them in her car to their hideout. As the wife was driving down the street, a woman pushing her baby in a stroller suddenly stopped in the middle of the intersection. In order to avoid hitting the woman and her child, the wife veered the car to the left, and intentionally struck a bystander, instantly killing him.If the wife is subsequently prosecuted for the murder of the bystander, she will most likely be found(A) guilty, because she intentionally veered the car, striking the bystander.(B) not guilty, by reason of necessity.(C) not guilty, by reason of duress.(D) not guilty, by reason of self-defense.
A
  1. (B) Generally, the defense of necessity is limited to those situations where the pressure comes from the physical forces of nature (i.e.,e.g., storms, hurricanes, earthquakes) rather than from other human beings. Usually, when the pressure is fromhuman beings, the defense, if applicable, is called duress, rather than necessity.However, choice (C) is incorrect because it has been held that duress cannot justifyan intentional killing. As a result, choice (B) is preferred in light of the Model PenalCode commentaries, which suggest that the defense (of necessity) should be available in the situation where a person intentionally kills one person in order to savetwo or more. Therefore, choice (A) is incorrect. Choice (D) is incorrect because thefacts do not indicate that the wife was acting in self-defense.
120
Q
  1. Believing she was pregnant, a woman went to see a doctor, a licensed gynecologist, because she wanted an abortion. The doctor examined her and determined that she was not pregnant, but decided that he would tell her she was, in order to earn his $500 fee. After receiving the $500, the doctor proceeded to have the woman admitted to a hospital by falsely informing the hospital authorities that she had a benign tumor on her uterus, which he was going to remove. He performed all the surgical procedures appropriate for an abortion under adequate hygienic conditions, but the woman began hemorrhaging and died.The doctor should be found guilty of(A) murder and larceny by trick.(B) manslaughter and false pretenses.(C) battery and larceny by trick.(D) murder, battery, and false pretenses.
A
  1. (B) The doctor is guilty of involuntary manslaughter based on criminal-negligence. For criminal-negligence manslaughter, most jurisdictions require that the defendant’s death-causing conduct involve a higher degree of negligence than ordinary (tort) negligence. In addition, most jurisdictions require that the defendant’s conduct create an unreasonable and high degree of risk of death or serious bodily injury to another. CLearly, the doctor’s conduct (in performing a surgical operation on a woman who wasn’t pregnant) was grossly or criminally negligent. He would, therefore, be guilty of manslaughter because the woman’s death was causally connected to the unnecessary operation. Also, the doctor wouLd be guilty of false pretenses, since he defrauded the woman of $500. False pretenses, although defined in slightly different ways in the various jurisdictions, consist of these five elements:(1) a false representation of a material fact, (2) which causes the victim (3) to pass title to (4) his property to the wrongdoer, (5) who knows his representation to be false and intends to defraud the victim. Choices (A) and (D) are incorrect because the elements of murder are not met. Choices (C) and (D) are wrong. The battery merges with the greater offense. The doctor may have been guilty of the battery of the woman had she survived the surgery but is criminally liable for homicide, not battery, as the same “harmful or offensive touching” that otherwise would have constituted a battery caused the woman’s death.
121
Q
  1. A young boy was one of the players on a little league baseball team. The boy’s father hated the coach because he rarely gave his son any playing time. The boy was mostly a benchwarmer who played only an inning or two during the game.One night, the father hid behind some shrubbery in front of the coach’s home. Armed with a baseball bat, the father intended to break the coach’s kneecaps when he came home from work. A man, who resembled the coach, was walking down the street. The father mistakenly believed that this man was the coach. The father jumped out of the bushes and hit the man in the kneecaps with the baseball bat. The man, who was a hemophiliac, died as a result of the injuries suffered in the beating.The father is(A) guilty of attempted murder of the coach and the murder of the man.(B) guilty of attempted murder of both the coach and the man.(C) guilty of attempted manslaughter of the coach and murder of the man.(D) guilty of attempted battery of the coach and murder of the man.
A
  1. (D) In order to be guilty of attempt, the defendant must specifically intend to commit the “target offense.” Choices (A) and (B) are, therefore, incorrect because the father intended to physically injure the coach, not to kill him. Choice (C) is likewise incorrect because attempted manslaughter is a legal anomaly. It is impossible to specificaLLy intend to commit an unintentional killing. Choice (D) is correct because the father did specifically intend to commit a battery on the coach. Furthermore, the fatherwill be guilty of the murder of the man, based on intent to cause serious bodily harm.
122
Q
  1. A defendant was driving his car recklessly at a high rate of speed through a residential neighborhood. He was traveling at a speed of over 100 M.P.H. when he lost control of the car and jumped a curb, striking a woman who was walking along the sidewalk. As a result of the collision, the woman suffered severe internal injuries and fractured both legs. She was hospitalized for 11 months and became permanently disabled.If the defendant is charged with attempted murder, he should be found(A) guilty, because a person is presumed to intend the natural and probable consequences of his acts.(B) guilty, because criminal liability is predicated upon the defendant’s willful and wanton disregard for the safety of others.(C) not guilty, because the defendant did not intend to kill the woman.(D) not guilty, because he lost control of the vehicle.
A
  1. (C) Be aware that the defendant is charged with attempted murder. LaFave and Scott in their Handbook on Criminal Law point out that the crime of attempt consists of:(1) an intent to do an act or to bring about certain consequences, which would, in law, amount to a crime, and (2) an act in furtherance of that intent which, as it is most commonly put, goes beyond mere preparation. As such, attempt is a specific intent crime. Since the defendant in this example did not have the (specific) intent to kill the woman, he would be found not guilty of the inchoate crime of attempted murder. Therefore, choices (A) and (B) are incorrect. Choice (D) is incorrect because while it is a correct statement of fact, the correct statement of Law in choice (C) is a stronger answer.
123
Q
  1. A man was at a pub drinking beer and playing darts with a group of friends. Quite intoxicated, the man left the bar around midnight. He was staggering down the street when the defendant approached him from behind. The defendant took out a stick, stuck it against the man’s back and told the man to give him his wallet or he was going to shoot him. Frightened, the man fainted and fell to the pavement. The defendant reached into the man’s pants pocket and stole his wallet.The defendant is guilty of(A) attempted robbery.(B) robbery.(C) larceny.(D) assault and larceny.
A
  1. (B) Robbery, a common law felony, consists of the six elements of larceny: (1) a trespassory (2) taking and (3) carrying away of the (4) personal property (5) of another (6) with intent to steal, plus two additional requirements: that (7) the taking be accomplished by force, violence, or intimidation, and (8) the property be taken from the victim or from his presence. To be sure, robbery requires that the taking be done by means of violence or intimidation. The elements of force and fear (orviolence or intimidation) are alternatives; if there is force, there need be no fear, and vice versa. As with the crime of assault, the word “fear” in connection with robbery does not so much mean “fright” as it means “apprehension.” With regard to the so-called “fear” factor, LaFave states that “if the circumstances are such that a reasonable person would not be scared, a jury might properly infer that the victim, in spite of his testimony to the contrary, was not in fact scared.” Criminal Law, pg. 785. But if the victim is actually frightened by the defendant into parting with his property, the defendant is guiltyof robbery. In the present case, the facts clearly indicate that the man was frightened. As a result, the defendant is guilty of robbery, despite the fact that he threatened the victim with a non-lethal stick. Therefore, choices (A), (C), and CD) are incorrect.
124
Q
  1. In which of the following situations is the defendant most likely to be found guilty of manslaughter rather than murder?(A) Not intending to kill, a defendant hits a person over the head with a pipe. As a result, the person dies from head injuries.(B) Not intending to kill, a defendant throws a large chunk of ice off an overpass onto a busy freeway below. The chunk of ice crashes through a windshield, killing a driver.(C) Not intending to kill, a defendant kidnaps a victim and takes her to a remote area of the desert. The defendant then drives off, leaving victim alone in the sweltering heat after the ransom is not paid. The next day, the victim dies from exposure because she is unable to reach the nearest town eight miles away.(D) Not intending to kill, a defendant punches a victim in the face, causing the victim to fall backwards and strike his head on a curb, killing him.
A
  1. (D) Choice (A) is an example of intent-to-inflict serious bodily injury murder. Choice (B) is an example of “depraved-heart” murder. Choice (C) is an example of felony murder. Conversely, choice (D) is characteristic of involuntary manslaughter or misdemeanor-manslaughter. Note that the defendant committed a battery (which, at common law, was a misdemeanor) by punching the victim in the face.
125
Q
  1. A defendant gave a man a gun and instructed him to kill a victim. The man shot and killed the victim with the gun. The man was charged with murder and acquitted.The defendant is now charged with the murder of the victim. He will be able to assert a valid defense if the man’s acquittal was based on which of the following?(A) Insanity.(B) Self-defense.(C) Diminished capacity.(D) The Wharton Rule.
A
  1. (B) This question deals with the interrelated issues of accomplice liability and defenses. if the man had been convicted of murder, then the defendant would also be criminally liable under an accomplice theory of liability because (1) he had the specific intent that the killing take place, and (2) he aided and abetted in the perpetration of the crime. However, if the acts of the principal in the first degree are found not to be criminal, then the accomplice, or principal in the second degree, cannot not be convicted of the crime. In this regard, there are certain defenses available to the principal in the first degree that are likewise available to the accomplice. One such example is self-defense, which justifies the homicide, therefore making it non—criminal. Thus, choice (B) is correct. Some defenses are personal to the principal and may not be asserted by the accomplice, such as insanity. insanity serves to excuse — not justify — the otherwise criminal act. Thus, choice (A) is wrong. Choice (C) is incorrect because diminished capacity would merely negate the specific intent to kiLl, thus mitigating the crime to manslaughter, but does not make the offense non-criminal. This is also a defense that is personal to the principal, if the man lacked the capacity to form the requisite intent, the defendant’s intent is not negated. Finally choice (D) is a distracter. Under the Wharton Rule, where two or more people are necessary for the commission of the substantive offense (e.g., adultery, dueling, bribery), there is no crime of conspiracy UNLESS more persons participate in the agreement than are necessary for the crime.
126
Q
  1. During a drunken quarrel between a husband and a wife, the husband pointed his gun at his wife and said, “If I didn’t love you, I’d kill you.” The husband thought the gun was unloaded but, in fact, earlier that day his son had loaded it. As a joke, he fired the gun at his wife, wounding her in the shoulder.The husband is later charged with committing a battery upon his wife. If the husband attempts to prove that he was so inebriated he could not have formed a criminal intent, this would constitute a(A) good defense, because the charge requires a specific intent.(B) good defense, because at least a general criminal intent is required for every offense.(C) poor defense, because voluntary intoxication is not a valid defense to battery.(D) poor defense, because the husband was not aware that the gun was loaded.
A
  1. (C) Voluntary intoxication is a defense only if it disproves the existence of a specific intent required for the crime. Choice (C) is correct, since voluntary intoxication cannot be a defense to battery, a general intent crime. Choices (A) and (B) are incorrect statements of law. Choice (D) is irrelevant because the husband’s belief that the gun was unloaded would not affect his intoxication defense.
127
Q
  1. An ex-con who had just been released from prison approached two men and asked if they wanted to take part in a bank robbery. The two men both agreed. The ex-con went ahead and planned the robbery. As part of his scheme, the ex-con stole a van, which he intended to use as the getaway vehicle.According to the ex-con’s plan, he would pick up the two men in the van on Friday morning and drive over to the bank where the robbery would occur. The ex-con instructed his cohort that he would be the getaway driver and wait in the van while they entered the bank, armed with shotguns. However, the day before the robbery was to take place, the excon was arrested on a parole violation for carrying a concealed weapon and was taken into custody. The two men, nevertheless, decided to carry out the robbery using the van that the ex-con had stolen. On Friday moming, the two men drove to the bank. When they entered, an undercover police detective was waiting and arrested the two men, as the police had received an anonymous tip regarding the robbery earlier that morning.The ex-con should be found guilty for which of the following crimes?(A) Automobile theft and solicitation.(B) Automobile theft and conspiracy.(C) Automobile theft and attempted robbery.(D) Automobile theft, conspiracy, and attempted robbery.
A
  1. (D) Conspiracy may be defined as (1) an agreement between two or more persons, which constitutes the act, and (2) an intent to thereby achieve a certain objective which, under the common law definition, is the doing of either an unlawful act or a lawful act by unlawful means. The agreement between the ex-con and the two men to take part in the bank robbery formed the conspiracy. The ex-con is guilty of automobile theft because he stole the van. Furthermore, the ex-con is guilty of attempted robbery because he (1) intended to rob the bank, and (2) performed an act in furtherance of the crime by stealing the carthat he intended to use in the bank robbery. Although many students will incorrectly choose choice (C), choice (D) is a better answer because the “substantial step” requirement was satisfied, thereby making the ex-con guilty of attempt, as well as conspiracy. Therefore, choices (A) and (B) are incorrect, as well.
128
Q
  1. A wealthy woman often wore expensive jewelry while walking her dog in the park. Her friends warned her against wearing such valuable jewelry because they feared she would be an easy target for muggers. In order to persuade the woman not to wear her expensive jewelry in the park, her friend decided to play a practical joke. One morning, the friend dressed like a man and hid in an area of the park that she knew the woman customarily walked through. As the woman was strolling through the park with her dog that morning, the friend jumped out from behind the bush brandishing a toy pistol and grabbed the woman’s diamond necklace from her neck. Startled, the woman became hysterical and began to plead for her life. The friend then removed her male garb, handed the necklace back to the woman and said, “I just wanted to frighten you to teach you a lesson.”If the friend is subsequently prosecuted, she should be found guilty of which, if any, of the following crimes?(A) Battery.(B) Assault.(C) Robbery.(D) No crime.
A
  1. (A) Obviously, the friend committed a battery when she grabbed the necklace from the woman’s neck. Criminal battery is defined as the unlawful application of force to the person of another. On the other hand, there are two types of criminal assault:(1) the attempted-battery type, and (2) the intent-to-frighten type. The attempted- battery type of assault requires an intent to commit a battery, (i.e., an intent to cause physical injury to the victim). Since the friend did not intend to injure the woman, she would not be guilty of the attempted-battery type of assault (even though she is guilty of the second type). Note that a majority of jurisdictions follow the attempted-battery type of assault, while only a minority of states recognize the intent-to-frighten type. Therefore, choice (B) is incorrect. Furthermore, choice (C) is wrong because robbery requires an intent to steal. Since the friend did not intend to steal the woman’s necklace, she cannot be convicted of robbery.
129
Q
  1. Shortly after breaking up with the defendant, a woman began dating the victim. The defendant, who still loved the woman, hated the victim. During a holiday weekend, the woman and the victim arranged to go camping in a federal park. The defendant and his friend decided to beat up the victim while he and the woman were on their camping trip. They went to the campsite where the woman and the victim were staying, but they couldn’t find the couple, who were hiking in the woods.Subsequently, the defendant was arrested and charged with conspiracy to commit an assault in a federal park. At trial, the defendant testified that he didn’t know he was in a federal park. Moreover, he stated that if he had known, he would never have agreed to the crime.If the jury believes the defendant, he should be found(A) guilty, because federal conspiracy laws do not require the mental retainment ofjurisdictional requirements.(B) guilty, because federal conspiracy laws require only an intent to commit a prohibited act, but do not require a knowledge of the surrounding circumstances.(C) not guilty, because he didn’t have the specific intent to commit the crime of assault in a federal park.(D) not guilty, because he did not agree to commit a crime in a federal park.
A
  1. (A) Another extremely popular Multistate testing area deals with federal statutory crimes, the reason being that this is one of the “gaps” or areas not adequately covered in the “general outline” courses. With respect to federal criminal law, the national government of the United States has very broad “police powers” to create crimes over conduct in federally owned or controlled territory not within the jurisdiction of any state. Thus, the federal government has territorial jurisdiction over (a) conduct on federal land areas not located within the states, such as the District of Columbia and Territories; (b) conduct on federaL enclaves (islands of federal territory located within the states), such as army posts, naval bases, post offices, and national parks; (c) conduct on ships and aircraft of American nationality when outside the jurisdiction of the states, as on the high seas or even in foreign waters; and (d) conduct by U.S. citizens which that takes place outside the jurisdiction of any state. For the most part, LaFave points out that federal criminal laws define offenses in terms of substantive misbehavior (e.g., theft) and the matter of jurisdictional requirements are dealt with separately. In fact, under the Federal Criminal Code, technical issues of jurisdiction are not prominent, “and the government is relieved of any burden of showing that the defendant knew of the special fact which results in federal jurisdiction.” LaFave, Criminal Law, pg. 113. By the same token, since federal conspiracy laws do not require the mental retainment of jurisdictional requirements, choice (A) is correct. Choices (B), (C) ,and (D) are all misstatements of law and, therefore, are incorrect.
130
Q
  1. A defendant decided to rob a bank. She used an unloaded gun in the robbeiy. The defendant approached the bank teller, pointed her unloaded pistol at him and said, “This is a stick-up. . . give me your money and no one will get hurt.”While the teller was handing the money to the defendant, the victim, a bank patron who was in line, saw the defendant pointing the gun at the teller and fainted. He fell backward and cracked his head on the marble floor. This resulted in a fatal head injury. Moments later, the defendant left the bank with the money. Thereafter, the defendant was arrested.If the defendant is prosecuted for felony murder and acquitted, the most likely reason will be because(A) her gun was unloaded.(B) there was not a sufficient connection between the victim’s death and the robbery.(C) the defendant didn’t intend to harm anyone during the robbery.(D) this jurisdiction has adopted the Redline limitation to the felony murder rule.
A
  1. (B) At early common law, one whose conduct brought about an unintended death in the commission or attempted commission of a felony was guilty of (felony) murder. American jurisdictions, however, have limited the rule in one or more of the following ways: (1) by permitting its use only as to certain types of felonies; (2) by more strict interpretation of the requirement of proximate or legal cause; and (3) by a narrower construction of the time period during which the felony is in the process of commission. LaFave, Criminal Law, pg. 545. With respect to the proximate or legal cause limitation, it is often said that the death must be a foreseeable consequence of the felony. That is to say, the death must have been the “natural and probable consequence” of the defendant’s conduct. Choice (B) is the best answer because looking at the matter with hindsight, it seems extraordinary that a death would actually come about in such an unforeseeable manner. Choice (A) is incorrect because an unloaded gun is irrelevant for robbery and felony murder. Choice (C) is incorrect because felony murder is based typically on unintended deaths which that result from the commission of certain enumerated felonies. Choice (D) is incorrect because the Redline limitation deals with co-felons, and the defendant acted alone here.
131
Q
  1. The defendant and her co-felon decided to rob a bank. They agreed to use unloaded guns in the robbery. As planned, the co-felon entered the bank while the defendant stationed herself outside as a lookout. The co-felon approached the bank teller, pointed her unloaded gun at him, and demanded themoney.While the teller was handing the money to the co-felon, the defendant got scared and fled. The teller, who had a weak heart started to feel faint. He handed over the money to the co-felon, and then clutched his chest and died.The co-felon was found not guilty by a jury in her trial, which preceded the defendant’s trial.The defendant is subsequently prosecuted for conspiracy and acquitted. Her acquittal most likely resulted because(A) the defendant abandoned her participation in the crime.(B) the teller’s death was accidental.(C) the co-felon was acquitted.(D) the teller was not placed in apprehension of bodily harm.
A
  1. (C) To constitute a conspiracy, there “must be a combination of two or more persons.” LaFave notes that this plurality requirement might be restated in terms of at least two guilty parties, for acquittal of all persons with whom the defendant is alleged to have conspired precludes his (or her) conviction. Criminal Law, pg. 488. Thus, if A and B are jointly charged with a conspiracy not alleged to involve any other parties and the jury returns a verdict of guilty as to A and not guilty as to B, A’s conviction may not stand. Martinez v. People, 267 R2d 654 (1954). Choice (A) is incorrect because her withdrawal from the conspiracy was not communicated to her co-felon and, therefore, her liability for crimes committed in furtherance of the conspiracy would not be abrogated. Choice (B) is incorrect because felony murder typically applies to unintended deaths which that result from the commission of certain enumerated felonies. Choice (D) is incorrect because whether the teller was placed in apprehension of bodily harm or not is irrelevant.
132
Q
  1. After weeks of deliberation, a boyfriend decided to embark on his plan to rob a bank. As part of his scheme, the boyfriend enlisted his girlfriend, who agreed to drive the getaway car. On the day of the robbery, the girlfriend and the boyfriend drove to the bank. After the girlfriend parked outside and the boyfriend went into the bank, she saw a security guard in the bank and changed her mind and fled on foot, leaving the keys in the car with the motor running. A few minutes later, the boyfriend went into the bank, robbed it, and then ran back to the car. He drove off and was speeding away from the bank when he looked behind to see if he was being followed. As he took his eye off the road, the boyfriend’s vehicle struck a pedestrian who was crossing the street, killing her.For the pedestrian’s death, the girlfriend should be found guilty of which, if any, of the following crimes?(A) Murder.(B) Involuntary manslaughter.(C) Voluntary manslaughter.(D) No crime.
A
  1. (A) This question is testing students on the scope of liability for conspiracy. First, it is important to realize that a conspiracy existed. According to LaFave, a conspiracy requires “(1) an agreement between two or more persons which constitutes the act, and (2) an intent thereby to achieve a certain objective which is the doing of either an unlawful act ora lawful act by unlawful means.” LaFave, Criminal Law, p. 525. As part of the boyfriend’s plan to rob the bank, he enlisted his girlfriend to drive the get-away car, and she agreed. Her subsequent decision not to go through with it is an insufficient basis for withdrawal, since she did not “thwart the success” of the conspiracy. Choices (B), (C), and CD) are incorrect. The general rule regarding the scope of the conspiracy is that a co-conspirator will be liable for all crimes committed in furtherance of the conspiracy. Since the killing of the pedestrian occurred during the commission of the robbery, the girlfriend will be liable to the same extent as her boyfriend. She will be guilty of murder. Choice (A) is correct.
133
Q
  1. After weeks of deliberation, the defendant decided to rob a local liquor store. The defendant purchased a ski mask and then went into the liquor store, put his gun into the face of the clerk, and demanded the money from the register. Terrified, the clerk gave him the money from the register. The defendant pocketed the money, grabbed a bottle of tequila off the shelf for his celebration, and then ran back to the car and drove off. About an hour later, the defendant was slowly approaching his hideout in the outskirts of town when a young child suddenly darted in front of the defendant’s car. He applied the brakes but couldn’t stop in time. The car struck the child, killing her.For the child’s death, the defendant should be found guilty of which, if any, of the following crimes?(A) Felony murder.(B) Involuntary manslaughter.(C) Voluntary manslaughter.(D) No crime.
A
  1. (D) The child’s death was unintentional and did not occur as a foreseeable result during the commission of the robbery. The death of the child occurred an hour later, and the defendant was not speeding at the time. Her act of suddenly darting in front of the defendant’s car was not a foreseeable consequence of his get-away from the robbery. Therefore, choice (A) is incorrect. Choice (B) is incorrect because the defendant lacked the requisite mental state for involuntary manslaughter, namely, gross or criminal negligence. He was driving slowly and applied his brakes at the time of the accident. Choice (C) is incorrect, since the killingwas not intentional. By process of elimination, choice (D) is correct; the defendant will be guilty of no crime.
134
Q
  1. Knowing that a homeowner was away on vacation, two men decided to burglarize her home. Since they didn’t have a car, the men asked the defendant to drive them to the home. The two men did not tell the defendant what they intended to do there. The defendant drove them to the house. While the defendant waited in the car, the two men entered the home by using a master key to unlock the front door. They then stole several items of expensive jewelry from the bedroom.Concealing the jewelry in their pockets, they left the house and got back into the defendant’s car. Unaware of the theft, the defendant drove the two men back to their apartments. The next day, the two men pawned the jewelry for $5,000. Two weeks later, the two men gave the defendant $500 and told him for the first time about the burglary. The defendant kept the money and did not report the theft to the police.If the defendant is subsequently prosecuted, he should be found(A) guilty of receiving stolen property but not guilty of burglary.(B) guilty of burglary but not guilty of receiving stolen property.(C) guilty of burglary and of receiving stolen property.(D) not guilty of either burglary or receiving stolen property.
A
  1. (D) In order to be liable as an accomplice to the crime of another, one must (a) aid, abet, or encourage the perpetrator (b) with the intent thereby to promote or facilitate the commission of the crime. Several terms have been empLoyed by courts and legislatures in describing the kinds of acts which that will suffice for accomplice liability. The most common are “aid,”“abet,”“advise,”“assist,”“cause,”“command,”“counseL,”“encourage,”“hire,” and “induce.” The defendant is not subject to accomplice liability because he was unaware that the two men were burglarizing the home. The distracter in this question is the fact that the perpetrators gave the defendant $500 after pawning the jewels. Taking the money (two weeks afterward) does not subject the defendant to accomplice liability because he didn’t specifically intend to facilitate the commission of the crime at the moment of its inception. If anything, the defendant would be viewed as an accessory afterthe factfor accepting the money. Therefore, choices (A), (B), and (C) are incorrect.
135
Q
  1. A defendant and his friend were walking down the street when they ran into a victim. The friend turned to the defendant and said that the victim owed him some money. The friend then stopped the victim and demanded the money. The victim refused to pay the money. The friend suddenly said to the defendant, “Give me your gun. I’m going to blow this lowlife away.” The defendant gave his friend his gun, and the friend shot the victim to death. As the friend and the defendant were about to leave, the defendant turned to his friend and said, “Let me have my gun back. I think the creep’s still alive.” The defendant then fired two more shots into the victim’s body. Unknown to the defendant, the victim was already dead.Which of the following is the most serious crime that the defendant can be convicted of?(A) Attempted murder.(B) Murder.(C) Assault with a deadly weapon.(D) Concealment of a deadly weapon.
A
  1. (B) in this question, a Lot of students will incorrectLy choose choice (A) because they are aware that under the modern view, impossibility is not a defense (to a charge of attempt) when the defendant’s actual intent — not limited by the true facts unknown to him — was to do an act or bring about a result proscribed by law. Although this is true, choice (B) is the preferred answer because the defendant is an accomplice (or principal in the second degree) because he aided and abetted the friend by supplying him the gun to kill the victim. LaFave notes that to be a principal in the second degree, one must be present at the commission of a criminal offense and aid, counsel, command, or encourage the principal in the first degree in the commission of that offense. Choice (B) is correct because a principal in the second degree is subject to and accountable for the same crime(s) committed by the principal in the first degree. Choices (C) and (D) are incorrect. While both are crimes the defendant could be convicted of, they are less serious crimes than murder, and, therefore, not the strongest answers.
136
Q
  1. Co-defendants were dealers at a casino. They had been employed by the casino for four years. One day, they were unexpectedly fired by the casino’s new manager. Apparently, the casino hired the new manager to get rid of some of the old-time employees and replace them with new personnel at a lower wage. Angered by their firing, the codefendants vowed to get back at the casino.As their revenge, they decided to plant a bomb in the casino and demand $1,000,000. After receiving the money, they would then reveal the location of the bomb and provide details for defusing it. The co-defendants agreed that the casino should be given adequate warning so that nobody would be injured. In accordance with their plan, one of the co-defendants, who was an electronics expert, built the bomb himself. He alone then drove to the casino where he placed the bomb in a hallway closet. Shortly thereafter, the other co-defendant phoned the casino and made a demand for the money. He said a bomb was in the casino and that it would explode in 24 hours unless the money was paid. The casino treated their demand as a crank call and refused to make any payment.With their plans having gone awry, the codefendants agreed that one of them should return to the casino and defuse the bomb. As one of the co-defendants was driving back to the casino, the bomb exploded, killing 30 people. A subsequent investigation revealed that a faulty wire caused the bomb to detonate prematurely. A state statute provides that detonating or attempting to detonate a bomb or explosive device in or near a building or dwelling is a felony.If the co-defendants are charged with violating the aforementioned statute, which of the following statements is correct?(A) Both are guilty, because each participated in the planning of the crime.(B) The co-defendant who built the bomb is guilty, because he built and transported the bomb, but the other co-defendant is not guilty, because his phone call was not a substantial step in the furtherance of the crime.(C) Neither is guilty, because the casino’s gross negligence in failing to heed the telephone warning constituted an independent intervening cause of the explosion.(D) Neither is guilty, because they did not intend for anyone to be killed, but the bomb exploded prematurely.
A
  1. (A) Both co-defendants are guilty of violating the statute because although only one built and transported the bombing device, both participated in the planning and carrying out of the crime. The other co-defendant who did not build the bomb is guilty as an accomplice (or accessory before the fact). An individual is criminally liable as an accomplice if he gives assistance or encouragement, or fails to act where he has a legal duty to oppose the crime of another. Certainly, both co-defendants were responsible for the commission of the crime. Choice (B) is incorrect because they are both guilty for the reasons stated above. Choice (C) is incorrect because gross negligence on the part of the casino would not abrogate guilt in a criminal trial. Choice (D) is incorrect because the intentions of the defendants do not matter, as they built a bomb and placed it in the hotel.
137
Q
  1. Two co-defendants were investment brokers at a bank. They had been employed by the bank for ten years. One day, they were unexpectedly fired by the bank’s new manager. Apparently, the co-defendants’ investments had not been performing well in the economic downturn. The co-defendants, upset and humiliated by their firing, vowed to get back at the bank.As their revenge, they decided to plant a bomb in the bank and demand $2,000,000. After receiving the money, they would then reveal the location of the bomb and provide details for defusing it. The co-defendants agreed that the bank should be given adequate warning so that nobody would be injured.In accordance with their plan, the co-defendants consulted the Internet, where they found all the information they needed on how to build a bomb. They built the bomb and placed it in a supply closet. Shortly thereafter, they phoned the bank and made a demand for the money. They said a bomb was in the bank and that it would explode in 24 hours unless the money was paid. The bank refused to pay the money.Realizing that their plan had gone awry, they drove to the bank to attempt to defuse the bomb. As they were driving back to the bank, the bomb exploded, killing 30 people.If the co-defendants are charged with murder and conspiracy to commit murder, which of the following statements is correct?(A) Both co-defendants are guilty of felony murder, but neither is guilty of conspiracy to commit murder.(B) Both co-defendants are guilty of murder and conspiracy to commit murder.(C) Both co-defendants are guilty of conspiracy to commit murder, but not murder.(D) Both co-defendants are not guilty of murder and conspiracy to commit murder.
A
  1. (A) Both co-defendants would both be guilty of felony-murder. By constructing the bomb and placing it in the bank, they would be criminally liable for the explosion, even though it prematurely detonated. At common law, one whose conduct brought about an unintended death in the commission orattempted commission of a felony was guilty of (felony) murder. Today, many jurisdictions Limit the rule by requiring that the felony must be dangerous to life (e.g., arson, burglary, robbery, or kidnapping). Certainly, placing a bomb that was activated to detonate would be dangerous to human life. On the contrary, neither co-defendant is guilty of conspiracy to commit murder because they did not intend to kill anyone. Thus, there was no agreement to commit murder. Choice (B) is incorrect because neither defendant is guilty of conspiracy. Choices (C) and (D) are incorrect for the reasons stated above.
138
Q
  1. A defendant and a classmate were students at a state college. They were spending a leisurely afternoon listening to music in the defendant’s dorm room. They had just consumed a six-pack of beer when the classmate asked the defendant if there was anything else to drink. The defendant indicated there wasn’t and suggested that his classmate drive to the store and buy another six-pack. The classmate told the defendant that his car wasn’t working and asked the defendant if he could borrow his. The defendant assented and gave his classmate the keys to his car. The defendant knew that the classmate was drunk when he lent him his car.The classmate was driving to the store at an excessive rate of speed. As he approached an intersection, he was traveling at 70 M.PJ-I. When he came to the intersection, the light turned red. The classmate, who made no effort to stop in time, drove through the red light and collided with another car. The driver of the other car, who had entered the intersection with a green light, was killed in the accident.In the event that the defendant can be convicted of manslaughter, it will most likely be upon the basis of(A) responsibility for the accident as an accomplice.(B) recklessness in lending his car to his classmate.(C) joint venture in lending his car to his classmate for a common purpose.(D) vicarious liability for the conduct of his classmate.
A
  1. (B) Choice (A) is incorrect because in order to be an accomplice to a crime, one must (1) give assistance, and (2) have the intent to promote or facilitate commission of a crime. The defendant lacked the requisite mens rea, since he didn’t intend or knowingly encourage the classmate to commit a homicide. Choice (C) is incorrect because joint venture is a torts principle and does not extend to criminal liability. Choice (D) is not the best answer because it is a general principle of criminal law that one is not criminally liable for how someone else acts, unless he directs or encourages or aids the other so to act. Thus, unlike the case of torts, an employer is not generally Liable for the criminal acts of his employee, even though the latter does them in furtherance of his employer’s business (except in the case of a statutory crime where the legislature has provided otherwise). By process of elimination, choice (B) is the best answer.
139
Q
  1. One afternoon, police officers observed a man and a14-year-old girl smoking marijuana together. They arrested the man and charged him with the separate offenses of (1) possession of a controlled dangerous substance, and (2) contributing to the delinquency of a minor. The girl was also arrested and charged with being an accomplice to the crime of contributing to the delinquency of a minor.At trial, the girl’s best defense is that(A) smoking marijuana does not necessarily make her an accomplice to the crime.(B) the man, the adult principal, must be convicted before any prosecution can be maintained against a minor.(C) a minor cannot be prosecuted for an adult crime.(D) since the statute was designed to protect minors, the girl cannot be prosecuted as an accomplice.
A
  1. (D) Some crimes are defined in such a way that they may be directly committed only by a person who has a particular characteristic or occupies a particular position, as with adultery, which can be committed only by a married person. La Fave observes, however, that this has not prevented courts from concluding that others outside the legislative class may be guilty of these crimes (commonly referred to as Wharton rule crimes) on an accomplice theory, or that other persons may likewise be guilty of a conspiracy to commit such crimes. Thus, for example, an unmarried man may be convicted of conspiring with a married man that the latter commit adultery. See LaFave, Criminal Law ppgs. 491—492. It is important, however, to distinguish the above situation from the present example in which the girl is a member of a legislatively protected class. In accordance with the rule enunciated in Gebarbi v. United States, 287 U .S. 112 (1932), one who may not be deemed an accomplice to a crime (because a contrary holding would conflict with the legislative purpose) may likewise not be found to be a member of a conspiracy to commit that crime. In Gebarbi, the U.S. Supreme Court held that a woman could not be convicted of conspiracy to violate the Mann Act when the man transported the woman from one state to another for immoral purposes, because the Mann Act was designed to protect women. Likewise, in the present example, the girt cannot be prosecuted as an accomplice to the crime of contributing to the delinquency of minors, because the crime statute is designed to protect minors. Choices (A), (B), and (C) are all misstatements of law and, therefore, incorrect.
140
Q
  1. A defendant and a victim were roommates at college. The defendant was playing the role of a serial killer in the school play and asked the victim if she could leave their apartment for a few hours while she rehearsed. The victim agreed but returned to the apartment shortly thereafter because she was curious and wanted to see the defendant rehearse. The victim quietly came back into the apartment and hid behind some curtains in the dining room in order to watch the defendant rehearse. In her role, the defendant earned a large butcher’s knife, with which she stabbed the curtains at her “victim.” Unknown to the defendant, the victim, who was hiding behind the curtain, was stabbed in the chest and killed.If the defendant is prosecuted for the victim’s death, she should be found(A) guilty of murder.(B) guilty of manslaughter.(C) guilty of battery.(D) not guilty.
A
  1. (D) The defendant should be found not guilty of any of the crimes in answer choices (A), (B), or (C) because she lacked the requisite mental states. She clearly did not intend to murder, seriously injure, or harm the victim in any way. Therefore, she would not be guilty of murder or battery. The only close call here is involuntary manslaughter, which is an unintentional killing due to criminal negligence. However, in order to show criminal negligence, the defendant must have been aware of an unreasonable or high risk of death or serious bodily injury. Since the defendant was unaware of the risk, her conduct was not criminally negligent. Therefore, choice (D) is correct.
141
Q
  1. A man and a woman conspired to rob a local bank. After entering the bank, they pulled out guns and ordered everyone to the floor. They demanded money from the tellers but, unknown to them, one of the tellers activated a silent alarm. Within minutes, the bank was surrounded by police officers.A standoff ensued for several hours with the man and the woman barricaded inside the bank with several hostages. The man decided to try and make a getaway. He took the bank manager and, using her as a human shield, exited the bank. A shooting ensued, and both the man and the bank manager were killed by the police. This jurisdiction follows the agency theory of felony murder.The woman is charged with felony murder for the deaths of the man and the bank manager. The woman’s attorney has filed a motion to dismiss both charges. The court should(A) grant the motion with respect to the bank manager’s death, but deny the motion for the death of the man.(B) grant the motion regarding the man’s death, but deny the motion regarding the death of the bank manager.(C) grant the motion regarding the deaths of both the man and the bank manager.(D) deny the motion regarding the deaths of both the man and the bank manager.
A
  1. (B) The majority rule dealing with causation for felony murder is the agency theory. Under the agency theory, the killing must be caused by one of the felons engaged in the underlying felony. An exception to this rule is the shield theory. If one of the felons, as we have in our question, takes a person hostage or uses them as a shield and they are killed, all of the participating felons are liable for the death. Therefore, the woman will be liable for the death of the bank manager. However, the woman will not be liable for the death of the man under the agency theory because he was killed by the police. Therefore, choices (A), (C), and (D) are incorrect.
142
Q
  1. A boyfriend decided to rob a grocery store after he was let go during a labor dispute. The boyfriend asked his girlfriend to drive the getaway car, to which she agreed, on the condition that no loaded weapons were used during the robbery. On the day of the robbery, the boyfriend and his girlfriend drove to the grocery store. Unknown to the girlfriend, the boyfriend entered the store with a loaded gun. The boyfriend approached one of the cashiers and told him to fill a bag with all the money from the register. When the cashier refused, the boyfriend shot and killed him. He then turned to the next cashier and pointed his gun at her. The cashier suffered a heart attack and died.The boyfriend then took the money from the registers himself and exited the store. One of the customers had called the police, and they were waiting outside. A shootout ensued, and the boyfriend was killed by the police.If this jurisdiction follows the agency theory of felony murder, the girlfriend is guilty if how many counts of murder?(A) 1.(B) 2.(C) 3.(D) None.
A
  1. (B) Under the agency theory, the killing must be caused by one of the felons engaged in the underlying felony. Accordingly, the girlfriend is guilty of felony murder for the death of the first cashier because he was shot and killed by her boyfriend, who was her co-felon. The second cashier’s death from a heart attack was causally connected to the robbery because she was threatened by the boyfriend after witnessing the murder of her co-worker. Since her co-felon was criminally responsible for causing the second cashier’s death, the girlfriend would be guilty of felony murder. Finally, the girlfriend is not guilty of the boyfriend’s death because his killing was not caused by her or a co-felon. Therefore choice (B) is correct, and choices (A), (C), and (D) are incorrect.
143
Q
  1. One night, a defendant entered a liquor store, pointed a gun at the cashier, and demanded the money from the register. The cashier was nervous and moving slowly, and when she accidentally spilled all of the money out of the bag and onto the floor, the defendant shot her and left. The bullet lodged close to the cashier’s spine, paralyzing her. The defendant was subsequently prosecuted and convicted of robbery and assault with a deadly weapon.Two months later, the cashier decided she wanted an operation to remove the bullet. The doctors warned the cashier that the procedure was very risky and could result in her death. The cashier decided to go ahead with the surgery and, unfortunately, died as a result.If the defendant is now prosecuted for felony murder, he should be found(A) not guilty, because he was not the cause of the cashier’s death.(B) not guilty, because the subsequent prosecution violates double jeopardy.(C) not guilty, because the cashier decided to have the surgery, aware of the risks.(D) guilty, because he was the cause of the cashier’s death.
A
  1. (D) One of the elements required for the commission of a crime is causation. As a general rule, the defendant’s criminal act must be the proximate or legal cause of the injury suffered by the victim. When dealing with an intervening act, as we are here in our question with the surgery, look to see whether the intervening act was a coincidence or a response to the defendant’s prior actions. If the intervening act is a response to the defendant’s prior actions, then the defendant will be criminally liable for the consequences. In our question, the surgery was a response to the prior action of the defendant in shooting the cashier, and the defendant will be criminally liable for her death. Therefore, choices (A) and (C) are incorrect. Choice (B) is incorrect. An exception to double jeopardy exists where the defendant was not initialLy charged with a more serious crime because the additional facts necessary to sustain that charge had not yet occurred or been discovered by the prosecution. This is known as the Diaz exception, from the holding of the Supreme Court case of Diaz v. United States, 223 US 442 (1912).
144
Q
  1. A defendant was driving his new sports car at a high rate of speed on a busy city street in the middle of the day. A pedestrian was struck by the defendant’s car as she was crossing the street and was seriously injured.A criminal complaint was filed against the defendant and, at trial, he testified that he did not intend to injure anyone, but admitted that he was indifferent as to the consequences of driving recklessly.Based on the defendant’s actions and testimony, he should be found guilty ofa. attempted murder.b. attempted manslaughter.c. assault with the intent to cause serious bodily injury.d. battery.
A
  1. (D) Choices (A) and (B) are incorrect because the defendant did not intend to injure anyone. Remember that attempt, like all inchoate crimes, is a specific intent crime. Choice (C) is incorrect for similar reasons in that the crime is defined as assault with the intent to cause serious bodily injury. This would also be a specific intent crime for which the defendant does not have the requisite mental state required for conviction. Therefore, by process of elimination, choice (0) is the strongest answer. Battery is a general intent crime for which the defendant’s reckless conduct resulting in the injury to the pedestrian will suffice.
145
Q
  1. One night, a victim was in her home when she heard the doorbell ring. When the victim opened the door, she was confronted by three defendants. They pushed the victim inside her house and threatened her with bodily harm if she didn’t cooperate. After tying her up with ropes, the trio then proceeded to ransack the victim’s home.They then placed many of the victim’s valuables and other possessions into a large sack and hurriedly left her house.What crimes should the defendants be convicted of?(A) Assault, battery, and robbery.(B) Larceny, robbery, and burglary.(C) Robbery and burglary.(D) Robbery only.
A

145.(C) At common law, robbery is a trespassory taking and carrying away of the personal property of another with intent to steal it. Additionally, the property must be taken from the person or his presence and be accomplished by force or intimidation. In addition, robbery may be considered a greater crime than the sum of the two lesser crimes of larceny and assault (or battery). Therefore, since larceny and assault (or battery) merges into robbery, choices (A) and (B) are incorrect. Next, we must consider whether the defendants are guilty of burglary, which, at common law, consisted of the breaking and entering of a dwelling house of another in the nighttime with the intent to commit a felony therein. In order to constitute a breaking at common law, there had to be the creation of a breach or opening. The victim opened the door for the intruders, making choice (D) a tempting answer. However, choice (D) is incorrect because a “constructive breaking” occurred. When entry is gained by fraud, force, threat of force, or through a chimney, then a “constructive breaking” is deemed to have occurred, even though the occupant may have herself provided the opening. Here, the robbers fraudulently got the victim to open the door, and forced their way past her into her home. This constituted a “constructive breaking.” As a result, choice (C) is correct because the defendants are guilty of burglary and robbery.

146
Q
  1. A defendant was angry at his friend for marrying the defendant’s former girlfriend. As the friend was painting his house one afternoon, the defendant fired a shot from his gun at him. Although the shot missed the friend, the bullet struck and killed the friend’s daughter. The defendant is subsequently charged with the first-degree murder of the daughter. The relevant statutes in effect in this jurisdiction are as follows:Section 169: Murder in the first degree is the unlawful and intentional killing of a human being with malice aforethought.Section 170: Malice is expressed when there is manifested a deliberate intention to take away the life of another.Which of the following, if established, would provide the defendant with his best defense?(A) He intended to kill the friend and not the daughter.(B) He intended only to wound the friend.(C) He was unaware of the elements of malice.(D) The killing was the result of negligence in missing the friend.
A
  1. (B) Choice (A) is wrong because it is well settled in criminaL law that under the doctrine of transferred intent in the unintended-victim (or bad-aim) situation—where A aims at B but misses, hitting C—it is the accepted view that A is just as guilty as if his aim had been accurate. Thus, where A aims at B with a murderous intent to kill, but because of a bad aim he hits and kills C, A is uniformly held guilty of the murder of C. Choice (C) is likewise erroneous because as a general rule, it is frequently said that ignorance of the law is no excuse. Note that ignorance or mistake as to a matter of fact or law is a defense if it is shown that the defendant does not have the mental state required by law for the commission of that particular offense. In the present case, the defendant did have the requisite mens rea, because he intended to inflict serious bodily injury on the friend. Choice (D) is incorrect because it is commonly said in civiL and in criminal cases that one is presumed to intend the natural and probable consequences of his acts. Thus, LaFave points out, if one carefully aims a gun at his enemy and pulls the trigger; and the bullet strikes the enemy in the heart and kills him, we ought to conclude, in the absence of some other facts, that he intended to kill (though he spoke no words of intent at the time). Similarly, a special application of the presumption that one intends to produce the natural results of his actions is found in the deadly weapon doctrine applicable to homicide cases: one who intentionally uses a deadly weapon on another human being and thereby kills him presumably intends to kill him. In this regard, LaFave states that the deadly weapon doctrine is not a category, of murder separate from the intent- to-kill category, but, rather, the intentional use of a deadly weapon authorizes the drawing of an inference that the user intends to kill. It shouLd be noted that the intentional use of a deadly weapon which that produces death is not necessarily murder, since we still allow the user (defendant) a chance to convince the jury that in spite of his intentional use of a deadly weapon, he did not actually intend to kill. In light of this explanation, choice (D) is wrong because there is a presumption that the defendant intended to kiLl the friend under the deadly-weapon doctrine. Consequently, by process of elimination, choice (B) is the best answer.
147
Q
  1. An off-duty police officer was standing on a street corner waiting for a bus. A man came up from behind and stole the police officer’s wallet from his pants pocket. As the man was running away with the wallet, the police officer pulled out his service revolver. The police officer yelled at the man to stop and then fired several shots in the man’s direction. The police officer did not aim directly at the man but shot at the pavement intending to frighten him. One of the bullets ricocheted off the sidewalk and struck the man, killing him.The police officer is guilty of(A) assault with a deadly weapon.(B) involuntary manslaughter.(C) voluntary manslaughter.(D) murder.
A
  1. (D) This is a classic example of depraved-heart murder, defined as “extremely negligent conduct which creates not only an unjustifiable but also a very high degree of risk of death or serious bodily injury (though unaccompanied by any intent to kill) and which actually causes the death of another.” Criminal Law, pg. 541. According to LaFave, shooting a gun at a point near, but not aiming directLy at, another person involves a very high degree of unjustifiable homicidal danger, which wiLl do for depraved-heart murder. As a result, the police officer is guilty of murder. Choices (A), (B), and (C) are incorrect, as murder is the most serious crime for which he can be convicted and is, therefore, the strongest answer.
148
Q
  1. In which of the following situations is the defendant most likely guilty of manslaughter?(A) A defendant was a guide on a nature walk in a national park. The group was traversing a mountainous path when a mountain lion suddenly appeared. The defendant, who was carrying a loaded gun, knew that mountain lions and coyotes were in the area. The defendant saw that the mountain lion was about to attack one of the hikers. Although the defendant could have easily shot and killed it, he did nothing. The wild animal pounced on the hiker. As she was being savaged, the defendant and the other members of the group ran to safety.(B) A defendant, a registered nurse, asked her friend out to lunch. While eating, the friend suddenly began choking on a chicken bone. The defendant did nothing to help her friend. The friend choked to death.(C) A defendant, a physician, was walking home from his office one afternoon when he saw a car hit a man who was trying to cross the street. The victim was knocked to the ground and seriously injured, while the car sped away. The defendant, who could have saved the man’s life if he had treated him, continued walking home without rendering assistance. The man died from loss of blood.(D) A defendant took his four-year-old daughter out in the yard to play. As they were playing catch, the telephone began to ring. The defendant ran into the house to answer the phone. While he was inside, the daughter’s ball rolled into the street. As she went to retrieve it, the daughter was struck and killed by a car.
A
  1. (A) As with other common law and statutory crimes which that are defined in terms of conduct producing a specified result, a person may be criminally Liable when his omission to act produces that result, but only if (1) he has, under the circumstances, a legal duty to act, and (2) he can physically perform the act. For criminaL liability to be based upon a failure to act it must first be found that there is a duty to act—a legal duty and not simply a moral duty. According to LaFave, there are seven situations which that do give rise to a duty to act: (1) duty based upon relationship; (2) duty based upon statute; (3) duty based upon contract; (4) duty based upon voluntary assumption of care; (5) duty based upon creation of peril; (6) duty to controL conduct of others; and (7) duty of Landowner. In this example, choice (A) is correct because the defendant’s duty to act to protect the hiker probably would arise out of contract. Since the defendant was a guide at the park, he was employed to take affirmative action to protect hikers from such foreseeable dangers. This situation is analogous to a lifeguard employed to watch over swimmers at a beach. The lifeguard cannot sit idly by while a swimmer at his beach drowns off shore. Omission to do so may make the lifeguard liable for criminal homicide. Note that for a duty to act, by virtue of contract, the victim need not be one of the contracting parties. LaFave, Criminal Law, pp. 182—186. Choices (B) and (C) are incorrect because there would be no legal duty to act under those circumstances. Choice (D) is incorrect because, even though there is a legaL duty to act between parent and child, the father wasn’t there to prevent the harm and save his daughter.
149
Q
  1. In which of the following situations would the defendant most likely be found NOT GUILTY of robbery?(A) In a dark alley, a defendant approached the victim and said, “Give me the gold ring on your finger or I’ll shoot you with this gun.” The victim gave him the ring. However, the ring really belonged to someone else, as the victim was just borrowing it for the evening to impress his girlfriend. In addition, the gun that the defendant had in his possession was really a water pistol.(B) A defendant broke into a house and took a stereo system. After he had placed the system in his car and was about to leave, the homeowner came home and saw him. She raced to the car and started to hit the defendant through the open window in an attempt to get her stereo back. The defendant punched her in the nose and drove away with the system.(C) A defendant was walking behind a shopper in a mall when he suddenly reached for her gold chain, pulled it from her neck, and ran away into the crowd. The shopper suffered a slight cut on her neck where the chain broke.(D) A defendant picked the lock on an apartment door. The noise startled the tenant, who had been sleeping. The defendant overpowered the tenant, tied him up, and forced him to disclose where he kept his money. The tenant told the defendant to look in the kitchen cabinet, which he did. The defendant found $120 in cash, took the money, and left the apartment.
A
  1. (B) Robbery consists of all the six elements of larceny: a (1) trespassory (2) taking and (3) carrying away of the (4) personal property (5) of another (6) with intent to steal it, plus two additional elements: that (7) the property be taken away from the person or presence of another, and (8) the taking be accompLished by means of force or “putting in fear.” Choice (B) is correct, since the defendant should be found guilty only of the crime of larceny, not robbery. The defendant took the homeowner’s stereo system from her house without any force or intimidation. Since the “taking” and “asportation” elements of robbery did not coincide with the violence or intimidation elements, the defendant would not be guilty of robbery. All eight elements are necessary for a robbery conviction. Choice (A) is incorrect, since the defendant did commit a robbery upon the victim, even though the ring belonged to someone else. To prove robbery, it is not necessary to show that the personal property belonged to the victim at the time of the taking with force or intimidation. Choice (C) is wrong, since the “taking” of the gold chain was directly from the shopper’s person. Choice (D) is also incorrect, since the defendant entered the tenant’s apartment, overpowered him, and forced him to reveal where he had hidden the money. Thus, the taking was accompanied by violence, and the defendant would be found guilty of robbery.
150
Q
  1. An owner of a pharmaceutical company manufactures aspirin tablets that it sells in interstate commerce. A woman purchased a bottle of the owner’s aspirin from a drugstore. Shortly after taking two of the aspirin tablets, the woman became extremely ill and began having convulsions. She was rushed to the hospital, where it was determined that the aspirin tablets contained strychnine, a poisonous chemical.The owner of the pharmaceutical company is subsequently charged with violating a federal statute that makes it a misdemeanor to transport impure drugs in interstate commerce.The owner should be found(A) guilty, only if he had the authority and responsibility for packaging the aspirin tablets.(B) guilty, only if he knew or should have known that the aspirin tablets were poisonous.(C) guilty, only if he personally supervised the packaging of the aspirin tablets.(D) guilty, only if he knew that other customers had purchased poisonous aspirin tablets in the past.
A

15o.(A) There is a great deal of confusion in the substantive criminal law between strict hability crimes and vicarious liability crimes. A vicarious liability crime is one wherein one person, though without personal fault, is made liable for the conduct of another (usually his employee). It is common, however, for a vicarious liability statute to also impose strict liability; in such an instance, there is no need to prove an act or omission by the defendant-employer (one by his employee will do), and there is no need to prove mental fault by anyone. Some criminal statutes, for example, specifically impose criminal liability upon the employer for the bad conduct of his employee (e.g., “whoever, by himself or by his agent, sells articles at short weight shall be punished by . . . ,“ or “whoever sells liquor to a minor is punishable by - . . “). In construing statutes of this type, courts often jump to the unwarranted conclusion that a statute which that imposes strict liability must of necessity also impose vicarious liability. LaFave notes, however, that there is no basis for assuming that vicarious liability necessarily follows from strict liability. The better view, according to La Fave, is that an employer does not “allow” or “permit” his employee to do an act unless he knows of or authorizes it. Based on this analysis, the correct answer is choice (A). Choices (B), (C), and (D) are all misstatements of law and, therefore, incorrect.

151
Q
  1. A husband and wife were unhappily married. The husband was an alcoholic who physically abused the wife when he became drunk. After the husband beat up the wife one night, she decided to kill him. The wife, who was experienced in firearms, planned to shoot her husband while he slept. Before carrying out her plan, the wife took out a $1,000,000 life insurance policy on her husband, naming herself as beneficiary.Shortly thereafter, the husband and wife celebrated their 10th wedding anniversary. They went out to dinner and had a truly enjoyable evening together. After a great night, the wife experienced a change of heart and decided she really loved her husband and didn’t want to kill him after all. The wife confessed and told her husband about her plan, begging forgiveness. The husband was outraged. He proceeded to file for divorce and notified the police about his wife’s plan.If the wife is charged with attempted murder, she should be found(A) guilty, because she purchased the life insurance policy.(B) guilty, because she intended to kill her husband.(C) not guilty, because she did not perform a substantial step in carrying out the murder.(D) not guilty, because she effectively withdrew from the criminal endeavor.
A
  1. (C) In order to be guilty of attempt, the defendant must commit an act that constitutes a “substantial step” toward the commission of the crime. To be sure, bad thoughts alone cannot constitute a crime. However, precisely the kind of act required is not made very clear by the language which that has traditionally been used by the courts and legislatures. It is commonly said that more than an act of preparation must occur. The traditional attempt statute requires an “act toward the commission of’ some offense; “conduct which tends to effect the commission of’ a crime; or an act “in furtherance of’ or “tending directly toward” the commission of an offense. LaFave, Criminal Law, pg. 504. Choice (A) is incorrect. Simply taking out an insurance policy would not, by itself, constitute a ‘substantial step” in the commission of attempted murder. Rather, the wife must engage in some action directed toward the actual killing (e.g., placing poison in a glass or pulling the trigger of a gun). Choice (B) is incorrect because, as stated above, the wife has to do more then merely intend to kill her husband; there needs to have been a substantial step. Finally, choice (D) is incorrect, as withdrawal is not a defense to attempt.
152
Q
  1. A mother hated a girl because she always seemed to outperform her daughter and make her feel inferior. Fearing that the girl would beat out her daughter for the last cheerleading position, the mother decided to kill the girl. One night while the mother and her boyfriend were having dinner, the mother asked him to kill the girl. The mother handed her boyfriend a gun and gave him the home address where the girl lived. Unknown to the boyfriend, the mother gave him the wrong address. By mistake, the boyfriend went to the home of a family who lived across the street from the girl.The boyfriend rang the doorbell and a woman opened the door. The boyfriend asked the woman if her daughter was home. The woman called for her daughter to come to the front door. When she did so, the boyfriend shot and killed both the woman and her daughter.The mother should be found guilty for which, if any, of the following crimes?(A) Conspiracy and voluntary manslaughter.(B) Conspiracy, attempted murder, and two counts of murder.(C) Solicitation and two counts of murder.(D) Solicitation and attempted murder.
A
  1. (B) Under an accomplice (or co-conspirator) theory of liability, the mother is guilty of two counts of murder for the killings of the woman and her daughter. To be sure, a co-conspirator is criminally liable for all “natural and probable” consequences of the conspiracy. With respect to the issue of “foreseeable” crimes, since the mother supplied her boyfriend with a gun (and the wrong address), it was probable that he would kill someone at that location. Many students will be diverted from choosing choice (B) because they know that attempt merges with the completed crime. However, there is no merger, since the mother is guilty of the attempted murder of the girl and the murder of the woman and her daughter. If the boyfriend had killed the girl, then, of course, there would be a merger. Choices (C) and (D) are not the strongest answer choices because the mother’s actions in supplying her boyfriend with the gun raises her culpability to a level beyond the mere solicitation of a crime. Similarly, her actions raise her culpability above manslaughter, and she will be guilty of murder. Therefore, choice (A) is incorrect.
153
Q
  1. One evening, a defendant was at a party and offered to sell an ounce of marijuana to a partygoer. The partygoer agreed to purchase the marijuana and gave the defendant $200. In return, the defendant handed the partygoer a bag containing what appeared to be marijuana. At the time of the transaction, the defendant knew that the bag did not contain marijuana but, instead, was oregano.The defendant is guilty for which, if any, of the following crimes?(A) Solicitation and attempted sale of narcotics.(B) Attempted sale of narcotics and false pretenses.(C) False pretenses.(D) Solicitation, attempted sale of narcotics, and false pretenses.
A
  1. (C) For the crime of solicitation to be committed, it is only necessary that the defendant (with the requisite intent) have enticed, advised, incited, ordered, or otherwise encouraged another person to commit a crime. Since the defendant was not actually selling narcotics, he cannot be guilty of solicitation. Therefore, choices (A) and (D) are wrong. Choice (B) is incorrect because in United States v. Oviedo, 525 F. 2d 881 (1976), the Court held that legal impossibility precluded a defendant from being guilty of attempted sale of heroin that turned out to be a non-narcotic substance. Therefore, choice (C) is the best answer if the defendant intended to defraud the partygoer by selling oregano instead of marijuana.
154
Q
  1. Aman had just won $14,000,000 in the lottery. To celebrate his good fortune, the man took a group of friends to a bar for some drinks. At the bar, the man ordered a round of drinks for everyone. As the man knew, these specific drinks that he ordered were highly intoxicating. A few minutes after finishing his drink, the man stood up to go to the bathroom. As he did so, he became very woozy, lost his balance and fell onto a table. The table flipped over and knocked the customer sitting there backward off his chair. The customer’s head struck the floor with such force that he suffered a concussion.Thereafter, the man was charged with the crime of reckless endangerment, which is defined in this jurisdiction as “reckless conduct causing physical injury to another.” At his trial, the man called the bartender who served the drinks on the day in question to testif’. The bartender testified that the drinks she served the man and his friends were 95% alcohol. At the close of the bartender’s testimony, the state objected and moved to strike her testimony. The trial judge sustained the motion.Was the trial judge correct in excluding the bartender’s testimony?(A) Yes, because reckless endangerment is a general intent crime.(B) Yes, because the man’s intoxication was voluntary.(C) No, because the testimony was evidence that the man did not possess the requisite mens rea.(D) No, because the testimony was evidence that the man’s conduct was not voluntary.
A
  1. (B) As a general rule, voluntary intoxication is no defense for any crime requiring recklessness. Although recklessness requires that the defendant be aware of the risk which that his conduct creates, LaFave states that “if the only reason why the defendant does not realize the recklessness of his conduct is that he is too intoxicated, he is guilty of the recklessness which the crime requires.” LaFave, Criminal Law, pg. 392. As such, voluntary intoxication cannot be a valid defense. Therefore, the trial court judge was correct in excluding evidence of the man’s intoxication. Choice (B) is better than choice (A) because choice (A) merely states a given rule but does not address the intoxication defense. In this question, the key issue is whether the judge will admit or exclude evidence of the man’s intoxication as a defense for the crime of reckless endangerment. Choices (C) and (D) are incorrect, as they are not on point to the issue presented in this question.
155
Q
  1. A boyfriend was unhappy in his relationship with his girlfriend. When the girlfriend got drunk, she would become abusive and beat her boyfriend. During the course of their relationship, the beatings became more violent and more frequent. Unable to endure the physical abuse any longer, the boyfriend hired a hit man to kill his girlfriend. One night while the girlfriend was asleep, the hit man entered the home with a key given to him by the boyfriend and shot the girlfriend to death. Afterward, the boyfriend was prosecuted for murder as an accomplice.The defense sought to have the judge instruct the jury that the boyfriend acted in self-defense. Based on the given facts, should the judge give such an instruction to the jury?(A) No, because the boyfriend’s belief in the necessity of deadly force in self-defense was unreasonable.(B) No, because the boyfriend could have avoided the danger by safely retreating.(C) Yes, because a reasonable jury could conclude that the boyfriend acted in self-defense by using necessary force to protect himself from the girlfriend’s constant violent attacks.(D) Yes, because a criminal defendant’s Sixth Amendment right to a jury trial prohibits a court from refusing to submit affirmative defenses to the jury.
A
  1. (A) A person is privileged to use deadly force in self-defense if (a) she reasonablybelieves that she is in immediate danger of death or serious bodily injury and (b) the use of such force is necessary to avoid this danger. The given facts indicate that the boyfriend was not in immediate danger of unlawful bodily harm. In fact, the girlfriend was asleep when he arranged to have the hit man killed her. The judge should not instruct the jury on seLf-defense, because the boyfriend has failed to show that he was in immediate danger of serious bodily injury when the murder occurred. Choice (B) is incorrect because there is not a duty to retreat on the Multistate unless the question tells you the jurisdiction has such a duty. Choice (C) is incorrect for the reasons stated above. Choice (D) is incorrect, as it is a misstatement of law.
156
Q
  1. A defendant was waiting in line to enter a movie theater when he noticed a free movie pass on the ground. The pass had a space where the owner of the pass needed to write in his name. The defendant waited until he got to the cashier’s window and presented the pass, seeking a free admission to the movie, and claiming that he received it as a birthday present from a friend. The cashier told him that the passes were invalid unless the holder’s name was entered on the pass. The defendant wrote his name on the pass, gave it to the cashier, and was admitted to the theater.The defendant is guilty of(A) larceny.(B) false pretenses.(C) forgery.(D) no crime.
A
  1. (D) In order for the finder of lost or mislaid property to be guilty of larceny, two req uirements must be satisfied. The finder must, at the time of the finding, (1) intend to steal it, and (2) either know who the owner is or have reason to believe (from ear- markings on the property or from the circumstances of the finding) that he can find out the owner’s identity. Not only was the defendant unaware of who the owner of the pass was, he also could not determine the owner’s identity from the earmarkings of the property. Therefore, the defendant would not be guilty of larceny, and choice (A) is incorrect. Choice (B) is incorrect because false pretenses requires that the defendant, by his lies, obtain title to the victim’s property. The defendant did not obtain title to tangible property but merely free admission to a movie. Choice (C) is wrong because the defendant entered his own name on the pass, rather than forging someone else’s name or signature.
157
Q
  1. A defendant purchased a new car and offered to sell his old car to his neighbor, who always had admired it, for $1,900. The neighbor accepted. As part of their deal, the neighbor took immediate possession of the car and promised to send the defendant a check as soon as possible. One week later, the defendant received a check in the amount of $900. The neighbor had inadvertently sent the check for the incorrect amount. The defendant went ahead and changed the numbers on the check from $900 to $1,900 and changed the words “nine hundred dollars” to “one thousand nine hundred dollars.” The defendant then took the check to the neighbor’s bank for payment. When he presented it to the teller, she noticed that the check had been altered. The bank immediately contacted the police who arrested the defendant and charged him with forgery. This jurisdiction defines forgery as “making a material alteration in any writing with the purpose to defraud.”At trial, the defendant testified that he honestly believed that the neighbor had made a mistake in drafting the check and that the neighbor meant to make it out for $1,900, the contract price.If the jury believes the defendant, it should find him(A) guilty, because his belief was not reasonable. (B) guilty, because he made no effort to contact theneighbor to determine whether he had made a mistake in sending only $900.(C) not guilty, because it is irrelevant whether his belief was reasonable or not.(D) not guilty, because the neighbor did not intend to make a partial payment.
A
  1. (C) Where the accused has an honest and reasonable belief which, if true, would negate the mens rea of a particular offense, such a belief operates as a complete defense by negating a material element of the crime. The general rule is that a reasonable mistake negates a general intent, whereas a specific intent can be negated by a mistaken beLief, whether reasonable or unreasonable. Choices (A) and (B) are incorrect. The forgery statute in this question requires the making of a “material alteration in any writing with the purpose to defraud”— a specific intent. Therefore, if the jury believes the defendant’s story that he honestly believed the neighbor meant to draft the check for $1,900, the defendant should be found not guilty, regardless of whether his belief was reasonable or not. Choice (C) is correct. Finally, choice (D) is incorrect because the neighbor’s intent is not relevant to determining the defendant’s guilt.
158
Q
  1. A cashier worked part-time at a convenience store. As the cashier was sitting behind the cash register, he noticed a boy enter the store. While the boy walked through the store, the cashier watched him on a hidden camera located behind the cashier’s counter. Thereupon, the cashier saw the boy pick up two packs of baseball cards and place them in his coat pocket. The boy then walked to the rear of the store, where he picked up a package of cupcakes.The boy took the cupcakes to the front counter and paid the cashier for the cupcakes only. The boy started to walk out of the store when the cashier said, “Hey, kid, I’m not charging you for the baseball cards because you paid for the cupcakes. But next time I’m not going to let you get away with it.” The baseball cards sold for $5 per pack.The boy has committed which, if any, crimes?(A) Larceny.(B) Larceny by trick.(C) False pretenses.(D) No crime.
A
  1. (A) In order to be guilty of common law Larceny, the defendant need only carry away the victim’s property a slight distance (three inches will suffice) with the intent to steal. In this example, the boy took the baseball card packs, placed them in his coat pocket, and attempted to walk out of the store without paying. Clearly, sufficient asportation occurred. Students should not be distracted by the fact that the storekeeper gave the boy permission to take the cards. Actually, the crime was completed when the boy placed the cards in his pocket (without intending to pay for them), even before reaching the counter area. Choices (B) and (C) are incorrect because there was not a false representation of material fact made by the boy. Choice CD) is incorrect for the reasons stated above.
159
Q
  1. A tenant was cleaning out the bedroom of his apartment as his lease had expired. He carried out his waterbed, dresser, and all of his clothes. As the tenant inspected the room one last time, he noticed a bookcase that he had screwed into the wall when he first moved in. While he pondered whether to remove the bookcase, the tenant concluded that it constituted a fixture and, therefore, was a permanent part of the realty. Nonetheless, the tenant decided that the bookcase would fit nicely in his new apartment, so he removed it from the bedroom wall. When the landlord learned that the tenant had removed the bookcase, he contacted the tenant and demanded that he return it to the apartment. The tenant refused.If the tenant is subsequently prosecuted for larceny, he will most likely be found(A) guilty, because the chattel was the personal property of the landlord.(B) guilty, because severance of the bookcase from the wall was sufficient asportation to constitute larceny.(C) not guilty, because the tenant cannot be found guilty of theft for removing his own property.(D) not guilty, because the bookcase was a fixture.
A
  1. (D) At common law, larceny was limited to the taking of tangible personal property. Since larceny was limited to the theft of goods and chattels, one cannot be guilty (of larceny) for misappropriations of real property, such as trees, crops, minerals, and fixtures. In the present example, the facts clearly state that the bookcase was regarded as a fixture. The tenant cannot be guilty of common law larceny for the removal of the bookcase. Therefore, choices (A) and (B) are incorrect. Choice (C) is incorrect because the book case was no longer the tenant’s property.
160
Q
  1. A man and a woman were driving eastbound along a steep two-lane winding highway. The woman was driving in front of the man. As the man approached the rear of the woman’s car, he became impatient and tried to pass her along a section of the highway designated as a no-passing zone. When the man swerved his car into the westbound lane to pass the woman, he didn’t see another driver, who was rounding a curve in the westbound lane. In order to avoid a head-on collision with the man, the other driver swerved his car to the shoulder of the roadway. The other driver, however, was unable to control his car along the narrow strip of the shoulder, and it fell down the steep mountain.The man and the woman both saw the other driver’s car slide down the hillside. They immediately stopped their vehicles and walked over to the edge of the roadway looking for the car. They saw the other driver’s car overturned in a gully about 200 feet down the mountain. The man and the woman heard the other driver moaning and calling for help. However, the man and the woman failed to provide any assistance. They walked back to their cars and drove off. Although the man and the woman passed through a business area a short while later, they neither reported the accident nor sought aid for the other driver. Hours later, the other driver died from injuries suffered in the accident.If the man and the woman are subsequently prosecuted for the other driver’s death, the most likely outcome would be thata. the man and the woman are both guilty of manslaughter.b. the man is guilty of manslaughter only. c. the woman is guilty of manslaughter only. d. neither the man nor the woman is guilty of manslaughter.
A

i6o. (B) For criminal liability to be based upon a failure to act, it must first be found that there is a legal duty to act. Such a legal duty can arise where the defendant either intentionally or negligently creates the victim’s peril. By attempting to overtake the woman’s car in a “no-passing” zone on a steep, winding highway, the man acted negligently, which created the peril. Since the man created the peril, he had a legal duty to render assistance. His failure to act establishes Liability based on criminal negligence, and he will be guilty of manslaughter for the other driver’s death. Choice (B) is correct. The woman will not be guilty of manslaughter; she neither created the victim’s peril norvoluntarily undertookto help the victim. No duty to act arose merely by the fact that the woman stopped her vehicle, walked to the edge of the roadway, and heard the victim’s call for help. As to the woman, a general rule applies, namely that one has no legal duty to aid another person in peril, even if that aid can be rendered without danger or inconvenience. Therefore, choices (A), (C), and (D) are incorrect.

161
Q
  1. A college student who was pledging a fraternity was required to steal a yield sign from a street intersection. At 10:00 P.M. one evening, the student went to the corner and removed the yield sign from the intersection. Motorists driving northbound were required to yield to other vehicles entering the intersection. Two hours later, a man was driving northbound toward the intersection after having just smoked marijuana. Failing to yield, the man crashed into a vehicle, killing the other driver.If the student and the man are prosecuted for the other driver’s death, who shall be held criminally liable?(A) The student only, because his conduct was the legal cause of the other driver’s death.(B) The man only, because he was high on marijuana when he collided with the other driver’s vehicle.(C) The man and the student, because their acts were concurrent causes of the other driver’s death.(D) Neither the man nor the student, if the other driver had the last clear chance to avoid the accident.
A
  1. (A) Four elements are generally required for the commission of a crime: (1) actus reus; (2) mens rea; (3) causal connection; and (4) harm or iniury to the victim. In order to be guilty of a crime, the defendant’s conduct must be the legal, or proximate, cause of victim’s harm. The problem of legal causation arises in both tort and criminal law situations. Here, the man’s failure to yield was the actual (or factual) cause of the accident, but the student’s removal of the yield sign was the legal cause. Consequently, choice (A) is correct, and the student will be held criminally responsible for the other driver’s death. Therefore, choices (B), (C), and (D) are incorrect.
162
Q
  1. A breeder and owner of vicious guard dogs trained his dogs to attack strangers at night. He often sold and leased his guard dogs to various business and factory owners who used the guard dogs to frighten away intruders from entering their premises at night. One evening, the breeder was in the back yard of his home training three of his guard dogs. The back yard was enclosed with a chain link fence and a latched gate that prevented the dogs from running out. After the training session, the breeder opened the gate and permitted the dogs to run loose in his front yard. Minutes later, a man was walking along the sidewalk in front of the breeder’s house when he was attacked by one of the dogs. The man suffered severe injuries and died as a result of the attack.The breeder should be found guilty of(A) murder.(B) involuntary manslaughter.(C) voluntary manslaughter.(D) reckless endangerment.
A
  1. (A) According to LaFave, “extremely negligent conduct, which creates what a reasonable man would realize to be not only an unjustifiable but also a very high degree of risk of death or serious bodily injury to another though unaccompanied by any intent to kill or do serious bodily injury and which actually causes the death of another” constitutes “depraved-heart” murder. See Criminal Law, p.541. Since the breeder had trained his guard dogs to attack at night and then opened the gate to let them run loose, he will be criminally responsible for the killing of the man. Such conduct on the breeder’s part is more extreme than the gross or criminal negligence standard sufficient for involuntary manslaughter. The breeder will be liable on the theory of depraved-heart murder. While one could argue that the breeder will be liable for the crimes in choices (B), (C), and (D), the most serious crime is in choice (A), which is, therefore, the strongest answer.
163
Q
  1. A collector had an extensive collection of sports memorabilia, which included baseball cards, autographed bats, balls, and old uniforms. The collector would often attend regional shows where he would display his vast collection and meet with other sports enthusiasts.One day at a sports memorabilia show, the defendant approached the collector with an old baseball glove with what appeared to be an authentic signature of a famous player. Unknown to the collector, the defendant signed the glove himself. The defendant had practiced the distinctive signature of the famous player for several hours before signing the glove with an extremely good likeness of his signature.The collector examined the glove and asked the defendant if the signature was authentic. The defendant replied that he had purchased the glove at an auction and could not vouch for the authenticity of the signature. The collector paid the defendant for the glove. Later, the collector took the glove to a handwriting expert who informed the collector that the signature on the glove was a very skillful fake, but a fake, nonetheless.The defendant is guilty of(A) false pretenses but not forgery, because the glove is not a document with legalsignificance.(B) forgery but not false pretenses, because thedefendant did not vouch for the authenticity ofthe glove.(C) false pretenses and forgery.(D) neither false pretenses nor forgery.
A
  1. (C) Forgery is a crime aimed primarily at safeguarding confidence in the genuineness of documents relied upon in commercial and business activity. Though a forgery, like false pretenses, requires a lie, it must be a lie about the document itself: the lie must relate to the genuineness of the document. According to LaFave, if the forger is successful in passing the forged document, receiving property or money for it, he is no doubt guilty of the crime of false pretenses in addition to that of forgery. Therefore choices (A), (B), and (D) are incorrect.
164
Q
  1. A man was visiting his friend at the friend’s cabin. The man decided to surprise the friend with some fireworks, so he purchased some fireworks and hid them under his friend’s chair on the friend’s porch. That evening, the friend went outside to smoke and dropped his unfinished cigarette next to his chair on the porch. The cigarette ignited the fireworks, and the fireworks immediately exploded, causing extensive damage to the friend and to his cabin. The man was arrested and is being prosecuted for arson.Will the man be found guilty of arson?(A) No, because the man never intended to hurt the friend.(B) No, because the man did not intend to burn down the cabin and was not guilty of wanton and willful misconduct.(C) Yes, because the man indirectly set fire to the friend’s cabin.(D) Yes, because the injury to the friend and to his cabin was the natural result of the man’s actions.
A
  1. (B) To be guilty of common-law arson one must burn the dwelling house of another with malice. Malice includes the intent to burn or wanton and willful misconduct that creates a plain and strong likelihood that a protected structure will be burned. To engage in wanton and willful misconduct, the defendant must have recklessly ignored an obvious risk. Because the man was only intending to hide the fireworks under a chair, he could not have been aware of the damage that would likely result. Additionally, hiding fireworks under a chair is not an obviously dangerous act. Thus, the man would not possess the requisite malice to support an arson conviction. Choice (A) is incorrect. The man’s intentions toward the friend would be relevant if he were charged with battery or another crime against the person, but arson is a crime against structures, and the man’s intent toward the person of the friend is immaterial. Choice (C) is incorrect. Because the man lacked the mental element of malice necessary to support a conviction for arson, the fact that he was the indirect cause of the burning of the cabin is not sufficient to overcome the lack of the requisite mental element. As such, he is not guilty of arson. And Choice (D) is incorrect. No matter how one might argue that this was the natural result of the man’s actions, such a result was certainly not obvious. As such, the man cannot be held guilty of wanton and willful misconduct. Additionally, it is arguable that a fire is not the natural result of hiding fireworks under a chair on the porch.
165
Q
  1. There is a state statute making it a misdemeanor “to falsely report a fire either intentionally or recklessly.” There were three college roommates who lived together in a small apartment. Two of the roommates decided to play a practical joke on the other roommate, which they liked to do from time to time because he was gullible. The two roommates were seated in the living room of their apartment. The other roommate was in an adjoining room and within earshot of the two roommates.Knowing that their roommate could hear their conversation, the two roommates falsely stated that a fire had been set at the student center at the college. After overhearing this conversation, the other roommate phoned the fire department and reported this information. Several fire trucks were dispatched to the college and determined the information to be false.If the two roommates are prosecuted for violating the aforementioned statute, they should be found(A) guilty, because they caused the false report to be made.(B) guilty, because they are accomplices to their roommate.(C) not guilty, because they didn’t report the information to the fire department themselves.(D) not guilty, because they didn’t knowingly believe that their roommate would report the information to the fire department.
A
  1. (A) Choice (B) is wrong because, in order to be subject to accomplice liability, the defendant(s) must intentionally encourage or assist another in the commission of a crime as to which the accomplice has the requisite mental state. The facts don’t necessarily indicate that the roommates intended to have their roommate falsely report a fire to the fire department, since they were playing a practical joke. Rather, their conduct was reckless. Their roommate is an innocent agent. The roommates would be guilty as the principals for their roommate’s false report. Because they are “principals,” they cannot be guilty as “accomplices,” so choice (B) is incorrect. Choice (C) is wrong because the roommates were responsible for the crime being committed and are, thus, subject to criminal liability as parties to the crime. Choice (D) is incorrect because the language of the statute does not require that the false report be made knowingly.
166
Q
  1. Two defendants were playing poker one night with some friends. Everyone that was playing, including the defendants, was drinking heavily. After losing a large amount of money, the defendants accused the victim of cheating. When the victim tried to leave, the defendants beat him with their fists and, as the victim fell to the ground, they continued by kicking him in the head. The victim suffered a fractured skull in the beating and died as a result of severe head trauma. Thereafter, the defendants reached into the victim’s pocket and stole his wallet.The defendants were arrested and charged with the murder of the victim. Prior to trial, the prosecutor admitted that it was impossible to determine which defendant was responsible for actually fracturing the victim’s skull.If the defendants are prosecuted for felony murder, which of the following would provide their best defense?(A) The jurisdiction adheres to the inherently dangerous felony requirement.(B) The jurisdiction follows the independent felony murder rule.(C) The jurisdiction follows the proximate cause theory.(D) The jurisdiction follows the agency theory.
A

i66. (B) In certain situations, like we have here in our question, the homicide will occur first before the underlying felony is committed. In jurisdictions following the independent felony murder rule, the defendant is not guilty of felony murder if the intent to commit the underlying felony is formed after the killing has occurred. Therefore, choice (B) is the best defense, as the defendants only decided to steal the victim’s wallet after he lay dead on the ground. Choice (A) is incorrect because robbery would qualify as an inherently dangerous felony. Choices (C) and (D) are incorrect because the defendants would be guilty under both theories, as they were the proximate cause of the victim’s death, and a third party did not kill the victim.

167
Q
  1. A defendant hated a victim and decided to kill him. The defendant put a loaded gun in his coat pocket and went outside for a walk, not sure if and when he may see the victim. The defendant passed by a bar and decided to go inside for a drink. While in the bar, he drank several bottles of beer and became intoxicated. While walking home from the bar, the defendant bumped into the victim. As soon as the defendant recognized him, he pulled out his gun and shot and killed the victim.In this jurisdiction, first-degree murder is defined as a premeditated and deliberate killing. All other forms of murder are second-degree murder.The defendant is guilty of(A) first-degree murder.(B) second-degree murder.(C) involuntary manslaughter.(D) voluntary manslaughter.
A
  1. (A) Choice (B) is incorrect. In order for intoxication to negate premeditation and deliberation, it must rob the defendant of his ability to premeditate and deliberate. Here, the defendant had premeditated the killing prior to becoming intoxicated, so it won’t be a defense to first-degree murder that at the time of the killing, he could no longer premeditate or deliberate due because to his intoxication. Choices (C) and (D) are incorrect for the reasons stated above.
168
Q
  1. Two women decided to steal some clothing from their favorite store. One of the women carried a large shopping bag containing a gun into the store. The other woman did not know that her partner had a gun in the shopping bag. After the women placed three dresses into the shopping bag, a clerk became suspicious and approached the women. The women tried to leave the store and, as they did, the clerk grabbed the bag, which fell to the floor. When the bag hit the floor, the gun discharged, killing the clerk.Which of the following would be the women’s best defense to the charge of felony murder?(A) They committed a larceny by placing the three dresses in the shopping bag.(B) The jurisdiction requires the killing to be independent of the felony.(C) The jurisdiction follows the agency theory of felony murder.(D) The killing, was accidental and unintended.
A

i68. (A) One of the elements of felony murder is that the underlying felony must be inherently dangerous. The best defense for the women would be that the underlying crime is larceny, which is not inherently dangerous. Choice (B) is incorrect because the killing was independent of the underlying crime. Choice (C) is incorrect. Under the agency theory, a defendant is guilty of felony murder only when the killing is caused by one of the co-felons. Here, one of the defendants caused the killing by placing the gun in the shopping bag, so this would not be the women’s best defense. Finally, choice (D) is incorrect because the killing often is accidental or unintentional. That is the purpose behind the felony murder rule; to hold defendants accountable for killings caused during the commission of inherently dangerous felonies even if they were accidental or unintentional.

169
Q
  1. A girlfriend lived with her boyfriend in a small apartment. The boyfriend was a gun enthusiast who kept a collection of antique pistols and firearms in the apartment. The boyfriend was also an avid baseball fan and had a fanatic devotion to his favorite team. One evening, the boyfriend was watching his beloved team play their archrival on television. His favorite team lost in extra innings. After the game, the boyfriend became despondent and told his girlfriend that he was so disgusted that he was going to kill himself. The boyfriend took one of the pistols from his gun collection and shot himself in the head. As the boyfriend fell to the floor, wounded, his girlfriend couldn’t believe her eyes. At first she thought that she should call 911 for an ambulance. But she was afraid that the police might think that she was responsible for the shooting. Consequently, she decided to do nothing. She proceeded to leave the apartment and spent the night at her mother’s home.The boyfriend did not immediately die from the gunshot wound. A subsequent medical examiner’s report concluded that the boyfriend died from loss of blood approximately two hours after the shooting. If the girlfriend had sought immediate medical assistance, the boyfriend would have lived.The girlfriend is subsequently prosecuted for her boyffiend’s death. She should be found(A) guilty of first-degree murder.(B) guilty of second-degree murder.(C) guilty of involuntary manslaughter.(D) not guilty, because she had no legal duty to render assistance.
A
  1. (C) According to LaFave, for criminal liability to be based upon a failure to act, it must first be found that there is a legal duty to act, not simply a moral duty. Generally, one has no legal duty to aid another in peril, even when that aid can be rendered without danger or inconvenience. However, the common law imposes an affirmative duty upon persons standing in certain personal relationships to other persons, as upon (1) parents to aid their small children; (2) husbands to aid their wives; (3) ship’s captains to aid their crews; and (4) employers to aid their employees. The novel question presented here is whether a roommate is under a legaL duty to render assistance? According to LaFave, “if two people, though not closelyrelated, live together under one roof, one may have a duly to act to aid the other who becomes helpless.” LaFave, Criminal Law, pg. 204. Therefore, choice (D) is incorrect. Since the facts indicate that the girlfriend and boyfriend were living together and carrying on a relationship, the girlfriend would have a duty to render assistance to her boyfriend. In situations where the victim dies on account of an omission to act, the defendant is generally guilty of involuntary manslaughter. Therefore, choices (A) and (B) are incorrect.
170
Q
  1. A federal agency has been established for the purpose of stabilizing, supporting, and protecting farm income and prices. The agency is authorized to enter into and carry out such contracts or agreements as are necessary in the conduct of its business.An export company entered into a contract with the federal agency to transport a variety of goods to foreign countries. Thereafter, the federal agency suspended the export company pending an investigation into possible misuse of official inspection certificates relating to commodities exported to a foreign country. During an investigation relevant to the charge, the president of the export company presented information to the federal agency. Subsequently, the president was charged with violation of a federal statute that states:“It is a felony for any person in a matter within the jurisdiction of a federal agency to knowingly make a false statement.”At the president’s trial, the government presented evidence to show that he falsified bills of lading and embezzled over $500,000 of surplus commodities for export. The president testified that he did not know that he was dealing with a federal agency and, hence, the federal government would have jurisdiction over his company’s export contract with the federal agency.As to the significance of the president’s ignorance, the court should instruct the jury that it may convict the president(A) if it finds that the president knew that he was doing business with a federal agency.(B) if it finds that the president was aware that he might have been doing business with a federal agency.(C) if a reasonable person would have been aware that he might have been doing business with a federal agency.(D) even if there was no reason the president should have known that he was doing business with a federal agency.
A
  1. (D) This question deals with violation of a federaL criminal statute. According to LaFave,federal criminal statutes define offenses in terms of substantive misbehavior, andthe matter of jurisdictional requirements are dealt with separately. The governmentis relieved of any burden of showing that the defendant knows about the special factwhich that results in federal jurisdiction. LaFave, Criminal Law, p. 113. Therefore,the jury should be instructed that it may convict the company’s president regardlessof his lack of knowledge that he was doing business with a federal agency. The onlyelement of knowledge required by the federal statute is that the defendant knowingly makes a false statement. Therefore, choices (A), (B), and (C) are incorrect.
171
Q

fill

A
  1. (A) According to the given facts, the defendant agreed to provide the man with information about how to break the combination to the safe. As such, the defendant isguilty of conspiracy because he entered into the agreement with the intentthatthecrime be committed. The main problem students have with this question is whetherthe woman is also a co-conspirator. Surely, the woman did not explicitly take partin the conversation (or agreement) between the defendant and the man. However,according to LaFave, “a mere tacit understanding will suffice and there need notbe any written or verbal-statement which expressly communicates an agreement.”Criminal Law, pg. 532. In this regard, the agreement need not be shown to havebeen explicit; it can be inferred from the facts and circumstances of the case. Therefore, choice (B) is incorrect. Undoubtedly, the woman was part of the conspiracybecause she approached the defendant the previous day and solicited his aid.Choices (C) and (D) are incorrect for the reasons stated above.
172
Q

fill

A
  1. (A) The issue presented is, is it possible for a person to become a party to the crime of conspiracy even in the absence of any agreement on his part? The answer is yes. According to LaFave, a person with knowledge of the conspiracy “who intentionally gives aid to the conspiratorial objective” is guilty of conspiracy. Criminal Law, pg. 534. Therefore, choices (C) and (D) are incorrect. Choice (B) is incorrect because the conspiratorial agreement involved three persons, namely, the mechanic, the janitor, and the defendant. Even if the mechanic is found innocent of conspiracy, the janitor and the defendant nonetheless could be convicted, which would satisfy the plurality requirement. At common Law, there must be two or more guiLty parties to a conspiracy. Exam Tip: Under the Model Penal Code (modern view), a unilateral agreement will suffice for a conspiracy conviction. For example, if A and B are charged with conspiracy and A is acquitted, B could still be convicted of conspiracy. However, you are advised to answer the Criminal LawMBE questions based on the common law (rather than the Model Penal Code), unless the question states otherwise.
173
Q

fill

A
  1. (A) Receiving stolen property is defined typically by statute as the receiving of stolenproperty knowing that it is stolen. Although most statutes do not specifically mention it, the receiver must, in addition to knowing the property is stolen, intend todeprive the owner of his property. Also, the stolen character of the property and thereceiver’s receipt of the property must coincide in point of time. In other words, ifthe property has been recovered by the owner or by the police, it has lost its statusas “stolen” property, and there can be no crime. When the informer agreed to actas an undercover police informant, the police arranged for him to set up a pawnbroker’s shop. The stolen television sold from the man to the informer was reported tothe police and held for eventual sale to the defendant. At this point, the televisionwas no longer “stolen” property, but “recovered” property. Therefore, the defendant did not receive “stolen” property, and she will be found not guilty. Therefore,choices (C) and CD) are incorrect. Choice (B) is incorrect because the defendant wasnot entrapped. She was already predisposed to purchase the television withoutpolice inducement.
174
Q

fill

A
  1. (B) Certainly, the defendant’s best argument that the homicide should be manslaughter rather than murder is that he acted with adequate provocation in attempting tokill the victim. Voluntary manslaughter consists of an intentional homicide committed under extenuating circumstances that mitigate, though they do not justifyor excuse, the killing. The principal extenuating circumstance is the fact that thedefendant (when he killed the victim) was in a state of passion engendered in himby adequate provocation. The facts indicate that the victim punched the defendantin the face several times immediately before the shooting. This battery would constitute “reasonable provocation,” causing the defendant to lose his normal self-control. Choice (A) is incorrect because the defendant cannot argue that beingpunched in the face would give rise to the use of deadly force and is, therefore,self-defense. Choice (C) is incorrect because, at common law, voluntary intoxication cannot mitigate murder to manslaughter. Choice (D) is incorrect. Just becausea killing is unintentional doesn’t necessarily mean that it will be mitigated frommurder to manslaughter. There are forms of murder, liked depraved-heart, or intentto cause serious bodily harm, which do not require the intent to kill.
175
Q

fill

A
  1. (C) At common law, burglary is defined as the (1) breaking and (2) entering of(3) thedwelling house (4) of another (5) with intent to commitafelonytherein. Choice (A)is incorrect because the defendant mistakenly entered the home, not intending tocommit a larceny or felony therein. Choice (B) is incorrect because, first, an antiquestore is notviewed as a dwelling house and, second, the defendant did not possessthe intent to steal. Choice (D) is wrong because the defendant, again, did not intendto steal or permanently deprive the neighbor of the chain saw. By process of elimination, choice (C) is the best answer. Be advised that gaining entry to the home byfraud would constitute a “constructive breaking,” which would satisfy the breakingrequirement for burglary.